Tuyển tập Đề thi vào lớp 10 chuyên Toán trường THPT chuyên Khoa học Tự nhiên - ĐHQG Hà Nội

139 10 1
Tuyển tập Đề thi vào lớp 10 chuyên Toán trường THPT chuyên Khoa học Tự nhiên - ĐHQG Hà Nội

Đang tải... (xem toàn văn)

Tài liệu hạn chế xem trước, để xem đầy đủ mời bạn chọn Tải xuống

Thông tin tài liệu

b) Giả sử đường tròn   K ngoại tiếp tam giác EMN cắt đường thẳng AC tại Q khác N.. Các đường thẳng PB, PC lần lượt cắt AD tại AD tại M, N. Đường trung trực của AM cắt đường thẳng AC, [r]

(1)



Sưu tầm tổng hợp

BỘ ĐỀ THI TOÁN VÀO 10

KHOA HỌC TỰ NHIÊN HÀ NỘI

Tài liệu tổng hợp

(2)

TUYN TẬP ĐỀ THI TOÁN

VÀO LỚP 10 CHUYÊN KHOA HỌC TỰ NHIÊN HÀ NỘI LI NÓI ĐẦU

Các vịphụhuynh thầy dạy tốn có thểdùng có thểdùng tuyển tập đềtốn để giúp em học tập Hy vọng Tuyển tập đềthi tốn vào lớp 10 chuyên Đại học khoa học tự nhiênnày sẽcó thểgiúp ích nhiều cho học sinh phát huy nội lực giải tốn nói riêng học tốn nói chung.

Mặc dù có sựđầu tư lớn vềthời gian, trí tuệsong khơng thể tránh khỏi hạn chế, sai sót Mong sựgóp ý thầy, cô giáo em học!

Chúc thầy, cô giáo em học sinh thu kết quảcao từ bộđềnày!

Nhằm đáp ứng nhu cầu giáo viên toán THCS học sinh luyện thi vào lớp 10 mơn tốn, website thuvientoan.net giới thiệu đến thầy cô em bộ đềthi vào lớp 10 chuyên Đại

học khoa học tựnhiên Hà Nội Đây bộđề thi mang tính chất thực tiễn c ao, g iúp c ác t hầy cô em học sinh luyện thi vào lớp 10 có tài liệu bám sát đề thi để đạt thànhtích cao, mang lại vinh dựcho thân, gia đình nhà trường Bộ đề gồm nhiều Câu tốn hay các thầy cả nước s ưu t ầm v s t ác, ô n l uyện qua sẽ giúp em phát triển t ư d uy môn t ốn t ừđó thêm u thích học giỏi mơn học này, tạo tảng để có kiến thức tốt đáp ứng cho việc tiếp nhận kiến thức ởcác lớp, cấp học nhẹnhàng hiệu quảhơn.

(3)

MỤC LỤC

PHẦN 1: ĐỀ THI

Đề số Đề thi Trang

1. Đề thi vào lớp 10 chuyên Đại học KHTN Hà Nội năm 2019 (vòng 1)

2. Đề thi vào lớp 10 chuyên Đại học KHTN Hà Nội năm 2019 (vòng 2)

3. Đề thi vào lớp 10 chuyên Đại học KHTN Hà Nội năm 2018 (vòng 1)

4. Đề thi vào lớp 10 chuyên Đại học KHTN Hà Nội năm 2018 (vòng 2)

5. Đề thi vào lớp 10 chuyên Đại học KHTN Hà Nội năm 2017 (vòng 1)

6. Đề thi vào lớp 10 chuyên Đại học KHTN Hà Nội năm 2017 (vòng 2)

7. Đề thi vào lớp 10 chuyên Đại học KHTN Hà Nội năm 2016 (vòng 1)

8. Đề thi vào lớp 10 chuyên Đại học KHTN Hà Nội năm 2016 (vòng 2)

9. Đề thi vào lớp 10 chuyên Đại học KHTN Hà Nội năm 2015 (vòng 1)

10. Đề thi vào lớp 10 chuyên Đại học KHTN Hà Nội năm 2015 (vòng 2)

11. Đề thi vào lớp 10 chuyên Đại học KHTN Hà Nội năm 2014 (vòng 1)

12. Đề thi vào lớp 10 chuyên Đại học KHTN Hà Nội năm 2014 (vòng 2)

13. Đề thi vào lớp 10 chuyên Đại học KHTN Hà Nội năm 2013 (vòng 1)

14. Đề thi vào lớp 10 chuyên Đại học KHTN Hà Nội năm 2013 (vòng 2)

15. Đề thi vào lớp 10 chuyên Đại học KHTN Hà Nội năm 2012 (vòng 1)

16. Đề thi vào lớp 10 chuyên Đại học KHTN Hà Nội năm 2012 (vòng 2)

17. Đề thi vào lớp 10 chuyên Đại học KHTN Hà Nội năm 2011 (vòng 1)

18. Đề thi vào lớp 10 chuyên Đại học KHTN Hà Nội năm 2011 (vòng 2)

19. Đề thi vào lớp 10 chuyên Đại học KHTN Hà Nội năm 2010 (vòng 1)

20. Đề thi vào lớp 10 chuyên Đại học KHTN Hà Nội năm 2010 (vòng 2)

21. Đề thi vào lớp 10 chuyên Đại học KHTN Hà Nội năm 2009 (vòng 1)

22. Đề thi vào lớp 10 chuyên Đại học KHTN Hà Nội năm 2009 (vòng 2)

PHẦN 2: HƯỚNG DẪN GIẢI

(4)

Bài

a, Giải phương trình:

2

26

2 26 30

30

x

x x

x

+

+ + = +

+

b, Giải hệphương trình: 2

2

2

( )(2 ) 27

x y

x y y xy

 + =

 

+ + + =



Bài

a, Tìm tất cặp số nguyên thỏa mãn: 2

(x − +x 1)(y +xy)=3x−1

b, Với x,y số thực thay đổi thỏa mãn 1≤ y ≤ xy + ≥ 2y, tìm giá trị nhỏ biểu thức 22

4

x M

y

+ =

+

Bài Cho hình vng ABCD, đường trịn (O) nội tiếp hìnhvng tiếp xúc với cạnh AB, AD hai điểm E,F Gọi G giao điểm đường thẳng CE BF

a, Chứng minh năm điểm A,F,O,G,E nằm đường tròn

b, Gọi giao điểm đường thẳng FB đường tròn M(M≠F) CMR M trung điểm đoạn thẳng BG

c, CMR trực tâm tam giác GAF nằm đường tròn (O)

Bài Cho x, y, z số thực dương thỏa mãn xy + yz + xz = Chứng minh rằng:

3

2 2 2 2 2

1 1

3

1 1 1 1 1

 

 

+ + ≥ + +

 

+ + +  + + + 

x y z

x y z x y z

SỞ GIÁO DỤC VÀ ĐÀO TẠO HÀ NỘI

ĐỀ CHÍNH THỨC

Đề thi gồm 01trang Đề số 1

KỲ THI VÀO LỚP 10 CHUYÊN KHOA HỌC TỰ NHIÊN

NĂM HỌC 2019 – 2020

Mơn: TỐN (VỊNG 1)

Thời gian làm bài: 150 phút

(5)

Bài

a Giải hệphương trình:

( )( )

2

2

3

x y xy

x y x xy

 + + =

 + + + =



b Giải phương trình:

( )

2

27 27

2 2

x x x

x

x x

+ + = +

+ −

+ − +

Bài

a Chứng minh với sốnguyên dương n, ta có

( )7 ( )7 ( )7

27n 10 10n 27 5n 10 27

 + +  + + +  + + + 

     

chia hết cho 42

b Với x y, số thực dương thay đổi thỏa mãn điều kiện

2

4x +4y +17xy+5x+5y≥1

Tìm giá trị nhỏ biểu thức: 2

17 17 16

P= x + y + xy

Bài Cho tam giác ABC cân A, có đường trịn nội tiếp ( )I Các điểm E F, theo thứ

tự thuộc cạnh CA AB, (E khác C A; F khác B A) cho EF tiếp xúc với

đường tròn ( )I điểm P Gọi K L, hình chiếu vng góc E F, BC

Giả sử FK cắt EL điểm J Gọi H hình chiếu vng góc J BC

a) Chứng minh HJ phân giác góc EHF

b) Kí hiệu S S1, diện tích tứ giác BFJL CEJK Chứng minh

1

2

S BF

S =CE

c) Gọi D trung điểm cạnh BC Chứng minh ba điểm P J D, , thẳng hàng

Bài Cho M tập tất 4039 số nguyên liên tiếp từ −2019 đến 2019 Chứng minh

trong 2021 sốđôi phân biệt chọn từM ln tồn ba số phân biệt có tổng

bằng

SỞ GIÁO DỤC VÀ ĐÀO TẠO HÀ NỘI

ĐỀ CHÍNH THỨC

Đề thi gồm 01trang Đề số 2

KỲ THI VÀO LỚP 10 CHUYÊN KHOA HỌC TỰ NHIÊN

NĂM HỌC 2019 – 2020

Mơn: TỐN (VỊNG 2)

Thời gian làm bài: 150 phút

(6)

Bài

a) Giải phương trình x x x x 12− + 3+ = + .

b) Giải hệ phương trình xy y2 22 y

x 2y 2xy x

 + = +

 

+ + = +



Bài

a) Tìm tất cặp số nguyên ( )x; y thỏa mãn (x y 3x 2y+ )( + )2 =2x y 1+ − b) Với a, b số thực dương thay đổi thỏa mãn a 2b b

3

+ = + Tìm giá trị nhỏ biểu thức M a b

a 2b b 2a

= +

+ +

Bài Cho tam giác ABC có đường trịn nội tiếp ( )I tiếp xúc với cạnh BC, CA, AB điểm D, E, F Gọi K hình chiếu vng góc B đường thẳng DE M trung điểm đoạn thẳng DF

a) Chứng minh hai tam giác BKM DEF đồng dạng với

b) Gọi L hình chiếu vng góc C đường thẳng DF N trung điểm đoạn thẳng DE Chứng minh hai đường thẳng MK NL song song với c) Gọi J, X trung điểm đoạn thẳng KL ID Chứng minh đường thẳng JX vng góc với đường thẳng EF

Bài Trên mặt phẳng cho hai điểm P Q phân biệt Xét 10 đường thẳng nằm mặt phẳng thỏa mãn tính chất sau:

i) Khơng có hai đường thẳng song song trùng

ii) Mỗi đường thẳng qua P Q, khơng có đường thẳng qua P Q Hỏi 10 đường thẳng chia mặt phẳng thành tối đa miền? Hãy giải thích

SỞ GIÁO DỤC VÀ ĐÀO TẠO HÀ NỘI

ĐỀ CHÍNH THỨC

Đề thi gồm 01trang Đề số 3

KỲ THI VÀO LỚP 10 CHUYÊN KHOA HỌC TỰ NHIÊN

NĂM HỌC 2018 – 2019

Mơn: TỐN (VỊNG 1)

Thời gian làm bài: 150 phút

(7)

Câu

a) Giải hệphương trình :

( )( )

3 3

( )

7 1 31

xy x y

x y x y x y

+ =



 + + + + + =



b) Giải phương trình: 3+ x(3 2− x)=7 x+5 2− x

Câu

a) Cho x y, số nguyên cho x2−2xyy xy2; −2y2−x chia hết cho

Chứng minh 2

2x +y +2x+ycũng chia hết cho

b) Cho a a1, 2, ,a50là số nguyên thỏa mãn: 1≤ ≤a1 a2 ≤a50 ≤50, 50 100

a + +a +a = Chứng minh từ sốđã cho chọn vài

sốcó tổng 50

Câu Cho ngũ giác lồi ABCDEnội tiếp (O) có CD/ /BE Hai đường chéo CE BD cắt

nhau P Điểm M thuộc BE cho MAB =PAE Điểm K thuộc AC cho MK song song AD, điểm L thuộc đường thẳng AD cho ML // AC Đường tròn ngoại tiếp tam giác KBC cắt BD, CE Q S (Q khác B, S khác C)

a) Chứng minh điểm K, M, Q thẳng hàng

b) Đường tròn ngoại tiếp tam giác LDE cắt BD, CE tai T R (T khác D, R khác E) Chứng minh M, S, Q, R,T thuộc đường tròn

c) Chứng minh đường tròn ngoại tiếp tam giác PQR tiếp xúc (O)

Câu Cho a b c, , số thực dương Chứng minh

1

2

ab bc

a b b c a b b c

  

+ + ≤

  

 + +  + + 

 

SỞ GIÁO DỤC VÀ ĐÀO TẠO HÀ NỘI

ĐỀ CHÍNH THỨC

Đề thi gồm 01trang Đề số 4

KỲ THI VÀO LỚP 10 CHUYÊN KHOA HỌC TỰ NHIÊN

NĂM HỌC 2018 – 2019

Mơn: TỐN (VỊNG 2)

Thời gian làm bài: 150 phút

(8)

Câu (3.5 điểm)

a) Giải hệ phương trình

2

2

1

x y xy

x x y y

    

   

b) Giải phương trình 2x 1 x  1  x  1 1x2 1x2

Câu 2(2.5 điểm)

a) Chứng minh không tồn số nguyên x, y thỏa mãn đẳng thức

2

12x 26xy 15y  4617

b) Với a, b số thực dương Tìm giá trị lớn nhát biểu thức

  2

1 1

M a b

ab

a b b a

 

 

    

 

 

Câu (3.0 điểm)

Cho hình thoi ABCD có BAD 900 Đường tròn tâm I nội tiếp tam giác ABD tiếp xúc với BD BA J L Trên đường thẳng IJ lấy điểm K cho BK song song ID a) Chứng minh CBK ABI

b) Chứng minh KCKB

c) Chứng minh bốn điểm C, K, I ,L nằm đường tròn

Câu (1.0 điểm)

Tìm tập hợp số nguyên dương n cho tồn cách xếp số 1;2; 3; ;n

thành a a a1; ; ; ;2 3 an mà chia số a a a a a a1; 1 2; 1 3; ;a a1 2 an cho n ta số dư đôi khác

SỞ GIÁO DỤC VÀ ĐÀO TẠO HÀ NỘI

ĐỀ CHÍNH THỨC

Đề thi gồm 01trang Đề số 5

KỲ THI VÀO LỚP 10 CHUYÊN KHOA HỌC TỰ NHIÊN

NĂM HỌC 2017 – 2018

Mơn: TỐN (VỊNG 1)

Thời gian làm bài: 150 phút

(9)

Câu (3.5 điểm).

1 Giải hệ phương trình 2 2

3

x y x y

x y xy

   



   



2 Với a, b số thực dương thỏa mãn ab  a b Chứng minh rằng:

  

2 2 2

1

1 2 1 1

a b ab

a b a b

 

   

Câu (2.5 điểm).

Giả sử p q số nguyên tố thỏa mãn đẳng thức p p  1 q q 21. a) Chứng minh tồn số nguyên dương k cho p 1 kq q, 2 1 kp. b) Tìm tất số nguyên tố p, q thỏa mãn đẳng thức p p  1 q q 1. Với a, b, c số thực dương thỏa mãn abbccaabc2 Tìm giá trị lớn biểu thức 2 2 2

2 2 2

a b c

M

a a b b c c

  

  

     

Câu (3.0 điểm) Cho tam giác ABC nhọn với ABAC Gọi E, F trung điểm CA, AB Đường trung trực EF cắt BC D Giả sử P nằm góc EAF nằm

ngoài tam giác AEF cho PEC DEF PEB DFE Đường thẳng PA cắt đường tròn ngoại tiếp tam giác PEF Q khác P

a) Chứng minh EQF BAC EDF

b) Tiếp tuyến P đường tròn ngoại tiếp tam giác PEF cắt CA, AB M, N Chứng minh bốn điểm C, M, B, N nằ đường tròn Gọi đường tròn đường tròn  K

c) Chứng minh đường tròn  K tiếp xúc với đường tròn ngại tiếp tam giác AEF

Câu (1.0 điểm) Cho n số nguyên dương với n 5 Xét đa giác lồi n cạnh Người ta muốn kẻ số đường chéo đa giác mà đường chéo chia đa giác thành k miền, mõi miền mọt ngũ giác lồi (hai miền khơng có điểm chung trong)

a) Chứng minh ta thực với n 2018,k 672

b) Với n 2017,k 672 ta thực khơng? Hãy giải thích

SỞ GIÁO DỤC VÀ ĐÀO TẠO HÀ NỘI

ĐỀ CHÍNH THỨC

Đề thi gồm 01trang Đề số 6

KỲ THI VÀO LỚP 10 CHUYÊN KHOA HỌC TỰ NHIÊN

NĂM HỌC 2017 – 2018

Môn: TỐN (VỊNG 2)

Thời gian làm bài: 150 phút

(10)

Câu (3.5 điểm).

a) Giải hệ phương trình  

  

3

2

x + y + xy x + y = xy + x + y = 

 

b) Giải phương trình

x

x    x

Câu (2.5 điểm)

a)Tìm tất giá tri m cho tồn cặp số nguyên  x y; thỏa mãn hệ phương

trình :

 

2 2

2

2

mxy m

m x y m

   

    

b) Với x, y số thực thỏa mãn điều kiện 0  x y 2;2x  y 2xy

Tìm giá trị lớn biểu thức P x x2  1 y y2 1

Câu (3.0 điểm) Cho tam giác ABC nhọn nội tiếp đường tròn  O với ABAC Phân giác góc BAC cắt BC D cắt đường tròn  O E khác A M trung điểm

đoạn thẳng AD Đường thẳng BM cắt đường tròn  O P khác B Giả sử đường

thẳng EP AC cắt N

a) chứng minh tứ giác APNM nội tiếp N trung điểm đoạn thẳng AC

b) Giả sử đường tròn  K ngoại tiếp tam giác EMN cắt đường thẳng AC Q khác N

Chứng minh B Q đối xứng qua AE

c) Giả sử đường tròn  K cắt đường thẳng BM M Chứng minh RA vng góc RC

Câu (1.0 điểm).

Số nguyên a gọi số “đẹp” với cách xếp theo thứ tự tùy ý 100 số 1, 2, 3,…, 100 ln tồn 10 số hạng liên tiếp có tổng lớn a Tìm số “đẹp” lớn

SỞ GIÁO DỤC VÀ ĐÀO TẠO HÀ NỘI

ĐỀ CHÍNH THỨC

Đề thi gồm 01trang Đề số 7

KỲ THI VÀO LỚP 10 CHUYÊN KHOA HỌC TỰ NHIÊN

NĂM HỌC 2016 – 2017

Mơn: TỐN (VỊNG 1)

Thời gian làm bài: 150 phút

(11)

Câu 1(3.5 điểm).

a) Giải hệ phương trình

2

2

4

4 10

x y

x xy x y

   

    



b) giải phương trình

64

5

5 6

x x

x x

x x

   

 

Câu 2(2.5 điểm).

a) Với x, y số nguyên thỏa mãn đẳng thức 2

2

x   y  Chứng minh

2 40

xy

b) Tìm tất cặp số nguyên  x y; thỏa mãn đẳng thức sau x4 2x2 y3.

Câu 3(3.0 điểm)

Cho hình vng ABCD nội tiếp đường trịn  O P điểm thuộc cung nhỏ AD

đường tròn  O P khác A, D Các đường thẳng PB, PC cắt AD AD M, N

Đường trung trực AM cắt đường thẳng AC, PB E, K Đường trung trực DN cắt đường thẳng BD, PC F, L

a) Chứng minh ba điểm K, O, L thẳng hàng

b) Chứng minh đường thẳng PO qua trung điểm EF

c) Giả sử đường thảng EK cắt đường thẳng FL AC cắt T Đường thẳng ST cắt đường thẳng PB, PC U V Chứng minh bốn điểm K, L, V, U thuộc đương tròn

Câu 4(1.0 điểm)

Chứng minh với số tự nhiên n 3luôn tồn cách xếp n số 1,2, 3, ,n

thành số x x x1, , , ,2 3 xn cho

2

i k j

x x

x   với số i j k; ;  mà 1   i j k n

SỞ GIÁO DỤC VÀ ĐÀO TẠO HÀ NỘI

ĐỀ CHÍNH THỨC

Đề thi gồm 01trang Đề số 8

KỲ THI VÀO LỚP 10 CHUYÊN KHOA HỌC TỰ NHIÊN

NĂM HỌC 2016 – 2017

Môn: TỐN (VỊNG 2)

Thời gian làm bài: 150 phút

(12)

Câu 1.(3,0 điểm)

1) Giả sử hai số thực phân biệt thỏa mãn a) Chứng minh

b) Chứng minh 2) Giải hệphương trình

Câu 2. (3,0 điểm)

1) Tìm số ngun khơng nhỏhơn cho chia hết cho 2) Với số thực thỏa mãn đẳng thức Tìm giá trị lớn nhỏ biểu thức

Câu 3.(3,0 điểm) Cho tam giác nhọn khơng cân có tâm đường tròn nội tiếp điểm Đường thẳng cắt Gọi điểm đối xứng qua

1) Chứng minh song song với

2) Gọi trung điểm đoạn thẳng Đường tròn ngoại tiếp tam giác cắt đường tròn ngoại tiếp tam giác khác Chứng minh bốn điểm nằm đường tròn

3) Chứng minh ba điểm thẳng hàng

Câu 4. (1,0 điểm)

1) Cho bảng ô vuông 2015 x2015 Kí hiệu ô ô hang thứ , cột thứ Ta viết sốnguyên dương từ1 đến 2015 vào ô bảng theo quy tắc sau:

i) Số1 viết vào ô (1,1)

ii) Nếu số viết vào ô ( ) số viết vào

iii) Nếu số viết vào số viết vào (xem hình 1) Khi số2015 viết vào ô

Hãy xác định

2) Giả sử số thực dương thỏa mãn

Chứng minh

;

a b a23a b 23b2

3 a b  

3 45

a   b

2 2

2

4

x y xy x y xy    

  



 x y; xy1 x1y1

;

x y x y2 22y 1 0.

3

xy P

y

ABC

I AI BC D E F; D

;

IC IB

EF BC

; ;

M N J DE DF EF; ;

AEM

AFN P A

; ; ;

M N P J

; ;

A J P

( ; )i j

i j

k ( ; )i j i1 k1

( 1;ij1)

k (1; )j k1 (j1; 1)

m n; 

m n

; ;

a b c ab bc ac abc   4

 

2 2 2

a      b c a b c ab bc ac 

SỞ GIÁO DỤC VÀ ĐÀO TẠO HÀ NỘI

ĐỀ CHÍNH THỨC

Đề thi gồm 01trang Đề số 9

KỲ THI VÀO LỚP 10 CHUYÊN KHOA HỌC TỰ NHIÊN

NĂM HỌC 2015 – 2016

Mơn: TỐN (VỊNG 1)

Thời gian làm bài: 150 phút

(13)

Câu 1.(3,0 điểm)

1) Với số thỏa mãn

Chứng minh

2) Giải hệ phương trình

Câu 2.(3,0 điểm)

1) Tìm số tự nhiên để số phương (số phương bình phương số nguyên)

2) Tìm nguyên thỏa mãn đẳng thức

3) Giả sử số thực lớn Tìm giá trị nhỏ biểu thức:

Câu 3. (3,0 điểm) Cho tam giác nhọn không cân với Gọi trung điểm đoạn thẳng Gọi hình chiếu vng góc đoạn Trên tia đối tia lấy điểm cho

1) Chứng minh

2) Gọi điểm đối xứng với qua Đường thẳng cắt .Chunwgs minh bốn điểm thuộc đường tròn,gọi đường tròn

3) Đường tròn ngoại tiếp tam giác cắt Chứng minh song song với

Câu 4.(1,0 điểm) Ký hiệu tập hợp gồm diểm phân biệt mặt phẳng Giả sử tất điểm không nằm đường thẳng Chứng minh có đường thẳng phân biệt mà đường thẳng qua hai điểm

; ;

a b c

 3   3  3 3 3a3b3c 24 3a b c   3b c a   3c a b 

a2b b 2c c 2a1

  3

2

27 26 27

x y xy

x y y x x x

   



      



n n5 n30

x y;  1 x y  3 xy

; ;

x y z

4 4

y

x z

P

y z z x x y

  

     

ABC

AB ACM

BC H B AM

AM N AN2MH

BN AC

Q A N AC BQ D

; ; ;

B D N C ( )O

AQD

 ( )O G D NG

BC

S 2015

S

2015 S

SỞ GIÁO DỤC VÀ ĐÀO TẠO HÀ NỘI

ĐỀ CHÍNH THỨC

Đề thi gồm 01trang Đề số 10

KỲ THI VÀO LỚP 10 CHUYÊN KHOA HỌC TỰ NHIÊN

NĂM HỌC 2015 – 2016

Mơn: TỐN (VÒNG 2)

Thời gian làm bài: 150 phút

(14)

Câu 1. 1) Giải phương trình ( )( 2) 1+ +x 1−x 2+2 1−x =8

2) Giải hệphương trình

2

2

1

2

 − + =

 

+ + =



x xy y

x xy y

Câu 1) Giả sử x, y, z ba sốdương thỏa mãn điều kiện x + y + z = xyz Chứng minh rằng: 2 2 2 ( ()(5 )( ) )

1 1

+ +

+ + =

+ + +

+ + +

xyz x y z

x y z

x y y z z x

x y z

2) Tìm nghiệm nguyên phương trình: 2( )

3

+ + + + +

x y x y x y xy

Câu Cho tam giác ABC nhọn với AB < BC Gọi D điểm thuộc cạnh BC cho AD phân giác ∠BAC Đường thẳng qua C song song với AD cắt trung trực AB F

1) Chứng minh tam giác ABF đồng dạng với tam giác ACE

2) Chứng minh đường thẳng BE, CF, AD đồng quy điểm, gọi điểm G 3) Đường thẳng qua G song song với AE cắt đường thẳng BF Q Đường thẳng QE cắt đường tròn ngoại tiếp tam giác GEC P khác E Chứng minh điểm A, P, G, Q, F thuộc đường tròn

Câu Giả sử a, b, c số thực dương thỏa mãn đẳng thức ab + bc + ca = Chứng minh ( ) 4

2

9

+ + ≤ + + +

abc a b c a b b c c a

SỞ GIÁO DỤC VÀ ĐÀO TẠO HÀ NỘI

ĐỀ CHÍNH THỨC

Đề thi gồm 01trang Đề số 11

KỲ THI VÀO LỚP 10 CHUYÊN KHOA HỌC TỰ NHIÊN

NĂM HỌC 2014 – 2015

Môn: TỐN (VỊNG 2)

Thời gian làm bài: 150 phút

(15)

Câu 1.

1 Giả sử x, y số thực dương thỏa mãn: y 22y2 2 44y4 4 88y4 4

x+y+x +y +x +y +xy =

Chứng minh rằng: 5y = 4x Giải hệ phương trình: 2

2

2 12

6 12

x y xy

x x y y y x

 − + =

 

+ = + +



Câu 2.

1 Cho x, y số nguyên lớn cho 4x2y2 – 7x + 7y số phương

Chứng minh rằng: x = y

2 Giả sử x, y số thực không âm thỏa mãn: x3+ y3+ xy = x2+ y2 Tìm giá trị

lớn nhỏ biểu thức:

2

x x

P

y y

+ +

= +

+ +

Câu 3.Cho tam giác ABC nội tiếp đường tròn (O) điểm P nằm tam giác thỏa mãn PB = PC D điểm thuộc cạnh BC (D khác B D khác C) cho P nằm đường tròn ngoại tiếp tam giác DAC đường tròn ngoại tiếp tam giác DAC Đường thẳng PB cắt đườngtròn ngoại tiếp tam giác DAB E khác B Đường thẳng PC cắt đường tròn ngoại tiếp tam giác DAC F khác C

1 Chứng minh bốn điểm A, E, B, F thuộc đường tròn

2 Giả sử đường thẳng AD cắt đường tròn (O) Q khác A, đường thẳng AF cắt đường thẳng QC L Chứng minh tam giác ABE đồng dạng với tam giác CLF

3 Gọi K giao điểm đường thẳng AE đường thẳng QB Chứng minh rằng:    

QKL+PAB=QLK+PAC

Câu 4.Cho tập hợp A gồm 31 phần tử dãy gồm m tập hợp A thỏa mãn đồng thời điều kiện sau:

i) Mỗi tập hợp thuộc dãy có hai phần tử

ii) Nếu hai tập hợp thuộc dãy có chung hai phần tử số phần tử hai tập hợp khác

Chứng minh rằng: m ≤ 900

SỞ GIÁO DỤC VÀ ĐÀO TẠO HÀ NỘI

ĐỀ CHÍNH THỨC

Đề thi gồm 01trang Đề số 12

KỲ THI VÀO LỚP 10 CHUYÊN KHOA HỌC TỰ NHIÊN

NĂM HỌC 2014 – 2015

Mơn: TỐN (VỊNG 2)

Thời gian làm bài: 150 phút

(16)

Câu 1.(3,0 điểm) 1) Giải phương trình

2) Giải hệ phương trình

Câu 2. (3,0 điểm)

1) Cho số thực thỏa mãn Chứng minh

2) Có số nguyên dương có chữ số cho chia hết cho ?

Câu 3.(3,0 điểm)

Cho tam giác nhọn nội tiếp đường trịn với Đường phân giác góc cắt điểm khác Gọi trung điểm điểm đối xứng với qua tâm Giả sử đường tròn ngoại tiếp tam giác cắt đoạn thẳng điểm khác

1) Chứng minh tam giác tam giác đồng dạng 2) Chứng minh vng góc với

Câu 4. (1,0 điểm)

Giả sử số thực dương thỏa mãn điều kiện Tìm giá trị nhỏ biểu thức

3x 1 2 x

1

1 1

4

x y

x y x xy

y xy     



  

 

     

  

   

; ;

a b c a b b c c a      8abc

        

3

a b c ab bc ca a b b c c a       a b b c   b c c a   c a a b 

5 abcde abc10d e 

101

ABC

 ( )O AB AC

BAC ( )O D A M AD E

D OABM AC

F A

BDM

 BCF

EF AC

; ; ;

a b c d abc bcd cda dab   1

 3 3

4

Pabcd

SỞ GIÁO DỤC VÀ ĐÀO TẠO HÀ NỘI

ĐỀ CHÍNH THỨC

Đề thi gồm 01trang Đề số 13

KỲ THI VÀO LỚP 10 CHUYÊN KHOA HỌC TỰ NHIÊN

NĂM HỌC 2013 – 2014

Mơn: TỐN (VỊNG 1)

Thời gian làm bài: 150 phút

(17)

Câu 1.(3,0 điểm)

1) Giải hệ phương trình

2) Giải phương trình

Câu 2.(3,0 điểm)

1) Tìm cặp số nguyên thỏa mãn

2) Với số thực dương thỏa mãn tìm giá trị nhỏ biểu thức

Câu (3,0 điểm) Cho tam giác nhọn nội tiếp đường trịn có trực tâm Gọi điểm nằm đường tròn ngoại tiếp tam giác ( khác ) nằm tam giác cắt khác cắt khác cắt

cắt Đường tròn ngoại tiếp tam giác đường tròn ngoại tiếp tam giác cắt khác

1) Chứng minh ba điểm thẳng hàng

2) Giả sử phân giác góc Chứng minh qua trung điểm

Câu 4. (1,0 điểm) Giả sử dãy số thực có thứ tự thỏa mãn điều kiện

Chứng minh

3 1

7

x y y x xy xy y x

      

   



2

3 1

x  xx  x

x y; 

2

5x 8y 20412

x y;  x y 1,

2

1 1

P x y x y

 

 

     

ABC

 ( )O H P

HBC

P B C, H

ABC

PB ( )O M B, PC ( )O N C BM AC E,

CN AB FAME

ANF

Q A

; ;

M N Q

APMAN PQ

BC

1 192

xx   x

1 192

xx   xx1  x2   x192 2013

192 2013 96

xx

SỞ GIÁO DỤC VÀ ĐÀO TẠO HÀ NỘI

ĐỀ CHÍNH THỨC

Đề thi gồm 01trang Đề số 14

KỲ THI VÀO LỚP 10 CHUYÊN KHOA HỌC TỰ NHIÊN

NĂM HỌC 2013 – 2014

Mơn: TỐN (VỊNG 2)

Thời gian làm bài: 150 phút

(18)

Câu 1.(3,0 điểm)

1) Giải phương trình:

2) Giải hệ phương trình

Câu 2. (3,0 điểm)

1) Tìm tất cặp số nguyên thỏa mãn đẳng thức

2) Giả sử số thực dương thỏa mãn điều kiện Tìm giá trị nhỏ biểu thức

Câu 3. (3,0 điểm) Cho tam giác nhọn nội tiếp đường tròn tâm Gọi điểm cung nhỏ ( khác không qua ) Giả sử điểm thuộc đoạn thẳng cho đường tròn đường kính cắt cung nhỏ điểm khác

1) Gọi điểm đối xứng với điểm qua Chứng minh ba điểm thẳng hàng

2) Đường tròn đường kính cắt điểm khác Chứng minh tâm đường tròn nội tiếp tam giác

Câu 4. (1,0 điểm) Giả sử số thực dương thỏa mãn: ; ; Tìm giá trị nhỏ biểu thức:

  

9 2012 2012

x  x   xx

2 2 4

2

x y y x y xy     

   



 x y;

x y 1xy x y   5 2x y 

 x y;  x1 y 1

2 y

x P

y x

 

ABC

O M

BC M B C; AM O P

AM MPBC N M

D M O N P D; ;

MP MD Q M P

AQN

; ;

a b c a b  3 c c b 1

a b c   

   

2

1 1

ab a b c ab a b c

Q    

  

SỞ GIÁO DỤC VÀ ĐÀO TẠO HÀ NỘI

ĐỀ CHÍNH THỨC

Đề thi gồm 01trang Đề số 15

KỲ THI VÀO LỚP 10 CHUYÊN KHOA HỌC TỰ NHIÊN

NĂM HỌC 2012 – 2013

Mơn: TỐN (VỊNG 1)

Thời gian làm bài: 150 phút

(19)

Câu 1.(3,0 điểm)

1) Giải hệ phương trình

2) Giải phương trình

Câu 2.(3,0 điểm)

1) Tìm hai chữ số cuối số

2) Tìm giá trị lớn hàm số , với

Câu 3. (3,0 điểm) Cho tam giác nhọn ( ) nội tiếp đường tròn Giả sử hai điểm thuộc cung nhỏ cho song song với tia nằm hai tia Gọi hình chiếu vng góc điểm hình chiếu vng góc điểm

1) Giả sử cắt điểm Chứng minh nằm đường tròn

2) Gọi giao điểm khác Giả sử cắt Chứng minh bốn điểm thuộc đường tròn

Câu (1,0 điểm) Với số nguyên lớn cố định, xét tập số thực đơi khác Kí hiệu số giá trị khác tổng

( ) Tìm giá trị lớn giá trị nhỏ

 

  3

2

9 26

xy x y

xy x y x y

  



    



x 4 2 4  x 2 2x

106 2012

41 57 A 

2

3

yx xx

2 x

ABC

AB AC ( )O

;

M N BCMN BC AN

;

AM AB P C AN Q

M AB

CP QM T T ( )O

NQ ( )O R N AM PQ S

; ; ;

A R Q S

n n

 1; ; ;2 n

Xx x x C X  i j

xx 1  i j n C X 

SỞ GIÁO DỤC VÀ ĐÀO TẠO HÀ NỘI

ĐỀ CHÍNH THỨC

Đề thi gồm 01trang Đề số 16

KỲ THI VÀO LỚP 10 CHUYÊN KHOA HỌC TỰ NHIÊN

NĂM HỌC 2012 – 2013

Mơn: TỐN (VỊNG 2)

Thời gian làm bài: 150 phút

(20)

Câu 1(3,0 điểm)

1) Giải hệ phương trình

2) Giải phương trình

Câu 2.(3,0 điểm)

1) Chứng minh không tồn ba số nguyên thỏa mãn đẳng thức:

2) Tìm tất cặp số nguyên thỏa mãn đẳng thức

Câu 3. (3,0 điểm) Cho hình bình hành với Đường phân giác góc cắt đường trịn ngoại tiếp tam giác khác Kẻ đường thẳng qua vng góc với Đường thẳng cắt đường thẳng 1) Chứng minh

2) Chứng minh tâm đường tròn ngoại tiếp tam giác 3) Gọi giao điểm , chứng minh

Câu 4. (1,0 điểm) Với số thực dương, tìm giá trị nhỏ biểu thức

 

2

1

( 2)

x y x y

y x y x

  

   

   

 

2

3

2

x x

x x

  

x y z; ; 

4 7 5

xyz

x y; 

x1 4 x 14y3

ABCDBAD90

BCDBCD O C d

A CO d CB CD; E F;

OBE ODC

  

OCEF

OC BD I

IB BE EI ID DF FI

;

x y

 

3

3 3

4

y x

x y y x y

P

  

SỞ GIÁO DỤC VÀ ĐÀO TẠO HÀ NỘI

ĐỀ CHÍNH THỨC

Đề thi gồm 01trang Đề số 17

KỲ THI VÀO LỚP 10 CHUYÊN KHOA HỌC TỰ NHIÊN

NĂM HỌC 2011 – 2012

Mơn: TỐN (VỊNG 1)

Thời gian làm bài: 150 phút

(21)

Câu 1.(3,0 điểm)

1) Giải phương trình : 2) Giải hệ phương trình:

Câu 2.(3,0 điểm)

1) Với số thực ta gọi phần nguyên số nguyên lớn không vượt ký hiệu Chứng minh với số nguyên dương , biểu thức

không biểu diễn dạng lập phương số nguyên dương 2) Với số thực dương thỏa mãn đẳng thức , Tìm giá trị nhỏ

nhất biểu thức :

Câu 3. (3,0 điểm) Cho hình thang với song song Các góc góc nhọn Hai đường chéo cắt điểm đoạn thẳng ( không trùng với ) Giả sử đường tròn ngoại tiếp tam giác cắt đoạn thẳng khác đường tròn ngoại tiếp tam giác cắt đoạn thẳng khác 1) Chứng minh năm điểm nằm đường tròn Gọi đường tròn

2) Giả sử đường thẳng cắt , chứng minh nằm đường tròn

3) Trong trường hợp thẳng hàng, chứng minh

Câu 4. (1,0 điểm) Giả sử tập tập số tự nhiên Tập có phần tử nhỏ 1, phần tử lớn 100 thuộc ( ) tồn thuộc cho ( ) Hãy tìm tập có số phần tử nhỏ

x 3 x 1 x 11

  

2 2

2

1

2

4 x y xy x y x y

x y 

  



 

a a a

a  

  n

2

3 1 27

n n  

 

 

; ;

x y z xy yz zx  5

     

3

6 6

x y z P

x y z

 

    

ABCD BC ADBAD CDA

AC BD I P

BC P B C; BIP

PA M PCIP PD N P

; ; ; ; A M I N D

( )K

BM CN Q Q

( )K

; ;

P I Q PB BD

PC CA

AA

x A x1 a b;

A x a b  a b A

SỞ GIÁO DỤC VÀ ĐÀO TẠO HÀ NỘI

ĐỀ CHÍNH THỨC

Đề thi gồm 01trang Đề số 18

KỲ THI VÀO LỚP 10 CHUYÊN KHOA HỌC TỰ NHIÊN

NĂM HỌC 2011 – 2012

Mơn: TỐN (VỊNG 2)

Thời gian làm bài: 150 phút

(22)

Câu I

1) Giải hệ phương trình

   

= +

= +

+

23 12

8

2

2

y x

xy y

x

2) Giải phương trình

2x+ + x2 − x+ = + x3 +

Câu II

1) Tìm tất số nguyên không âm (x, y) thoả mãn đẳng thức

(1+ 2)(1+ 2)+4 +2( + )(1+ )=25

xy y

x xy y

x

2) Với số thực a, ta gọi phần nguyên số a số nguyên lớn không vượt a ký hiệu [a] Chứng minh với n ngun dương ta ln có

( ) n

n n

n n

=    

 

+ + + +

+

1

3

7

3

Câu III

Cho đường tròn (O) với đường kính AB = 2R Trên đường thẳng tiếp xúc với đương tròn (O) A ta lấy điểm C cho góc

30 =

ACB Gọi H giao điểm thứ hai đường

thăng BC với đường trịn (O)

1) Tính độ dài đương thẳng AC, BC khoảng cách từ A đến đương thẳng BC theo R

2) Với điểm M đoạn thẳng AC, đường thẳng BM cắt đường tròn (O điểm N (khác B) Chứng minh bốn điểm C, M, N, H nằm đường trịn tâm đường trịn ln chạy đường thẳng cố định M thayđổi đoạn thẳng AC

Câu IV

Với a,b số thực thoả mãn đẳng thức

4 ) )(

( +a +b = , tìm giá trị nhỏ

của biểu thức 4

1

1 a b

P= + + +

SỞ GIÁO DỤC VÀ ĐÀO TẠO HÀ NỘI

ĐỀ CHÍNH THỨC

Đề thi gồm 01trang Đề số 19

KỲ THI VÀO LỚP 10 CHUYÊN KHOA HỌC TỰ NHIÊN

NĂM HỌC 2010 – 2011

Mơn: TỐN (VÒNG 1)

Thời gian làm bài: 150 phút

(23)

Câu I

1) Giải phương trình x 3+ + 3x 1+ = 2) Giải hệ phương trình ( )( )

2

5x 2y 2xy 26

3x 2x y x y 11

 + + =

 + + − =



Câu II

1) Tìm tất số nguyên dương n để n2 +391

số phương

2) Giả sử x, y, z số thực dương thỏa mãn điều kiện x + y + z = Chứng minh rằng: xy z 2x2 2y2

1 xy

+ + +

≥ +

Câu III

Cho tam giác ABC có ba góc nhọn M làđiểm nằm tam giác Kí hiệu H hình chiếu M cạnh BC P, Q, E, F hình chiếu H đường thẳng MB, MC, AB, AC Giả sử bốn điểm P, Q, E, F thẳng hàng

1) Chứng minh M trực tâm tam giác ABC 2) Chứng minh BEFC tứ giác nội tiếp

Câu IV

Trong dãy số gồm 2010 số thực khác xếp theo thứ tự a , a , a , , a1 2 3 2010ta đánh dấu tất số dương tất số mà tổng với số liên tiếp liền sau số dương

Chứng minh dãy số cho có số dương tổng tất số đánh dấu số dương

SỞ GIÁO DỤC VÀ ĐÀO TẠO HÀ NỘI

ĐỀ CHÍNH THỨC

Đề thi gồm 01trang Đề số 20

KỲ THI VÀO LỚP 10 CHUYÊN KHOA HỌC TỰ NHIÊN

NĂM HỌC 2010 – 2011

Mơn: TỐN (VÒNG 2)

Thời gian làm bài: 150 phút

(24)

Câu I 1) Giải phương trình

x2 −x+2=2 x2 −x+1 2) Giải hệ phương trình

   

+ = +

= + −

3

1 2

y y x

xy y x

Câu II 1) Tìm chữ số tận số 13 2009 2009

13 + +

2) Với a, b chữ số thực dương, tìm giá trị nhỏ biểu thức

) ( )

( a b b b a

a

b a P

+ +

+ + =

Câu III Cho hình thoi ABCD Gọi H giao điểm hai đường chéo AC BD Biết bánkính đường trịn ngoại tiếp tam giác ABC a bán kính đường trịn ngoại tiếp tam giác ABD b

1) Chứng minh

b a BH AH =

2) Tính diện tích hình thoi ABCD theo bán kính a, b

Câu IV Với a, b, c số thực dương, chứng minh

5 14

8 14

8 14

8

3 2

2

2 2

2

c b a ca a

c c bc

c b

b ab

b a

a ≥ + +

+ + +

+ + +

+ +

SỞ GIÁO DỤC VÀ ĐÀO TẠO HÀ NỘI

ĐỀ CHÍNH THỨC

Đề thi gồm 01trang Đề số 21

KỲ THI VÀO LỚP 10 CHUYÊN KHOA HỌC TỰ NHIÊN

NĂM HỌC 2009 – 2010

Môn: TỐN (VỊNG 1)

Thời gian làm bài: 150 phút

(25)

Câu I 1) Giải phương trình 14 x+35+6 x+1=84+ x2 +36x+35

2) Chứng minh

1 ) (

1

3

4

2

4

4 + − = +

− +

+ + +

+ n

n n

n

Với n nguyên dương

Câu II 1) Tìm số nguyên dương n cho tất số

n + 1, n + 5, n + 7, n + 13, n + 17, n + 25, n + 37 nguyên tố 2) Mỗi lần cho phép thay cặp số (a,b) thuộc tập hợp

{(16,2),(4,32),(6,62),(78,8)}

=

M cặp số (a + c, b + d) cặp số (c, d) thuộc M

Hỏi sau số hữu hạn lần thay ta nhận tập hợp cặp số M1 ={(2018,702),(844,2104),(1056,2176),(2240,912)} hay khơng?

Câu III Cho đường trịn (O) (O’) cắt hai điểm A B Trên đường thẳng AB ta lấy điểm M cho điểm A nằm đoạn BM (MA).Từ điểm M kẻ tới đường tròn (O’) tiếp tuyến MC MD (C D cáctiếp điểm, C nằm (O)) Đường thẳng AC cắt lần thứ hai đường tròn (O) điểm P đường thẳng AD cắt lần thứ hai đường tròn (O)tại Q Đường thẳng CD cắt PQ K

1) Chứng minh hai tam giác BCD BPQ đồng dạng

2) Chứng minh M thay đổi đường trịn ngoại tiếp tam giác KCP ln qua điểm cố định

Câu IV Giả sử x,y,z số thực thoả mãn điều kiện

2 , ,

0≤x y z≤ x+ y + z =

Tìm giá trị nhỏ lớn biểu thức :

(1 )(1 )(1 ) 12

4 4

z y x

z y x

M = + + + − − −

SỞ GIÁO DỤC VÀ ĐÀO TẠO HÀ NỘI

ĐỀ CHÍNH THỨC

Đề thi gồm 01trang Đề số 22

KỲ THI VÀO LỚP 10 CHUYÊN KHOA HỌC TỰ NHIÊN

NĂM HỌC 2009 – 2010

Mơn: TỐN (VỊNG 2)

Thời gian làm bài: 150 phút

(26)

HƯỚNG DN GII

Đề s

Bài

a, Điều kiện

26 ≥ −

x đặt a = 26x+5 b = x2+30 (a ≥ 0, b > 0)

Phương trình trở thành

( )( )

2

2

2 3 26 30

+ = ⇔ − + = ⇔ = ⇔ + = +

a

a b a b a b a b x x

b

Vậy x = 25 nghiệm phương trình b, thay = x2 + y2vào phương trình thứ2 ta

(x + 2y)(x2 + y2 + 3y2 + 4xy) = 27

( )3

2

x+ y =27

⇔ x = - 2y thay vào PT thứ ta (3 - 2y)2 + y2 =

y = y =

5

x =

5

Bài

a, từ biểu thức 2

(x − +x 1)(y +xy)=3x−1 ta nhận thấy 3x - phải chia hết cho (x2 – x + 1)

ta có (3x - 1)(3x - 2) = 9x2 - 9x + = 9(x2 – x + 1) - phải chia hết cho (x2 – x + 1)

suy chia hết cho (x2 – x + 1)

(x2 – x + 1) =

y = 0, 1, -2 thay vào ta có y => (x,y) = (1, 1),(1, -2) (-2, 1) b, Từ giả thiết xy + ≥ 2y => 4xy + ≥ 8y

(27)

⇒ 4(x2 + 4) ≥ 8y + - y2

⇒ 4(x2 + 4) ≥ 4(y2 + 1) + (5y + 2)(2 - y) ≥ 4(y2 + 1)

2

4

+

⇒ =

+

x M

y

Dấu “=” xẩy x = y = 2, Mmin =

Bài

a, Do đường tron (O) nội tiếp hình vng ABCD nên E F trung điểm cạnh AB AD => ∆ ABF ∆ BCE => góc EBG góc BCG => góc BGC vơng => AEGF nằm đường tròn, mà AEOF nằm đường tròn => AEGOF nằm đường trịn

b, Ta có AB tiếp tuyến đường trịn (O) nên góc BEM =góc EFM,

lại có góc EAG EFG chắn cung EG nên góc EAG = EFG

suy EM//AG E trung điểm AB => M trung điểm BG

Bài

3

2 2 2 2 2

1 1

3

1 1 1 1 1

 

 

+ + ≥ + +

 

+ + +  + + + 

x y z

x y z x y z

Ta có:

1 +x = xy + yz + xz + x2 = (x + y)(x + z)

1 + y2 = xy + yz + xz + y2 = (x + y)(y + z)

1 + z2 = xy + yz + xz + z2 =(z + y)(x + z)

VT= 2( )

( )( )( )

x y z

x y y z z x

+ +

+ + +

Ta có:

2

2 2

1 1

 

 + + 

 + + + 

 

x y z

x y z

≤( ) 2 2 2

1 1

 

+ +  + + 

+ + +

 

x y z

x y z

(28)

=( )

( )( ) ( )( ) ( )( )

 

+ +  + + 

+ + + + + +

 

x y z

x y z

x y x z x y y z z y x z

= 2( )

( )( )( )

x y z

x y y z z x

+ +

+ + +

Do

VP= 4( )

3( )( )( )

x y z

x y y z z x

+ +

+ + + 2

1 1

 

 + + 

 + + + 

 

x y z

x y z

Bất đẳng thức trở thành

2 2

1 1

x y z

x y z

+ +

+ + + ≤

3

Ta có:

2

1 ( )( )

 

= ≤  + 

+ +

+ +  

+

x x x x

x y x z

x y x z

x

2

1

( )( )

1

 

= ≤  + 

+ +

+ +  

+

y y y y

x y y z

x y y z

y

2

1 ( )( )

 

= ≤  + 

+ +

+ +  

+

z z z z

x z y z

x z y z

z =>

2 2

1 1

x y z

x y z

+ +

+ + + ≤

3

Dấu “=” xảy x = y = z =

3

Đề s

Câu

a Ta có

( )( )

( )( )

( )( )

2

2

3

3

8

x y x y

x y xy

x y x xy

x y x xy

 + + =  + + =

 ⇔

  + + + =

+ + + = 

 

Do phương trình thứ nên x+ ≠y ta kết hợp hai phương trình

(29)

( )( )

2

2

2

x

x xy x y x x y

x y

= 

+ + = + ⇔ − + − = ⇔ 

= −

TH1:

1

5

y

x y y

y

= 

= ⇒ + + = ⇔ 

= − 

TH2: x= −2 y thay vào phương trình thứ ta có −4(y− = ⇔ =1) y

Vậy hệđã cho có hai cặp nghiệm ( )x y; ( ) (1;1 ; 1; 5− )

b ĐK:

;

2

xx + ≤x

Đặt ( )

5

a= − x +x b= ,− x a b( ≥0) Ta có

2

32 32

2

a b

a b

− −

=

+ + (1)

Ta thấy, a> ≥b 32−a2 < 32−b2 1

2

a+ < +b tức VT<VP, mâu thuẫn Tương tự với a<b mẫu thuẫn Do a=b, tức phương trình ban đầu tương đương với

( ) TM(( ))

5

0 TM

x

x x x

x

= 

− + = − ⇔ 

=

Vậy phương trình có hai nghiệm x=1, 0x=

Câu

a Trước hết ta chứng minh ( )

mod 42 ,

xx ∀ ∈x

(1)

Thật vậy, ta có ( )( )( )

1 1

x − =x x xx+ x +x +

Dễ thầy x x( −1)(x+1) tích số ngun liên tiếp nên chia hết cho

Theo định lí Ơle ( )

0 mod ,

x − ≡x ∀ ∈x  , tức

xx chia hết cho

Vậy

xx chia hết cho BCNN 6; 7( )=42 Khẳng định ( )1 chứng minh

Từđó

( )7 ( )7 ( )7

27n 10 10n 27 5n 10 27

 + +  + + +  + + + 

     

( )7 ( )7 ( )7 ( )

27n 10 10n 27 5n 10 27 mod 42

≡ + + + + + + + +

( )

27n 10 10n 27 5n 10 27 mod 42

≡ + + + + + + + +

( )( )

42 n mod 42

≡ +

( )

0 mod 42

Từđó ta có khẳng định tốn

b Đặt a= +x y Sử dụng bất đẳng thức AM-GM, ta có ( )

2 2

4

x y a

(30)

Hay

2

5

1

2a

 +  ≥

 

 

Từđó, ta có 2( 1)

a≥ − Suy

( )2

2 2

17 17 16 17 18 17 2

2

P= x + y + xy= axyaa ≥ − = −

Dấu “=” xảy

5

x= =y

Câu

a Sử dụng định lí Talet tam giác LKE với JHEK, ta có

LH LJ

HK = JE

Sử dụng định lí Talet tam giác JHE với FL EK , ta có

FL LJ

EK = JE

Do

FL LH

EK = HK

Hai tam giác FLH EKH có 90

FLH EKH

∠ = ∠ = FL LH

EK = HK nên

FLH EKH LFH KEH

∆ ∽∆ ⇒ ∠ = ∠ Mặt khác, ta lại có ∠LFH = ∠FHJ (so le trong) ∠KEH = ∠EHJ (so le trong) Do HJ phân giác góc EHF

b Go HJFL nên SFJL =SFLH Suy SBFJL =SBFL+SFLH =SBFH (1)

Chứng minh tương tự, ta có SCEJK =SCEH

(2)

Theo chứng minh câu a, hai tam giác FTL EKH đồng dạng nên

FHB FHL EHK EHC

∠ = ∠ = ∠ = ∠

Hai tam giác FHB EHC có ∠FBH = ∠ECHFHB= ∠ECH nên đông

dạng với Suy FBH 22 ECH

S BF

S = CE

Ta kết hợp (1) (2), ta thu

2

2

FBH

ECH

S S BF

S = S = CE

(31)

c Khơng tính tổng qt, ta giả sử P nằm phíc với B so với AD hình vẽởtrên Gọi M giao điểm PJ EK Áp dụng định lí

Menelaus cho tam giác KFE với cát tuyến MJP, ta có

MK PE JF

ME PF JK =

Mà hai tam giácBFH CEH đồng dạng với có FL EK hai

đường cao tương ứng nên JF FL BF

JK = EK = CE

Suy MK PE BF

ME PF CE =

(3)

Để chứng minh ba điểm P J D, , thẳng hàng, ta cần chứng minh M D J, ,

thẳng hàng Theo định lí Menelaus đảo áp dụng cho tam giác LKE, điều

tương đương với ta phải chứng minh

MK JE DL

ME JL DK =

Lại có JE EK CE

JL = FL = BF .1

DL DL DC DB AF AC AF

DK = DB DK DC = AB AE = AE

Do đó, cần chứng minh MK CE AF

ME BF AE =

(4)

Kết hợp (3) (4) , ta đưa toán chứng minh CE AF PE BF

BF AE = PF CE

Hay AF PF BF22

AE PE =CE

(32)

Gọi T, N tiếp điểm đường tròn (I) với AB AC, Đặt , , ,

a= AB= AC x=BD=CD y=PF =TF z=PE=EN Ta chứng minh

( )( )

2

x x y x z

a

x yz

+ +

=

− (6)

Thật vậy, sử dụng định lí cosin tam giác ABC AEF, ta có

2 2 2

2 cos

AE AF EF AB AC BC

A

AE AF AB AC

+ − + −

= =

Suy

( ) ( ) ( )

( )( )

2 2 2 2

2

2

a x y a x z y z a x

a x y a x z a

− − + − − − + = −

− − − − ,

Hay

( ) ( ) ( )

( )( )

2 2 2 2

2

2

2 a x y a x z y z a x

a x y a x z a

− − + − − − + −

− = −

− − − −

Từđây, ta có

( ) ( )( )

2

2

2

yz x

ax+ +y z a+ +x y x+z = a ,

Hay

( ) 2( ) 2( )( )

2

xyz ax x+ +y z a+x x+y x+z =

Như thế, ta có ( ) ( ) ( )( )

ax  xyz ax x+y x+z =

Do a>x nên (6) chứng minh, Sử dụng (6) vừa chứng minh ta có

( )( )

( )( ) (( ))

2 2

2

2

2

x x y x z

x y

x y

AF PF a x y y x yz y BF

x x y x z

AE PE a x z z z x z CE

x z

x yz

+ +

− − +

− − −

= = = =

+ −

− − +

− − −

Đẳng thức (5) chứng minh Ta có điều phải chứng minh

Bài Đặt Mn ={x x| ∈ , x ≤2n−1} Ta chứng minh mệnh đề tổng quát: Trong 2n+1 số phân biệt từtập hợp Mn, tồn ba sốphân biệt có tổng bẳng 0 Ta chứng minh

phương pháp phản chứng Giả sử tồn sốnguyên dương n cho thể chọn 2n+1 số

phân biệt từ tập hợp Mn mà khơng có ba số phân biệt có tổng Gọi n

là số nhỏ có tính chất vây Khi n>1 ( với n=1 mệnh đềđúng) Vì n

số nhỏ làm cho mệnh đềkhông nên mệnh đềđúng với n−1 Nếu số

được chọn có 2n−1 số thuộc Mn−1 mệnh đềđúng với n−1, tồn ba số

phân biệt sốđược chọn có tổng Mẫu thuẫn Vậy có tối đa 2n−2 sốđược

chọn thuộc Mn−1 Suy bốn số − + − +2n 2, 2n 1, 2n−2, 2n−1, có ba sốđược

(33)

• Nếu hai số cặp (− +2n 1, 2n−1) chọn Chia tập

{ }

\ 1, 1, n

M − +n n− thành 2n−2 cặp

(1; 2n−2 , 2; 2) ( n−3 ,) … − − +,( 1; 2n ,) … − + −,( n 1, n) ta thấy từmỗi cặp ta

chọn tối đa số Suy lấy tối đa 2+ n− =2 2n số Mẫu

thuẫn

• Nếu chỉcó số cặp (− +2n 1, 2n−1) chọn theo lí luận ởtrên,

cặp (− +2n 2, 2n−2) chọn Khơngmất tính tổng qt ta giả sử 2n−1

được chọn cịn 2− n khơng chọn Lúc chia phần tử lại

thành 2n−5 cặp

(1; 2n−3 , 2; 2) ( n−4 ,) …,(n−2;n), ( 2; 2− − + … − + − −n 3, ,( n 3; n ,) ba số (− + − + −n 2, n 1, n) phần tử lẻ cặp n−1 Từmỗi cặp ta lấy tối

đa số, từ ba sốta lấy tối đa số Từđó ta lấy tối đa 2+ n− + + =5 1 2n số Mẫu thuẫn

Vậy trường hợp dẫn đến mẫu thuẩn, tức điều giả sửsai Mệnh đề chứng minh Áp dụng mệnh đề cho n=1010 ta có điều phải chứng minh

Đề s

Bài

a) Giải phương trình x x x x 12− + 3+ = +

Điều kiện xác định phương trình x≥ −1 Để ý x x 02− + > .

Đặt a= x 1; b+ = x x a 0; b 02− + ( ≥ > ) Khi phương trình cho viết lại thành ( )( )

2

b 2ab 2a− + = ⇔ b b 2a− + + =0

Do a 0; b 0≥ > nên ta có b 2a 0+ + > Khi từ phương trình ta b 1= Do ta có phương trình x x 12− + = ⇔x x 02− = ⇔ ∈x { }0;1 .

Kết hợp với điều kiện xác định ta tập nghiệm phương trình S={ }0;1

b) Giải hệ phương trình xy y2 22 y

x 2y 2xy x

 + = +

 

+ + = +



Hệ phương trình cho viết lại thành 2xy 2y2 2 2 2y

x 2y 2xy x

 + = +

 

+ + = +



(34)

2

2

x 4y 4xy x 2y x 2y x 2y x 2y x 2y x 2y x 2y

x 2y x 2y

x 2y x 2y

+ + = + + ⇔ + = + + ⇔ + − + − =

 + − =  = −

⇔ + − + + = ⇔ ⇔

+ + = = − −

 

+ Thế x 2y= − vào phương trình thứ hệ cho ta

( ) 2 2 ( )2

y 2y y− + = + ⇔y y −2y 0+ = ⇔ y 1− = ⇔ =0 y

Từ tương ứng ta x 1=

+ Thế x= −2y 2− vào phương trình thứ hệ cho ta

( ) 2 5

y 2y y y y 3y y ;

2

− − − + 

 

− − + = + ⇔ + + = ⇔ ∈  

 

 

Từ với y

− −

= ta x 1= + với y

− +

= ta x 1= −

Vậy hệ phương trình có nghiệm ( ) ( )x; y 1;1 , 5; , 5;

2

 − −   − + 

=  +    − 

   

Bài

a) Tìm tất cặp số nguyên ( )x; y thỏa mãn (x y 3x 2y+ )( + )2 =2x y 1+ −

Để ý 2x y+ =(3x 2y+ ) (− x y+ ) nên phương trình cho viết lại thành

( )( ) (2 ) ( ) x y 3x 2y+ + = 3x 2y+ − x y 1+ −

Đặt a x y; b 3x 2y= + = + Khi ta có ab2 = − −b a 1 hay a b 1( + = −) b 1.

Từ suy b 1− chia hết cho b 12+ Do ta b b b 12 + −( − )( + ) chia hết cho

b 1+ hay chia hết cho b 12+ Suy b 1; 22+ ∈{ } nên b∈ −{ 1;0;1}.

+ Với b= −1 ta a= −1, ta ( ) (x; y = 1; 2− ) + Với b 0= ta a= −1, ta ( ) (x; y = 2; 3− ) + Với b 1= ta a 0= , ta ( ) (x; y = 1; 1− )

(35)

b) Với a, b số thực dương thay đổi thỏa mãn điều kiện a 2b b

+ = + Tìm giá trị

nhỏ biểu thức M a b

a 2b b 2a

= +

+ +

+ Lời giải Ta chứng minh M 2≥ với dấu đẳng thức xẩy chẳng hạn a b 3= = Thật vậy, áp dụng bất đẳng thức AM – GM ta có

( )

b b 3b 2b 3b b 3b

3b b 2a a 2b b 2a+ = 3b b 2a+ ≥ + + = +

Như ta cần a b 3b a 2b a 2b+ + + ≥

Đặt x a 2b; y b(x 0; y 0)

= + = ≥ ≥ Khi giả thiết viết lại thành x y 2− = Cũng từ ta có b 3y ;a x 6y= = 2− Bất đẳng thức cần chứng minh viết lại

thành x 6y2 9y23 x 6y2 9y23 x y

x x x x

− −

+ ≥ ⇔ + ≥ −

Biến đổi tương đương bất đẳng thức ta

( ) ( )

( ) ( )

2 3 3

2

3 2 2

x x 6y 9y x x y x 6xy 9y x x y 9y 6xy x y y 9y 6xy x y 3y x

− + ≥ − ⇔ − + ≥ −

⇔ − + ≥ ⇔ − + ≥ ⇔ − ≥

Bất đẳng thức cuối Vậy tốn giải hồn tồn

+ Lời giải Xét biểu thức M b a b b

3 a 2b b 2a

+ = + +

+ +

Áp dụng bất đẳng thức Cauchy – Schwarz dạng phân thức ta có

( )2

2 2 a b b

b a b b

M

3 a a 2b b b 2a b 3b a a 2b b b 2a b 3b

+ +

+ = + + ≥

+ + + + + +

Áp dụng tiếp bất đẳng thức Cauchy – Schwarz ta lại có

( ) ( )( )

b b 2a b 3b b b 2a+ + = + + 3b ≤b 1 b 2a 3b+ + + =2b a 2b+

Từ ( ) ( )

2

a b b a 2b a 2b

a 2b a a 2b b b 2a b 3b a a 2b 2b a 2b a 2b

+ + + +

≥ = = +

(36)

Suy M b a 2b

+ ≥ + nên M a 2b b

3

≥ + − = Vậy tốn giải hồn toàn

Bài

a) Chứng minh hai tam giác BKM DEF đồng dạng với nhau.

Đường tròn ( )I nội tiếp tam giác ABC nên ta có BD BF tiếp tuyến Do BI đường trung trực đoạn thẳng DF nên BI vng góc với DF M Từ BMDK nội tiếp đường trịn, BMK BDK CDE  = = Cũng CE tiếp tuyến với đường tròn

( )I E nên ta có CDE DFE = Từ suy  

BMK DFE= Mặt khác BKM BDM DEF  = =

nên hai tam giác BKM DEF đồng dạng

b) Chứng minh hai đường thẳng MK NL song song với nhau.

Ta có tứ giác BKMD CLDN nội tiếp đường tròn nên suy DMK DBK =

 

DCN DLN= Mặt khác BK song song với CN nên ta có DBK DCN = Từ suy  

DMK DLN= nên MK song song với LN

c) Chứng minh đường thẳng JX vng góc với đường thẳng EF.

Ta có DMK DCN 90 CDN 90 = = 0−= 0−DFE 90= 0−DMN, KMN 90= 0 Do tứ

giác KMNL hình thang vng Ta có J trung điểm KL nên J nằm đường trung trực đoạn thẳng MN hay JM JN= Mặt khác XM XN 1ID

2

= = nên suy X nằm đường trung trực MN Do XJ vng góc với MN Trong tam giác DEF MN đường trung bình nên ta có MN song song với EF Do suy JX vng góc với EF

Bài

L K

X

J I F

E

D N

C B

A

(37)

Gọi m, n theo thứ tự số đường thẳng qua P Q Gọi S số miền tạo thành Do đường thẳng qua điểm P hặc điểm Q nên ta có m n 10+ = Ta xét trường hợp sau

+ Trường hợp Nếu m 0= n 0= , chẳng hạn m 0= tất 10 đường thẳng cho đồng quy P Khi dễ thấy số miền tạo mặt phẳng 20 Do ta có

S 20=

+ Trường hợp Nếu m 0> n 0> , m 1≥ n 1≥ Từ mặt phẳng cho với hai điểm P Q ta vẽ thêm m đường thẳng qua điểm P, số miền tạo thành 2m Lần lượt vẽ thêm đường thẳng qua điểm Q Khi vẽ đường thẳng đường thẳng cắt m đường thẳng qua P m điểm phân biệt, m điểm phân biệt chia đường thẳng vừa vẽ thành m 1+ phần Nói cách khác thìđường thẳng vừa vẽ qua (vì chia đôi) m 1+ miền 2m miền tạo Do lúc số miền tạo 2m m 1+( + )

Kể từ đường thẳng thứ hai đến đường thẳng thứ n qua điểm Q đường cắt m đường thẳng phân biệt qua điểm P m điểm phân biệt khác Q Các điểm phân biệt với điểm Q chia đường thẳng vừa vẽ thành m 2+ phần Do mối lần vẽ đường thẳng số miền tăng thêm m 2+ Do số miền tao từ đường cịn lại qua Q (n m 2− )( + ) Như ta có

( ) ( )( ) ( )

S 2m m 1= + + + n m 2− + =mn 2m 2n mn m n mn 19+ + − = + + − = +

Áp dụng bất đẳng thức AM – GM ta có mn 1(m n)2 1.100 25

4

≤ + = =

Từ ta S 25 19 44≤ + = Dấu xẩy m n 5= =

Vậy số miền tạo tối đa 44 số đường thẳng qua P số đường thẳng qua Q

Đề s

Câu

a) Giải hệphương trình :

( )( )

3 3

( )

7 1 31

xy x y

x y x y x y

+ =



 + + + + + =



(38)

( ) ( ) ( )

3

2

2

2

( )( ) 7( 1) 31

( )

( ) 31

xy x y

x y x xy y xy x y xy

xy x y

x y x y xy xy x y xy

 + =

 ⇔ 

+ − + + + + + + =



+ = 

⇔  +  + − + +  + + +  =

 

  

Đặt a= +x y b; = xythì hệtrên trởthành: ( 2 ) 3 ( )

2

3 31

ab

a a b b a b

= 

 − + + + + = 

( )

( ) ( ) ( )

( )

( ) ( )

( ) ( )

( )

( )

( )

( )

3

2

3

3

3

3

2

2

3 31

3 31

3 ( ) 7( ) 24

6( ) 3.2 24

30

27 ( )

( 3) 3( ) 10

3 3( ) 10

ab

a ab b a b

a b a b ab ab a b

a b ab a b ab a b

a b a b a b

a b a b

a b a b

a b a b a b

a b do a b a b

= 

⇔  − + + + + = 

 

⇒ +  + − − + + + =

⇔ + − + − + + − =

⇒ + − + − + + − =

⇔ + + + − =

⇔ + − + + =

 

⇔ + −  + + + +  =

⇒ + = + + + + >

3

2

a b a

ab b

+ = =

 

⇒ ⇒ 

= =

  (do ( )

2

4 )

a = x+ yxy = b

2

1

x y

x y

xy

+ = 

⇒ ⇒ = =

= 

Vậy hệcó nghiệm ( ) ( )x y; = 1;1

b) Giải phương trình: 3+ x(3 2− x)=7 x+5 2− x Điều kiện xác định:

2

x

≤ ≤

Đặt a= x b, = 2− x a b( , ≥0) Khi phương trình tươngđương với:

( ) ( )

( )( )

2 2

2

9

2

2

2 3

2 ( 2)

2

+ = +

⇒ + + + = +

+ = 

⇔ + − + + − − − + =

⇔ + − + + − − + − =

⇔ + − + − =

ab a b

a b ab a b

a b

a ab a ab b b a b

a a b b a b a b

(39)

( ) ( )

( )( )

( )2

2 (2 ) 10

2 3 3

1

3 1 ( )

3

1 1 1 ( )

+ = ⇒ = − ⇒ + − = + 

⇔  + = ⇒ = − ⇒ + − = + − 

 − − = = ⇒ =

⇔  ⇔ 

− =

  = ⇒ =

a b b a a a a

a b b a a a a a

a a a x tm

a a x tm

Vậy phương trình có tập nghiệm 1;1

S =  

 

Câu

a) Cho x,y…

Ta có:( ) ( )

( ) ( ) ( )( )

2 2

2

2 2

2

x xy y xy y x x xy y x

x xy xy y x y x y x y

− − + − − = − − −

= + − + − + = + − −

Lại có: 2

2 ,

xxyy xyyxchia hết cho (x y)(x 2y 1)

⇒ + − − chia hết cho

TH1: Nếu x+ychia hết cho5 y≡ −x(mod 5)

( )

2 2

0 x 2xy y x 2x x x 3x (mod 5)

⇒ ≡ − − ≡ + + = + , xchia hết cho chia

+)Nếu xchia hết cho ycũng vậy, toán chứng minh

+)Nếu xchia cho dư y chia dư 2,

2

2x + y +2x+ ≡y 2.9+ +4 2.3=30≡0(mod 5) Ta có điều phải chứng minh

TH2) Nếu x−2y−1chia hết cho x≡2y+1 mod 5( )

( )2 ( )

2

0 x 2xy y 2y (y y 1) y y mod

⇒ ≡ − − ≡ + − + − = +

Do ychia dư xcũng chia dư nên:

( )

2

2x + y +2x+ =y 2.16 16+ +2.4+ =4 60≡0 mod Vậy ta có điều phải chứng minh

b) Cho………

Nếu tồn n:1≤ ≤n 50 :a1+a2+ +an =50thì kết luận tốn hiểu nhiên

Xét:

1

1

49

1 49 :

51

n

n n

a a a

n a

a a a + +

+ + + ≤ 

≤ ≤  + + ⇒ ≥

+ ≥ 

1

2 50

1: 49

49

n n

n n

TH a a a a

a a a

+ + +

= ⇒ + + + =

+ + + =

Nên n≤24⇒a1≤an+2;a2≤an+3; ;ana2n+1

1 2 2 49 50

49 a a an an+ an+ a n+ an+ a a

⇒ = + + + ≤ + + + < + + +

(40)

1 1

2

25 49 25

48; 50

n

n n

n a a a na a a a

a a a a +

≥ ⇒ = + + + ≥ ≥ ⇒ < ⇒ =

⇒ + = + + =

TH2: an+1≥3

( )

( ) ( )

( ) ( ) ( )

2 50

2

1 16 17 17 17

17 17 17

100 49

49 49 49 33

49 16 16 16 17

2

n n n

n

n

a a a a a a

n a n n

a a a a a n a a

a a a a a

+ + +

+

+ + + = − + + + ≤

⇒ ≥ − ≥ − ⇒ ≥

⇒ ≥ + + + + + + ≥ + − ≥ +

⇒ < ⇒ = ⇒ = = =

Nếu an+1<18đặt a1+a2+ +an+1=50+k k( ≥1)

( )

1

1

18 50 49

17 50

n

k n

a k k

k a a

+

+ +

⇒ ≥ ≥ + − = +

⇒ ≤ ⇒ + + =

Nếu an+1≥19

( ) ( )

1 45

49 49 49 19 47

n

n a n n

a a a

+

⇒ ≥ − ≥ − → ≥

⇒ = = = =

( ) ( ) ( ) ( )

45 2 44 45 n 44 44 45 44 47 44 49

a ≥ ⇒ a +a + +a + a + +a ≥ + na ≥ + − >

Đặt an+1=50−k(0≤ ≤k 31)⇒ +a1 +ak +an+1=50(do a1 = =ak =1)

Vậy ta có điều phải chứng minh

Câu 3:

a) Chứng minh điểm K, M, Q thẳng hàng

Do tứ giác BCKQBCDAnội tiếp nên: CKQ  =CBQ=CADKQ/ /AD.Mặt khác / /

MK ADnên K, M, Q thẳng hàng

b) Đường tròn ngoại tiếp tam giác………….

(41)

/ /

MQ ADnên   RMQ=RLD=ETD⇒tứgiác RTMQ nội tiếp

Chứng minh tương tựRMSQ nội tiếp đó: M S Q R T, , , , thuộc đường tròn

c) Chứng minh đường tròn ngoại tiếp

Bổđề: cho tam giác ABC, M nằm d / /BClấy E khác M d, AM cắt BC

I Đường qua M/ / ABcắt BE J , IJ / /AE

Chứng minh MJ cắt AE, AC S T, ME cắt AC G Ta có MG//BC suy MA AG

MI = GC ,

ME cắt AB P ta có: MS AP AG MA AE/ /IJ MJ = PB = GC = MI

Quay trở lại toán:

AM cắt BC, (O) I J khác A Áp dụng bổđềta có: IR/ /AE IQ, / /AB Do

  

IRE = AEC= AJC ⇒nên RIJC tứ giác nội tiếp Chứng minh tương tựta có DQIJ

tứ giác nội tiếp

Do đó:     

2 180

RJI +IJQ+RPD= PCD+CPD= nên RPQJ nội tiếp Kẻ tiếp tuyến Jxcủa

(O) Ta có:

         

  

xJR xJA RJA ADJ PDC ADP MAC

ADP PAD APB

PEJ MAC PED

= − = − = +

= + =

⇒ = =

Suy : Jx tiếp xúc với (PQR)hay ta thu được: (PQR)tiếp xúc với ( )O

Vậy ta có điều phải chứng minh

Câu 4: Áp dụng BĐT Cauchy-Schwarz ta có:

1 1

2

2

2

ab bc ab bc

a b b c a b b c a b b c a b b c

a c b b a c b b

a b b c a b b c a b b c a b b c

a b c b

a b a b b c b c

      

+ + ≤ + +

 + +  + +   + +   + + 

 

 

      

=  +  +  ≤ +  + + 

+ + + + + + + +

      

   

= +  + + =

+ + + +

   

Vậy ta có điều phải chứng minh

(42)

Đề s

Câu (3.5 điểm).

a) Giải hệ phương trình

2

2

1

x y xy

x x y y

    

   

Phương trình thứ hệ viết lại thành xyx2 y21 Thế vào phương trình thứ hai hệ ta

 

    

  

2 3 3

3 3 2

2

2

1 2

0

0

2

0

2

x x x y y x x xy x y x xy y

x y xy y x y x xy y y x y

x y x y

x y x xy y

x y

x xy y

            

           

    

 

          

  

 

Dễ thấy x  y khơng thỏa mãn phương trình thứ hệ Do từ xy thay vào phương trình thứ hệ ta

2 1 1 1

x        x x y

Vậy hệ phương trình cho có nghiệm     x y;  1;1 ,  1; 1

b) Giải phương trình 2x 1 x  1  x  1 1x2 1x2

Điều kiện xác định phương trình   1 x Biến đổi tương đương phương trình ta

    

         

     

  

2

2

2 1 1

2 1 2 1 1

2 1 2 1 1

1 1

x x x x x

x x x x x x x x

x x x x x x x x

x x x x

       

            

            

      

Đặt ax 1;b  1x a 0;b 0, ta có a2 b2 2.

Phương trình viết lại thành a a2 b2b Từ ta có hẹ phương trình  

2 2

2

a b

a a b b

   

  



(43)

+ Xét trường hợp b 0, phương trình thứ hệ tương đương với

 2 2

b abb

Khi ta có hệ hương trình    

2 2

3 2

2

2

a b b b

b a b b

a a b b

a a b b

 

     

 

 

 

     

 



Từ ta a b2 b3 a3 a b2 2ba3 b3  a b.

Thay vào phương trình thứ hệ ý đến điều kiện ta

2 1 1 1

a      a a b

Từ ta x  1 1   x x 0, thỏa mãn điều kiện xác định Vậy phương trình có nghiệm x 0

Câu 2(2.5 điểm).

a) Chứng minh không tồn số nguyên x, y thỏa mãn đẳng thức

2

12x 26xy 15y 4617

Trước hết ta chứng minh bổ đề: Với số ngun tố có dạng p 4k 3 ta ln có

 

2 a p ,

a b p a b Z

b p



   

 

 Thật vậy, ta xét hai trường hợp sau

+ Trường hợp Nếu hai số a b chia hết cho p ta suy điều cần chứng minh

+ Trường hợp Nếu hai số a b khoog chia hết cho p Khi ta có    a p;  b p; 1

Theo định lí Fecmat ta có  

     

1

4

1

1 mod mod

2 mod

1 mod mod

p k

k k

p k

a p a p

a b p

b p b p

 

 

 

 

   

 

    

 

   

 

 

 

Mặt khác ta có a4k2 b4k2  a2  k  b2 2k1 chia hết cho a2 b2 nên chia hết cho p. Từ suy chia hết cho p, mà p số nguyên tố nên ta p 2 Điều mâu

thuẫn p số nguyên tố lẻ

Như trường hợp không xẩy hay bổ đề chứng minh

(44)

 

     

2 2

2 2

2

2 2

12 12 15 38 19 12 12 15 19

3 4 19 4 19

4 4 19 2 19

x xy y xy x xy y

x xy y x xy y

x xy y y x y y

     

     

      

 

 

 

Do 19 số nguyên tố có dạng 4k 3 nên áp dụng bổ đề ta suy

2 19 19 19 19

2 19 19 19 19

x y x y x x

y y y y

   

     

   

   

   

   

   

   

   

Từ ta 4x24xy 5y2192 Điều dẫn đến mâu thuẫn 4617 khơng chia hết cho 192.

Vậy không tồn cặp số nguyên  x y; thỏa mãn yêu cầu toán

b) Với a, b số thực dương Tìm giá trị lớn nhát biểu thức

  3

1 1

M a b

ab

a b b a

 

 

    

 

 

Áp dụng bất đẳng thức Bunhiacopxki ta có

 3   2  3   2

;

a b b a b b a a a b

a b

   

   

         

   

Từ ta 3 3

1

;

b a

a b a a b b

a b a b

a b b a

 

   

 

  Do suy

  3 3

1 1

1 a b a b a b b a a b a b

a b

a b a b a b

a b a b a b b a

    

   

 

       

  

   

 

Suy    

   

1

1

1

a b ab a b a b a b ab a b

a b

M

a b ab a b ab ab a b

        

    

  

Vậy giá trị lớn M 1, đạt a  b Câu 3(3.0 điểm).

(45)

a) Chứng minh CBK ABI Ta có

   ;  

CBKCBDKBD ABIABDIBD

Lại có ABD CBD Mặt khác ID song song với BK nên ta có

  

IBDIDBDBK Từ suy

 

CBKABI

b) Chứng minh KCKB

Dễ thấy KJC LJI Lại có IJLIBJ LBI Kết hợp với CBK ABI ta

 

CBKCJK nên tứ giác BCKJ nội tiếp đường trịn Do BJC BKC900 hay

KCKB

c) Chứng minh bốn điểm C, K, I ,L nằm đường tròn

Tam giác IJL cân I nên ta có ILJ  IJLIBJ Mà ta có IBJ JBK nên ILJ JBK

Mặt khác tứ giác BCKJ nội tiếp đường tròn nên JBK JCK

Từ ta ILJ JBK JCK nên suy tứ giác CKIL nội tiếp đường tròn Câu (1.0 điểm)

Trước hết ta chứng minh bổ đề: Với hợp số n 4 ta ln có n1 ! n

Thật vậy, n hợp số n 4 nên ta viết na b a b, N;1a b, n

Khi ta suy 2a b,  n Từ dễ thấy ta ln có n1 ! n Trở lại toán Ta thấy ann ann ain i 1;n1 Khi ta có

1

i n

a a a n a a a n

  

 , điều mâu thuẫn với giả thiết toán Do ann Giả sử n hợp số n 4, theo bổ đề ta có

 

1 .2 n 1 !

a a a   n n

Mặt khác theo ta lại có a a a n1 .2 n Như a a a1 .2 n1 a a a n1 .2 n có số dư chia cho n Điều mâu thuẫn với giả thiết toán

K J

I L

D

C B

(46)

Từ suy n 4 Mà n hợp số nên ta n 4

Ta thấy với n 4 số 1; 3;2; viết dãy số 1;1.3;1.3.2;1.3.2.4 chia cho có số dư

lần lượt 1; 3;2;

Đề s

Câu

1) Hệ phương trình tương đương với     2

3

x y x y

x y xy

    

    

Do ta có phương trình 3  3 1

x

x y xy x y

y

            

 + Với 2 32 2

3

x x

x y xy y y

 

   

 

 

 

       

 

 

 

, hệ phương trình vơ nghiệm

+ Với 2 2 2 1;

2;

3

y y x y

x y

x y xy x x

  

     

 

   

  

           

  

 

 

Vậy hệ phương trình cho có nghiệm    x y;  1;1

2 Với a, b số thực dương thỏa mãn ab  a b Chứng minh rằng:

  

2 2 2

1

1 2 1 1

a b ab

a b a b

 

   

Cách Do ab  a b nên ta

     

2 1 1 ; 1 1

a  aab  a b ab ab  bab  a b ab b

Đẳng thức cần chứng minh tương đương với

          

   

        

        

2

2

1

1 2 1 1

1 1

1 2 1 1

1 1 1

a b ab

a b a a b b a b a b

a b b a ab

a b a b a b a b

a b a b a b ab a b

 

      

   

 

     

             

(47)

Cách Đẳng thức cần chứng minh tương đương với

   

     

2

2 2 2

1 1

1 2 1 1

a b b a ab

a b a b

   

   

Mà ta có a b  1 b a2 1ab ab 1 nên đẳng thức tương đương với

       

   

2 2 2 2 2 2 2

2

2

1 2 1 1 4 1

2

1

1 1

a b a b a b a b ab a b

a b

a b ab a b ab ab a b

           

 

           

Do đẳng thức cuối nên đẳng thức cần chứng minh Câu

1 Câu (2.5 điểm).

1 Giả sử p q số nguyên tố thỏa mãn đẳng thức p p  1 q q 21. a) Chứng minh tồn số nguyên dương k cho p 1 kq q,  1 kp Nếu pq ta có 1 1

1

p q

p q

p q

   

      

 , điều vô lí p, q số ngun tố Do pq, p q số nguyên tố nên p1q q21p.

Như tồn số nguyên dương m, n thỏa mãn p 1 mq q; 2 1 np, thay vào đẳng thức cho ta mn Do tồn số nguyên dương k cho 2

1

p kq

q kp

   

    b) Tìm tất số nguyên tố p, q thỏa mãn đẳng thức p p  1 q q 21.

Thế pkq1 vào hệ thức q2  1 kp ta q2  1 k kq 1q2k q2   k 1 0. Xem phương trình phương trình bậc hai ẩn q, để phương trình có nghiệm ngun dương

 

4 4 1 4 4

k k k k

       phải số phương

Ta có k4 k4 4k  4 k2 22 nên ta  k2 12. Từ ta k4 4k  4 k2 12  k k2  k 1.

(48)

Vậy số p 3;q 2 số nguyên tố cần tìm

2 Với a, b, c số thực dương thỏa mãn abbccaabc 2 Tìm giá trị lớn biểuthức

2 2

1 1

2 2 2

a b c

M

a a b b c c

  

  

     

Biến đổi giả thiết abbccaabc 2 ta

         

        

1 1 1

1 1 1

1 1 1

a b c a b c

a b a c b c

        

   

     

Đặt ; ;

1 1

x y z

a b c

  

   , ta thu xyyzzx 1

Biểu thức M viết lại thành

 2  2  2

2 2

2 2

1

1

1 1

1 1

1 1 1 1 1 1

1 1

a b c x y z

M

a b c

x y z

x y z

x y z

  

     

        

  

  

Để ý ta có x2  1 x2 xyyzzx xy x z Áp dụng tương tự ta

        

     

             

2 2

x y z

M

x y x z y z x y x z y z

x y z y z x z x y xy yz zx

x y y z z x x y y z z x x y y z z x

  

     

      

  

        

Ta chứng minh 9xy y z z x8x  y z xy yzzx Vì x  y z2 3xyyzzx3 nên x   y z Nên ta

    

2 9 3

8 4 3

9

M

x y z

x y z xy yz zx

   

 

   

Vậy giá trị lớn M 3

4 , đạ

1

3

3

(49)

Cho tam giác ABC nhọn với ABAC Gọi E, F trung điểm CA, AB Đường trung trực EF cắt BC D Giả sử P nằm góc EAF nằm ngồi tam

giác AEF cho PEC DEF PEB DFE Đường thẳng PA cắt đường tròn ngoại

tiếp tam giác PEF Q khác P

Hình 1

Hình

a) Chứng minh EQF BAC EDF Vì tứ giác PEQF nội tiếp đường trịn nên ta có

     

 

     

0

180 EF

EQF EPF P PFE DEC DFB

EAD EDA FAD FDA BAC EDF

     

     

b) Tiếp tuyến P đường tròn ngoại tiếp tam giác PEF cắt CA, AB M, N Chứng minh bốn điểm C, M, B, N nằ đường tròn Gọi đường tròn đường trịn  K

Khơng tính tổng quát ta giả sử M nằm A, C N nằm tia đối tia BA (các trường hợp cịn lại chứng minh tương tự) Khi ta có

    

    

0

0

180 180 EF

180 EF 180 EF

MNB NPF PFN P DFE

DEC D C AEF ACB

     

      

Do tứ giác NCMB nội tiếp đường tròn  K

F I

K N

M R

Q

P E

D C

B A

Z

Y X

A

B C

D

E

(50)

c) Chứng minh đường tròn  K tiếp xúc với đường tròn ngại tiếp tam giác AEF

Ta có nhận xét: PAB DAC Thật vậy, gọi X, Y, Z điểm đối xứng với P qua EF, AE, AF từ giả thiết PEC DEF suy DEY DEX, mà ta có EXEY nên

DXDY Tương tự ta có DXDZ nên DXDYXZ, lạ có AYAPAZ ta

được DAZ DAY Kết hợp tính đối xứng ta PAB DAC

Đường tròn ngoại tiếp tam giác PEM cắt đường tròn ngoại tiếp tam giác AEF R khác E Ta thấy RPN REMRFA nên tứ giác PRFN nội tiếp đường tròn Lại tứ giác BNCM nội tiếp đường tròn nên BAC 1800NBC 1800 CMN AMN.

Từ ta thu A RE AFEABC AMN 1800PRE nên ba điểm A, R, P thẳng hàng

Gọi giao điểm EF AD I, theo tính chất đường trung bình I trung điểm AD

Ta lại có AEI ARF, kết hợp với nhận xét ta AEI ∽ARF

Từ suy AED ∽ARB Ta thu

    EF  EF  ER 

ABRADEDECDAEPPABPFERPRMP

Từ tứ giác NMRB nội tiếp đường trịn hay năm điểm M, N, R, B, C nằm đường trịn  K

Lại có ERMEPM EFP EFRRFP RAE RNM

Vậy đường tròn ngoại tiếp tam giác REF đường tròn  K tiếp xúc R

Câu (1.0 điểm).

Cho n số nguyên dương với n 5 Xét đa giác lồi n cạnh Người ta muốn kẻ số đường chéo đa giác mà đường chéo chia đa giác thành k miền, mõi miền mọt ngũ giác lồi (hai miền khơng có điểm chung trong)

a) Chứng minh ta thực với n 2018,k 672

Kí hiệu đa giác 2018 cạnh A A A A1 3 2018, kẻ đường chéo A A A A A A1 5; 1 8; 1 11; ;A A1 2015 đa giác A A A A1 3 2018 chia thành 672 ngũ giác lồi gồm:

1 5; 8; ; 2012 2013 2014 2015; 2015 2016 2017 2018

(51)

b) Với n 2017,k 672 ta thực khơng? Hãy giải thích

Giả sử ta chia đa giác lồi 2017 cạnh thành 672 ngũ giác lồi đường chéo Gọi p số giao điểm đường chéo nằm đa giác Do đỉnh ngũ giác lồi là đỉnh đa giác cho p giao điểm đường chéo nên tổng số góc ngũ giác

   

0 0

.360 2017 180 2015 180

p    p

Mặt khác số ngũ giác lồi 672, ngũ giác lồi có tổng số góc đỉnh 3.1800 nên tổng số góc ngũ giác lồi 672.3.1800.

Từ ta 2 2015 180 672.3.1800

2

p   p , vô lý Vậy ta thực với n 2017,k 672

Đề s

Câu 1(3.5 điểm).

a) Giải hệ phương trình  

  

3

2

4

1

x y xy x y

xy x y

    



   



Phân tích lời giải Hệ phương trình cho hệ phương trình đối xứng dạng 1,

đó ta sử dụng phép đặt ẩn phụ S  x y P; xy Tuy nhiên để đơn giản hóa ta cần

biến đổi hệ phương trình trước.Biến đổi tương đương hệ phương trình ta  

         

   

         

3 3

2 2

3 3

2

3

4

1 1 2 4

2 2 4

1 4

x y xy x y xy x y

x y xy x y

xy x y xy x y xy

x y xy x y x y xy x y

xy x y xy xy x y xy xy



       

     

 

   

          

 

   

 

         

 

   

          

 

    

(52)

        

     

   

    

3

2

2

2 2 2

2

1

1

1

1 1 1 0 1; 1

1

0

2 0

x y xy x y xy x y xy xy

x y x y xy xy x y xy

x y xy x y xy

x

x y xy x y x y

y

x y

x y

x y xy x y

       

         

 

        

 

 

            

   

          

 

  

   

   

Thay vào hệ phương trình cho ta    x y;  1;1 nghiệm hệ

b) Giải phương trình

x

x    x

Phân tích.Để ý 7x 2  5x8x 3, ta đặt 7x  2 a; 5 x b

thì phương trình cho viết lại thành 2  

5

a b a b

a b    a b      

Lời giải Điều kiện xác định phương trình

7 x

   .

Đặt 7x  2 a; 5 x b a 0;b 0 Khi ta có a2b2 8x 3. Như phương trình cho viết lại thành

  2

1

5

5

a b

a b a b

a b a b

a b

  

    

          

  

+ Với ab ta có phương trình x     x x

+ Với a b ta có phương trình 7x  2 5 x hay ta

 

2

2 2

2

7 25 33 10

9 3

1

16 87 71

33 10

x x x x x x x

x x

x

x x

x x x

           

 

    

 

 

      

   

 



Kết hợp với điều kiện xác định ta tập nghiệm phương trình 3;1

S       

Câu 2(2.5 điểm)

a) Tìm tất giá trị m cho tồn cặp số nguyên  x y; thỏa mãn hệ phương

(53)

  2

2

2

mxy m

m x y m

   

    

Lấy hiệu theo vế hai phương trình hệ ta m xy 2x2y2 3m. + Nếu m 0, ta thấy hệ phương trình vơ nghiệm

+ Nếu m 0, từ phương trình ta

   

     

2 2 2

2

3 1

1 1

xy x y y x x

y x x y x x

                     Do  x y; nhận xác giá trị nguyên nên ta xét trường hợp sau

+ Trường hợp Với 12 2

1

x x

y x y

 

    

 

 

 

     

 

 

 

, hệ phương trình khơng có nghiệm ngun

+ Trường hợp Với 12 2 2

1

1

x x x

y

y x y

  

       

  

  

  

         

  

 

 

+ Trường hợp Với 12 2 1

1

1 1

x x x

y

y x y

  

        

  

  

  

        

  

 

 

+ Trường hợp Với 12 2

1

x x

y x y

 

     

 

 

 

      

 

 

 

, hệ phương trình khơng có nghiệm ngun

Vậy phương trình có nghiệm nguyên   x y;   1; , 1;1 , 2; , 2;1      Thay nghiệm vào hệ phương trình cho ta tìm 1;

2

mm  thỏa mãn Vậy 1;2

2

m     

  thỏa mãn yêu cầu toán

b) Với x, y số thực thỏa mãn điều kiện 0  x y 2;2x  y 2xy Tìm giá trị lớn biểu thức P x x2  1 y y2 1.

Từ giả thiết 2x  y 2xy ta 2

x  y Áp dụng bất đẳng thức  

2 2

2

abab

ta có

2

2 2 2

1 1 4

2

2 x y

x y x y x y

   

(54)

Hoàn toàn tương tự ta

2

4 2 4

1 16 1 16

2

2

x y x y x y

 

 

        

 

Do 0  x y nên ta có   2

2

4

1 x y y x x

y y

 

        

 

 

 

Từ kết hợp với 12 42

x  y ta

2

2 2 2 2

2 2

4 4

4 x x 4

y x x y x x x

y y y x

   

                

Hoàn toàn tương tự ta có   4

4

16

1 x y 16 y 16 x x

y y

 

        

 

 

 

Từ kết hợp với 14 164

x  y ta

4

4 4 4 4

4 4

4 16 16

16 x 16 x 16 16 17

y x x y x x x

y y y x

   

                

Do P x x2  1 y y2 1x4 y4 x2 y2 17 5 22 Dấu xẩy x 1;y 2

Vậy giá trị lớn P 22, đạt x 1;y 2

Câu 3(3.0 điểm) Cho tam giác ABC nhọn nội tiếp đường tròn  O với ABAC Phân giác góc BAC cắt BC D cắt đường tròn  O E khác A Gọi M trung điểm

của đoạn thẳng AD Đường thẳng BM cắt đường tròn  O P khác B Giả sử đường

thẳng EP AC cắt N

a) Chứng minh tứ giác APNM nội tiếp N trung điểm đoạn thẳng AC Do AE phân giác góc BAC nên ta

có E điểm cung BC

Từ ta AMP ANP nên tứ giác

AMNP nội tiếp đường trịn Do APM ANM Lại có

 

APMACB nên suy ANM  ACB

Từ dẫn đến MN dong song với BC, mà M trung điểm AD nên suy N

D

E

R Q

P

N M

O

C B

(55)

trung điểm AC

b) Giả sử đường tròn  K ngoại tiếp

tam giác EMN cắt đường thẳng AC Q khác N Chứng minh B Q đối xứng qua AE

Khơng tính tổng qt ta giả sử Q nằm N C(các trương hợp lại chứng minh tương tự)

Do tứ giác EMNQ nội tiếp nên MEQ MNA Mà ta có MNA ACB ACB AEB

nên ta suy AEQ AEB Lại có BAE CAE AE chung nên suy

ABE ACE

  

Do ABAG EBEQ nên AE đường trung trực BQ, suy Q B đối

xứng qua đường thẳng AE

c) Giả sử đường tròn  K cắt đường thẳng BM M Chứng minh RA vng góc RC

Tứ giác ERMN nội tiếp đường trịn nên ta có ENR EMR AMP

Lại có ENC ANP AMP nên ta ERN ENC Ta có REN PMN PAN PEC REN CEN

Kết hợp với cạnh NE chung ta suy REN  CEN

Suy RNNCNA nên R

NAC, điều dẫn đến tam giác RAC vuông R hay ta RA vng góc với RC

Câu 4(1.0 điểm) Số nguyên a gọi số “đẹp” với cách xếp theo thứ tự tùy ý 100 số 1, 2, 3,…, 100 tồn 10 số hạng liên tiếp có tổng lớn a Tìm số “đẹp” lớn

Lời giải Tổng 100 số dãy số 1 100 100 5050

(56)

bằng 505 cách cách chọn ví dụ mà tổng 10số liên tiếp nhỏ 505, số a lớn 505 không thỏa mãn

Thật vậy, xét cách xếp sau 100,1,99,2,98, 3, ,51,50 (chia thành cặp có tổng 101, viết số lớn đứng trước xếp cặp cạnh theo thứ tự giảm dần số lớn hơn) Nếu 10 số liên tiếp gồm cặp số tổng 10 số 505 Nếu không 10 số gồm số đầu nhỏ cặp kết thúc số lớn cặp khác Các số thuộc sáu cặp khác x,101 x, x 1,102    x, , x 4,105 – x 10 số chọn số chọn số 101x đến x 5 (trong dãy ) Dễ thấy tổng 10 số liên tiếp không vượt 505 Vậy a 505

Đề s

Câu (3.5 điểm).

a) Giải hệ phương trình

2

2

4

4 10

x y

x xy x y

   

    



Biến đổi tương đương hệ phương trình ta có

 

      

2

2

2

4 5

4 10 4 2 5 2 1 4 5 2 1

x y x y xy x y xy

x xy x y xy x y x y xy x y

 

  

          

  

  

             

  

  

Đặt a  x ;y b 4xy, hệ phương trình trở thành

 

2 2

2

3

2

5 5

5

1

5 5 5 1

b a b a b a b

a b

a

ab a a a a a a a a

  

    

              

 

    

    

              

    

   

Từ ta có hệ phương trình 1; 11

1 2;

2

x y

x y

xy x y

   

   

  

 

     

 

Vậy hệ phương trình cho có nghiệm   ; 1;1 2;

x y        b) giải phương trình

2

64

5

5 6

x x

x x

x x

  

 

(57)

   3

2

5x 6x 5 5x 6x  5  4x 4x

Đặt a  5x2 6x 5;b 4x a 0 Khi phương trình viết lại thành

 1   2 1 0

a a  b  b a b a abb    a b

Từ ta 5x2 6x  5 4x, phương trình tương đương với

2 2

0

1

5 16 11

x x

x

x x x x x

 

   

 

   

 

       

 

 

 

Vậy phương trình cho có nghiệm x 1

Câu 2(2.5 điểm).

a) Với x, y số nguyên thỏa mãn đẳng thức 2

2

x  y  Chứng minh

2 40

xy

Biến đổi giả thiết 2 3 1 2 1 3 2 1

2

x   y   x   y   xy

Vì số phương chia dư hoặc 4, mà 3x2 2y2 1 nên x2 y2 chia cho có số dư 1, từ ta x2 y25

Vì số phương chia dư hoặc 4, mà 3x22y2 1 nên x2 y2 chia cho có số dư 1, từ ta x2 y28

Do nguyên tố nên ta x2 y240

b) Tìm tất cặp số nguyên  x y; thỏa mãn đẳng thức sau x4 2x2 y3. Ta có x4 2x2 y3  x4 2x2  1 y3  1 x2 12 y1 y – y 1. Gọi dy1; y2 y 1 Khi ta có y12d y2  y 1d nên ta

 2  2 

1

y  y  ydy d Do d nguyên tố nên ta có hai trường hợp

+ Khi 3d ta x2 12 y1 y – y 1 9 nên x2 129x2 1 3 Điều vơ lý số phương chia cho khơng thể có số dư

(58)

Do y1; y – y2  1 1

Khi y1 y – y 1là số phương nên ta đặt y 1 a ; y – y2  1 b2 a, b số nguyên dương  a b; 1 Tứ ta

  2    

2 2 2

b  a 1 – a 1  4b 4a 12a 12  2b – 2a 3 2b2a 3 

Vì  2b 2a2 32 2b2a2 3 nên ta xét trường hợp sau + Trường hợp Với

2

2

1

2

2

2 3

b

b a

a

b a

 

     

 

 

 

     

 



, hệ khơng có nghiệm ngun

+ Trường hợp Với

2

2

2

1 y 1

2 3

0

1 y – y 1

2

b

b a a

x y

b a

b a

   

          

   

      

   

              



Thử lại vào phương trình ban đầu ta thấy thỏa mãn Vậy phương trình có nghiệm  0;0

Câu (3.0 điểm) Cho hình vng ABCD nội tiếp đường trịn  O P điểm thuộc cung

nhỏ AD đường tròn  O P khác A, D Các đường thẳng PB, PC cắt AD

AD M, N Đường trung trực AM cắt đường thẳng AC, PB E, K Đường trung trực DN cắt đường thẳng BD, PC F, L

(59)

a) Chứng minh ba điểm K, O, L thẳng hàng

Ta có KAKM suy tam giác AKM cân Do ta KAM KMA,

Mà ta lại có KMA KBA 900và KABKAM 900 nên suy KAB KBA hay tam giác AKB cân ta K Do ta KAKBKM

Lại có OBOD nên OK đường trung bình tam giác DKM, suy OK // MD

Chứng minh tương tự ta có OL đường trung bình tam giác NCA, suy OL // AD Theo tiên đề Ơclit ba điểm K, O, L thẳng hàng

b) Chứng minh đường thẳng PO qua trung điểm EF

Ta có E thuộc đường trung trực AM EAM 450 nên tam giác EAM vuông cân.

Do suy ME vng góc với AC Hồn tồn tương tự ta có NF vng góc với BD Ta có MN song song với BC nên theo định lí Talet ta có PB PC

MBNC

Hạ PX vng góc với AC PY vng góc với BD, ta có PX, EM, BO song song với

Do ta XO PB

(60)

Lại có PY, FN, CO song song với nên ta có YO PC

FONC

Từ dẫn đến XO YO

EOFO nên suy XY EF song song với

Ta có PXO PYO XOY 900 nên tứ giác PXOY hình chữ nhật, PO qua trung điểm XY Do XY song song với EF nên PO qua trung điểm EF

c) Giả sử đường thảng EK cắt đường thẳng FL AC cắt T Đường thẳng ST cắt đường thẳng PB, PC U V.Chứng minh bốn điểm K, L, V, U thuộc đương trịn

Ta có LK song song với AD nên LK vng góc với ES Do KOA OAD 450 nên

 450

KEO

Mà ta có EOS 900 nên OK phân giác góc EOS.

Suy tam giác EOS cân nên ta có KSKE, suy KL đường trung trực ES hay E S đối xứng với qua KL Hoàn toàn tương tự ta có F T đối xứng qua KL

Từ ta EOF SOT nên EFOSTO

Gọi giao điểm OP EF I, ta có I trung điểm EF Do tam giác OEF cân nên ta có IOIEIE

Suy tam giác IOF cân nên IOF IFOOTS

IOEIOF EOF900 nên IOE OTS 900.

Gọi giao điểm OP ST H nên ta cóTOH IOE , suy TOH HTO 900. Từ suy THO 900 hay PO vng góc với ST.

Ta có PLF PCDPCD PBD BPO nên PLF PBO PVH Lại có PH vng góc với UV nên VPHHVP 900.

Mà ta lại có PLFPLK PLK 900 nên PLK HVP.

(61)

Câu 4(1.0 điểm) Chứng minh với số tự nhiên n 3luôn tồn cách xếp n số

1,2, 3, ,n thành số x x x1, , , ,2 3 xn cho

2

i k j

x x

x   với số i j k; ;  mà 1   i j k n

Lời giải

Dãy a a a1, , , ,2 3 as chiều dài s 3 tùy ý gọi dãy “tốt”

2

j k j

a a

a   với số (i, j, k) thỏa mãn (1   i j k s).

Nếu dãya a a1, , , ,2 3 as dãy tốt dãy , , , , 2a a1 2 a3 asvà dãy

1

2a 1, 2a 1, 2a 1, , 2as 1 dãy tốt

Từ nhận xét ta suy dãyx1, x , , ,2 x3 xs dãy tốt số 1, 2, 3, , s (s 3) dãy , , , , , 2x1 x2 x3 xs x1 1, 2x2 1, , 2xs 1là dãy tốt số 1, 2, 3, , 2s(chú ý rằng2

2

k m

xx

không số nguyên) +) (1, 3, 2) dãy tốt số 1, 2,

+) Với n 3luôn tồn k để 3.2k1  n 3.2 k Theo nhận xét trên, ta xây dựng dãy tốt từ dãy tốt từ số 1, 2, 3, , 3.2k sau ta bỏ số n1,n2,n 3, , 3.2k nhận dãy tốt từ số 1, 2, 3, , n (trên dãy tốt ta bỏ số hạng dãy cịn lại dãy tốt)

Cách khác:

Với n 3 ta có cách xếp 1, 3,

Ta chứng minh toán với n với 2n

Thật vậy, giả sử ta có cách xắp xếp với n cách xếp có dạng 1, , , , ,2 n

x x x x x thỏa mãn với 1   i j k n ta có

2

i k j

x x

x   Ta chứng minh

tồn dãy 2n thỏa mãn đề

Xét dãy sau 2x ,2x ,2x , ,2x , 2x1 2 3 n 11,2x21,2x31, ,2xn1, dãy số gồm tất số từ đến 2n Xét ab thuộc dãy

+ Nếu a, b khác tính chẵn lẻ

2

(62)

+ Nếu a, b tính chẵn lẻ

2

ab thuộc dãy

Nếu a, b chẵn (trường hợp a, b lẻ chứng minh tương tự ) Trường hợp Khi

2

ab lẻ

2

ab không thể nằm a, b cách xây dựng dãy (thỏa

mãn)

Trường hợp Khi

2

ab chẵn

Giả sử

2

ab nằm a, b dãy

i k

a 2x , b 2x ,

2 j

ab x

   với

i j k suy dãy ban đầu x ,x ,x , , x1 2 3 n cách xếp không thỏa mãn đề (mâu thuẫn )

Vậy điều giả sử sai nên

2

ab không nằm a b dãy.

Vậy với trường hợp trung bình cộng a, b nằm a, b suy xây dựng cách xếp thỏa mãn cho trường hợp 2n Như chứng minh tốn với n với 2n

Mặt khác tốn với n với n1 Nên theo ngun lí quy nạp ta có điều

phải chứng minh

Đề s

Câu

1) Giả sử hai số thực phân biệt thỏa mãn

a)

b) Với

Vậy

;

a b

2

3

a b b a    

  

  

2 3 0

a b a b

    

a b a b  3 a b

       a b a b   3

0 ( )

3

a b l a b    

   

a b    a b3 27

 

3 3 27 3 9 27

a b ab a b a b ab

          

 2  

2 3 3 4 2 3 4 2

a a b b a b ab a b ab

             

3 45

(63)

Nhận xét: Đây toán bản, đề u cầu ta làm đó, với vài phép biến đổi tương đương kết hợp đẳng thức quen thuộc ta suy điều phải chứng minh

Nhắc lại kiến thức phương pháp:

• Hằng đẳng thức

• Hệ phương trình đối xứng dạng Lấy (1) + (2), ta có:

Lấy (1) – (2), ta có: Từ suy ra:

Bài tập kết thúc Bài tập tương tự:

1. Cho thỏa mãn Tính

2. Cho thỏa mãn Tính giá trị biểu thức

2) Ta thấy nghiệm phương trình

Nếu , nhân hai vế phương trình với , ta

Nhận xét: Bài toán sử dụng phương pháp tích từ phương trình xuống phương trình hai hệ, đưa phương trình đẳng cấp bậc hai để tìm mối liên hệ hai biến ngược lại phươngtrình một, tìm nghiệm hệ phương trình

Nhắc lại kiến thức phương pháp:

• Phương trình đẳng cấp bậc hai ẩn (*) Làm nháp: Chia phương trình (*) cho ta có:

 2 2 2

a b aab b

 3 3 2 2 3 3

a b aa babb

2

(1) (2)

ma nb p

mb na p

  



  



 2   2

m abn a b  p

 2   a b

m a b n a b n a b

m    

   

   

 2  

2

2p m a b 2ab n a b m n 2ab n.n ab

m m

 

   

           

;

a b a24b3; b24a3 a5b5 ;

a b a22b24b7; b22a24a7

3

abab

0

x y 

0

yy

2

2 2 2

2 5

4

4

x y xy xy y xy

x y xy x y xy

 

     

 

 

     

 

2 2 2

2 5

2

x y xy x y xy x xy y x y xy

 

     

 

 

    

 

 

     

2 5

2

x y xy x y xy x y x y x y x y

 

     

 

 

     

 

 

 

 

2

1

2 2 4

,

2 5

x y xy

x y x y

x y xy

x y x y

  

   

   

  

   

   

   

,

x y ax2bxy cy 20

(64)

Coi phương trình bậc hai ẩn , giả sử tìm , ta phân tích

Ý tưởng: Dạng hệ sử dụng phương pháp đặc trưng khó tìm đại lượng biểu diễn nhân tử chung hai biến Nhìn vào vế trái phương trình, với phương trình bậc nhất, cịn phương trình hai bậc hai, tương tự vế phải, bậc phương trình hai lớn bậc so với phương trình Điều làm ta nghĩ đến phép nhân để đồng bậc hóa chúng, nhân nhân chéo phương trình Tuy nhiên quan sát kỹ chút, hai phương trình xuất nên ta xuống phương trình hai (thực chất việc nhân chéo ta giảm biến , hệ phương trình cho trở thành:

Việc cịn lại tìm nghiệm hệ phương trình cho

Bài toán kết thúc Bài tập tương tự:

1. Giải hệ phương trình

Đáp số:

2. Giải hệ phương trình

Đáp số:

Câu

1) Ta có , suy

Mà , suy ,

nên

ta có , suy , nên

Nhận xét: Bài tốn tìm số ngun thỏa mãn điều kiện, sử dụng tính chất chia hết tổng, hiệu, tích,…

Nhắc lại kiến thức phương pháp:

• Biến đổi đại số

Ta có nên:

• Một số ln chia hết cho

2

0

x x a b c

y y                  

 

   

x

y x myx n

y

  

2

(*) ax bxy cy x my x ny x my

x ny   

          

;

x y

5xy 5xy2x3y

y

 

2 2

2

2 5

4

2

x y xy x y xy x y xy

x y x y x y y x xy y

x y    

 

      

   

  

        

  

   

 

2 2

3 5

x y xy x y xy    

  



 ;   0; , 2; , 1;

5

x y       

   

2 2

3

9

x y xy x y xy

  



  



 ;   0; , 1; , 5;

3

x y       

   

  

1 1

xy  xyxy1xy  1 x y

      

1 1 1

xy  x y xy       x y x yxyx1y1 y1x1 x y

 2 1 1

x   xx1x1 2x1 x2 x3

(x−1)(y− =1) xy x y− − +1

(65)

Ta có

• Hai số chia hết cho số hiệu hai số

• Tính chất chia hết với số ngun

• Một tổng hai số hạng chia hết cho số có số hạng chia hết cho số số hạng cịn lại chia hết

; hồn tồn tương tự ta có

• Có hai số thỏa mãn: Số thứ chia số thứ hai số thứ hai chia hết cho số thứ hai số bằngnhau

Ta có

Thay lại vào đề ta có

• Số thứ chia hết cho số thứ hai số thứ hai ước số thứ

vì suy

Vậy

2) Ta có

Ta có

Phương trình có nghiệm

Vậy

Nhận xét: Bài tốn tìm giá trị biểu thức sau biến đổi biểu thức trở thành tham số phương trình bậc hai theo ẩn

Nhắc lại kiến thức phương pháp:

(xy x y− − +1) ( xy x y− − +1)

( ) ( )

( 1) ( 1) ( 2) ( 1)

xy xy x y

x y xy x y xy x y xy x y

 − − − +

 ⇒ + − − − +

− − + − − +



 

(x 1) (y 1) (xy x y 1) (x 1) (y 1) (x 1)(y 1)

⇔ − + −  − − + ⇔ − + −  − −

a bka ba b k; ;

( ) ( ) ( )( ) ( ) (2 )( ) ( )( )

1 1 1 1 1

x− + y−  xy− ⇒ x− + xy−  xy

( ) ( )( ) ( )( )

( )( ) ( )( )

2

1 1 1

1 1

x x y x y x y x y

 − + − − − −

 

− − − −



         

2

1 1 1

x x y x y

        

(y−1) ( x−1)

       

1

1

1

x y

x y x y y x

  

      

  



 

( 2 ) ( )2 ( )( ) ( )2

1 1 1

x −  x− ⇔ xx+  x

(x 1) (x 1) (x 2)  (x 1) (x 1)

⇔ +  − ⇔ − +  − ⇒  −

( ) ( ) ( )2 { } 2 x− ⇒1 x− ∈1 U = ± ±1;

2 1

x≥ ⇔ − ≥x (x− ∈1) { }1;

1 2

1 3

x x y x x y

 − =  = ⇒ =

⇔ − = ⇔ = ⇒ =

 

2

x y= = x y= =3

2 2 2 1 0 2 2 1

2

x y x yy   y x y   y  

 2   2 

3

xy xy P

x y x y

 

    

2

3Px y 2xy P

   

2

4 12P  

0

 

2 1

4 12

3 3

P PP P

           

3 12

3 21

MaxP= ⇔ = −x

6

(66)

• Biến đổi giả thiết

Ta có

• Đưa biểu thức cho ẩn biểu thức cố định

Thay vào biểu thức ta có

Đặt ta

• Tìm giá trị lớn nhất, nhỏ biểu thức cách đưa biểu thức trở thành tham số phương trình bậc hai

Khi ta có

• Phương trình bậc hai có nghiệm khơng âm Ta có Vì phương trình có nghiệm nên

• Khi xét đến giá trị nhỏ nhất, giá trị lớn biểu thức cần phải xét xem “dấu xảy nào?”

Ta có dấu xảy vào Phương trình bậc hai có nghiệm kép , thay vào biểu thức ban đầu ta có:

Vậy

Câu

2 2 2 1 0 2 2 1

2

x y x yy   y x y   y  

2 1

x y

y=− − P 2 2

1

3

2

xy P

x y

=

− −

+

( 2 ) 2

2

3

3

xy xy x y x y

= =

− −

− − +

xy a= 22

3

a P

a

=

− −

2

2 3 2 0

3

a

P Pa a P a

= ⇒ − + =

− −

2

1 3P

∆ = −

∆ ∆'

2

0 3P

∆ ≥ ⇔ − ≥

(1 3P)(1 3P) ( 3P 1)( 3P 0)

⇔ − + ≥ ⇔ − + ≥

1 1

1

3 3

PP P

⇔ − ≤ ≤ ⇔ ≤ ≤ ⇒ ≤

1 ' 0

3

P= ⇔ ∆ = ⇒

2

3

a=

2

2 2 1 0 2 7 12

7

3

3

6

xy y y y x

y  

               

   

  

3 12

3 21

MaxP= ⇔ = −x

6

(67)

1) Ta có phân giác mà tam giác cân,

Nhận xét Bài toán chứng minh hai đường thẳng song song sử dụng định lý Ta-lét đảo Nhắc lại kiến thức phương pháp

• Tâm đường trịn nội tiếp tam giác giao điểm ba đường phân giác

Trong có tâm đường trịn nội tiếp nên suy phân giác

• Tính chất đường phân giác tam giác Trong có phân giác nên

• Hai điểm đối xứng qua đường thẳng đường thẳng trung trực đoạn thẳng nối hai điểm

+ điểm đối xứng qua nên trung trực ; + điểm đối xứng qua nên trung trực

• Một điểm thuộc trung trực đoạn thẳng điểm cách hai đầu mút đoạn thẳng

+ mà trung trực nên ; + mà trung trực nên ; suy

• Định lý Ta-lét đảo: Hai điểm thuộc hai đoạn thẳng, chia hai đoạn thành đoạn thẳng tương ứng tỷ lệthì hai đường thẳng chứa hai đoạn thẳng song song

Theo định lý Ta-lét đảo ta có

2) Ta có

Tương tự:

Mà nội tiếp

AD BD AB

DC AC

  BED CDF;

BE AB BC FE

CF AC

   

ABC

I AI AD

BAC

ABC

AD DB AB

DC AC=

E D BI BI DE F D CI CI DF

B BIBI DE BD BE=

C CICI DF CD CF=

BD BE CD CF=

BE AB BE CF

CF AC= ⇔ BA CA=

BE CF

BA CA= EF BC

  

BC EF EFD EDB BED 

 1800    

APM AEM BED APM DEF

      

DFE APN APN APM DFE FED MPN   

     1800

(68)

Nhận xét: Bài toán chứng minh tứ giác nội tiếp cách chứng minh tổng hai góc

Nhắc lại kiến thức phương pháp:

• Hai đường thẳng song song tạo góc vị trí so le Ta có , suy

• Tam giác có hai cạnh tam giác cân, tam giác cân có hai góc kề cạnh đáy

Tam giác có nên cân suy , suy

• Tứ giác nội tiếp có tổng hai góc đối diện

Tứ giác tứ giác nội tiếp nên

Hồn tồn tương tựcó

• Trung điểm; Đường trung bình tam giác

+ đường trung bình nên ; + trung điểm nên ;

suy

• Tứgiác có cặp cạnh song song hình bình hành

Tứ giác có nên hình bình hành

• Hình bình hành có hai góc đối diện + hình bình hành nên ;

+ (hai góc so le suy tính chất đường trung bình);

suy

• Tứgiác có tổng hai góc đối diện tứ giác nội tiếp

Tứ giác có nên tứ giác tứ giác nội tiếp

3) Ta có , suy điểm thẳng hàng

Nhận xét: Bài toán chứng minh ba điểm thẳng hàng cách chứng minh ba điểm thuộc đường thẳng đặc biệt

Nhắc lại kiến thứcvà phương pháp:

• Tứ giác nội tiếp có góc ngồi góc khơng kề với

Tứ giác tứ giác nội tiếp nên mà ( ), suy

• Trung điểm, Đường trung bình, Hình bình hành,…(đã nhắc lại phần trên)

180°

EF BCFED EDB =

BDE

BD BE= ∆BDE B BDE BED =

   FED EDB BED= =

180°

APME  APM AEM+ =1800

 1800    

APM AEM BED APM DEF

⇔ = − = ⇒ =

      

DFE APN APN APM DFE FED MPN    MJDFE 1

2

MJ DF MJ DF

  

= 

N DF

2

DF DN=

( )

( )

MJ DN DF MJ DN DF

 = =

 

MJND MJ DNMJ DN= MJND

MJNDMJN MDN

 

MDN EDFMN FE

  1800

MJN MPN 

180°

MPNJ  MJN MPN 1800 MPNJ

 

APM DEF  JPM JNM JEM  JPM APMA P J; ;

MPAE  MPA BED= BED DEF = =BDE

(69)

+ đường trung bình nên ;

+ trung điểm nên ; suy nên hình bình hành nên mà (hai góc nội tiếp chắn cung đường tròn ngoại tiếp tứ giác ) suy , suy nên

hay ba điểm thẳng hàng

Câu

1) Theo đề bài, số nguyên dương xếp theo hàng chéo bảng: Hàng chéo thứ có số, hàng chéo thứ hai có số,……

Giả sử số nằm hàng chéo thứ ta có:

Áp dụng ta có

Số hàng chéo thứ

Như số 2015 nằm vị trí thứ hàng chéo thứ (vị trí áp chót) Tọa độ

2) Theo Cơsi cho số ta có

BĐT tương đương (1)

Đặt ( )

Áp dụng bất đẳng thức Schur bậc 3:

với số thực không âm

Chứng minh bất đẳng thức

Do vai trò nhau, giả sử Ta xét

(điều phải chứng minh)

Ta có

MNDEF 1

2

MN FE MN FE

  

= 

J FE

2

EJ= EF MN JEJE MN= MNJE  

MEJ MNI= MEJ MNP = MJ MNPJJPM DEF  JPM APM=

PJ PAA P J; ;

x k

 1  1 1 1 8 1 1 8

2 2

k k k k x x

x k

      

    

1

2

x k     

2015

x 1 8.2015 63

2

k     63

k  1 1954

2

k k

 

2015 1954 62   63

2,62

3

4 3 3 2

4abc ab bc ac   4 a b c  1 abc   a b c abc3 a b c

 

3

2 2 3 2 2

ab  c a b cab bc ac 

3a2 x b; y c; z x y z; ; 0

3 3 3 2 3 2 3 2 3

x y z xyz x y z x z y

      

     

3 3 3

xyzxyz xy x y  yz y z xz x z

        

x x y x z y y x y z z z x z y

         

; ;

x y z

; ;

x y z x y z  z z x z y    

    2    0

x x z y y z xxz yz y   x y x y z  

           

x x z x y y y z x y x x z x y y y z y x

             

        

x x y x z y y x y z z z x z y

         

     

3 3 3

(70)

Dấu “=” xảy

Nhận xét: Bài toán sử dụng bất đẳng thức Cosi, phép đặt ẩn phụ đồng thời chứng minh qua bất đẳng thức trung gian bất đẳng thức bổ đề (bất đẳng thức Schur bậc ba) để suy điều phải chứng minh

Nhắclại kiến thức phương pháp:

• Bất đẳng thức Cosi cho ba số thực dương

• Bất đẳng thức Schur bậc ba cho số thực dương

Bài toán kết thúc

Bài tập tương tự:

1. Chứng minh với , ta ln có:

2. Cho số thực dương thỏa mãn Chứng minh

ĐỀ SỐ 10

Câu

1) Đặt Ta có

Nhận xét: Bài tốn sử dụng phương pháp đặt ẩn phụ, khai thác biến đổi từ giả thiết, điểm nhấn đẳng thức bậc ba để suy điều phải chứng minh

Nhắc lại kiến thức phương pháp:

• Hằng đẳng thức

• Hằng đẳng thức

3 3 3

2 x y z x z y

  

1

;

x y z

a b c x y z

  

    

  

3

a b c   abc

     

3 3 3

xyzxyz xy x y  yz y z xz x z

; ;

a b c

 2 2  

2 abcabc 8 a b c 

; ;

a b ca b c  3

1 1 12

5

a b c abc

 

    

 

 

 

3 3

a b c x b c a y c a b z         

    

 3   3  3 3 3a3b3c 24 3a b c   3b c a   3c a b 

 3 3 3 3 24

x y z x y z

      

 3  3    

24

x y z x y z x y y z x z

          

   

24 x y y z x z

      24 2  a4 2b b ac c  2 4a0

   

24 24 a 2b b 2c c 2a

       a 2b b 2c c 2a1

 3 3 2 2 3 3

x y xx yxyy

  3 3  2   2 3

3

x y z   x y  x y z  x y z z

 2  

3 3 3 3 3

x x y xy y x y z x y z z

        

   2  

3 3 3 3 3

x y z xy x y x y z x y z

        

  

3 3 3

x y z x y xy xz yz z

(71)

Ý tưởng: Bài toán cho giả thiết cồng kềnh, phức tạp chí bậc to, quan sát đại lượng bên vế phải, ta thấy biểu thức ,

độc lập so với mặt khác:

Vì đặt giả thiết cho tương đương với Khoan nói đến chuyện biến đổi nói, ta quan sát đến biểu thức cần chứng minh, với phép ẩn phụ ta hồn tồn rút theo Xét biểu thức biểu thức không chứa nên ta cần tìm mối liên hệ cho triệt tiêu, dễ thấy chứa chứa nên suy , tương tự ; Do ta cần chứng minh Bây ta quay ngược lại với giả thiết, xuất đẳng thức bậc ba, nên ta khai triển nó, ta được:

nên suy ra:

điều phải chứng minh Bài toán kết thúc

Bài tập tương tự:

1. Cho ba số thực dương thỏa mãn điều kiện

Chứng minh 2) Ta có

Vậy

Nhận xét: Bài hệphương trình hay chỗ kết hợp cảhai phương trình, sau nhóm lại đẳng thức đưa vềphương trình biểu diễn mối liên hệ thếngược lại tìm nghiệm hệphương trình

Nhắc lại kiến thức phương pháp:

• Hằng đẳng thức

   

3 3 3

x y z x y y z z x

      

3a b c  3b c a  3c a b 

3a b c  3b c a  3c a b  3a3b3c

3 ; ;

xa b c y   b c a z   c a b 

 3 3 3 3 24

x y z   xyz

; ;

a b c

; ;

x y z a2b c

; ;

x y z c xc y c

 

2 2

x y  abab y z 2b2cz x 2c2a

x y y z z x     

 3 3 3 3    

3

x y z  xyzx y y z z x  

   

3 3 3 24 3

xyzx y y z z x    xyz

x y y z z x  

     

; ;

a b c

 3   3  3 3

12

a b c     a b c   b c a   a c b 2abc1

  3

2

27 26 27

x y xy

x y y x x x

   



      



  

  3

2

27 26 27

x y

x y y x x x

   

  

     



   

3 7 3 2 2 27 27 9

y x x y x y x x x

         

      2 3

3 8 3 12 6 3 1

y x xy x y x y x y x

          

  3 3

2 3

x y x x y x

         

1

y x

    2 1 7

8

2

x y x x

x y     

    

    

 ;   1; , 7;

2

x y    

 

 

;

x y

 3 3 2 2 3 3

(72)

• Hằng đẳng thức

• Giải phương trình tổng quát

Ý tưởng: Phương trình thứhai hệ phức tạp, ta sẽđi khai thác trước, đểý bên vế phải phương trình hai, xuất tổng mà ta thấy

có bóng dáng đẳng thức thêm đại lượng :

Chuyển đại lượng sang vếtrái phương trình hai, ta được: ( )

Điều lại ta chưa khai thác tới phương trình một, quan sát ( ) chút, ta thấy vế phải lũy thừa bậc ba, ta biến đổi vế trái ( ) dạng Bởi lẽ

ta sẽnghĩ đến chuyện đưa vếtrái ( ) dạng Hay nói cách khác, kết hợp phương trình hệ, ta cần chứng minh:

Sẽcó hai cách để chứng minh, xi, hai chứng minh ngược:

• Từ phương trình hệ, vào vế trái ( ), ta được:

• Ta có

Khi ta có , ngược lại phương trình hệ ta sẽtìm nghiệm hệphương trình ban đầu

Bài toán kết thúc Bài tập tương tự:

1. Giải hệphương trình

Câu

1) Đặt ( )

vì ( )

Lại có nên

  3 3  2   2 3

3

x y z   x y  x y z  x y z z

   

     

  

   

2

3 2 3

2

3 3

3 3

3 3

3 3

3 3

3

x x y xy y x y z x y z z x y z xy x y x y z x y z x y z x y xy xz yz z

x y z x y y z z x

        

        

       

      

 3  3

f x g x    

   

   

    2      2  0    

f x g x f xf x g x g xf x g x

 

         

2

27x 9x

 2  

2

27x 9x3 3x 3 3x  3x 313

 3

3

26x 27x 9x 3x1 x 1 1

x

   3

3 8 27 3 1

xy   x y  xi

i i

3 8 3 23

xy  xyi  3

2

x y 

   3

3 8 27 2

xy   x y  x y 

x2y 2 i

     3

3 23 3 2 2 2

xy   x y x  y  x y 

 3 3 3 3    

2 2

x y  xy   x y x  y

     

3 x y x y 27 x y 2x 2y xy

         

  3 3

2 3

x y   x    x y x  y x

  3

2

27 7 12

x y xy

x y y x x x

   



      



2

5 30

n x n y    

  

 x y, ?; ,x y0

  

2 25 1.25

y x y x y x

       x y, ?; ,x y0

y x y x   13

25 12

y x y y x x

 

    

 

 

    

 

(73)

Thay vào ta tính thảo mãn

Nhận xét: Bài tốn số học sử dụng tính chất số phương, phương trình ước số Nhắc lại kiến thức phương pháp:

• Sốchính phương viết dạng với sốngun + sốchính phương nên có dạng với sốngun + sốchính phương nên có dạng với số nguyên

• Hằng đẳng thức

Trừ vế theo vế cho ta

• Phương trình ước số

;

suy

(do số tựnhiên) 2) Ta thấy sốchính phương Đặt 139 na a

n+ x2 x

30

n+ y2 y

( )( )

2

A B− = A B A B+ −

2 30 n x n y         ( ) ( ) ( )( )

2 30 5 25

yx = n+ − n+ ⇔ y x y x− + =

(y x y x− )( + )=25 1.25= = −( ) ( )1 25− =5.5= −( ) ( )5 5−

1 26

25

25 26

1 25

5 10

5

5 10

5

1 26

25

25 26

1 25

y x y y x x y y x y y x x y y x y y x x y y x y y x x y y x y y x x y y x y y x x y

 − =  =  + =  = −      − =  =   + = = −      − =  =    + =  = −  ⇔  − = −  = −  + = −  = +     − = −  = −  + = −  = +     − = −  = −    + = −  = +  

13 30 169 12 144 13 30 169

12 144 30 25

0

5 30 25

0

13 30 169 12 144 13 30 169 12 144

y n x n y n x n y n x n y n x n y n x n y n x n  =  + =  =  + =     =  + =  = −  + =     =  + =      =  + =  ⇔ ⇔   = −  + =   =  + =       = −  + =    = − + =       = − + =     = + =                      

30 169 139

5 144 139 139

139 139

30 25

5

n n

n n n

n n n n n n  + =  =  + =  =  =     ⇔ ⇔ ⇒ = ⇒ =  + =  = −    + = = −     n

1 x y  3 xy x y,  ? x y,

3; ; ?

x y x y

   

 

; ; , , ?

x a y b x y    c a b c

2

2 2

1

1

3

a b c

a b c x y a b

c a b x y c

                         

 2 2 2

1

a b a b

      2a2b2ab 3

a 1b 1

(74)

Nhận xét: Bài toán số học sử dụng tính chất số phương, phương trình ước số Nhắc lại kiến thức phương pháp:

• Khi số ngun ta có số phương

Ta có số nguyên nên ta biển diễn sau với số nguyên dương

Từ ta có

• Hằng đẳng thức

• Phương trình ước số

Vì nên

Ta có

Vậy

3) Ta có

Dấu = xảy

Nhận xét: Bài toán sử dụng kỹ thuật chọn điểm rơi, kết hợp với bất đẳng thức Cosi đưa bất đẳng thức quen thuộc gọi bất đẳng thức Nesbitt, bất đẳng thức có tới 45 cách chứng minh từ tìm giá trị nhỏ biểu thức cho

Nhắc lại kiến thức phương pháp:

• Bất đẳng thức Cosi cho hai số thực dương

a a

; ;

x y x y+ +

2

2

3

x a x a y b y b

x y c x y c

 = ⇒ =

 = ⇒ =

 + + = ⇒ + + =



, ,

a b c

 2

2 2

2 2

1

1

1

3

a b c

a b c

x y a b a b a b c a b

x y c    

    

          

 

    

 

    

( )2 2 2 2

2 2

x y z+ − =x +y +z + xyxzyz

( )2 2 2 2 2 2 2

1 2

a b+ − −a b− = ⇔a +b + + abab a b− − =

( ) ( )

2ab 2a 2b ab a b a b b

⇔ − − = ⇔ − − = ⇔ − − − =

(a 1)(b 2)

⇔ − − =

(a−1)(b− = =1 2.1 1.2) = = −( ) ( ) ( ) ( )2 1− = −1 2−

;

a b≥ (a−1 ;) (b− ≥ −1)

1

1

1

1

a a x b b y a a x b b y

 − =  =  =

 − =  =  =

  

 ⇔ ⇒

 − =  =  =

  

− = = =

  

  

(x y; ) (∈ 9; , 4; 9) ( )

4 4

y

x z

P

y z z x x y

  

     

4

4

4 4 4

y

x z

P

y z z x x y

   

     

4

4

4 4 4

y

x z

y z x z x y

  

        

4 x y z

y z x z x y

 

 

     

 

 

4

9

x x y z

y         



2

(75)

• Bất đẳng thức Cosi cho ba số thực dương

• Bất đẳng thức Nesbitt cho ba số thực dương Chứng minh: Bất đẳng thức cho tương đương với:

Áp dụng bất đẳng thức Cosi cho basố thực dương, ta có:

Nhân hai bất đẳng thức với suy điều phải chứng minh

Ý tưởng: Bài tốn bất đẳng thức đối xứng, vai trị biến nhau, khơng khó để thấy , dấu đẳng thức xảy Với giá trị

thay ngược lại , ta có:

(*)

Để bất phương trình (*) có nghiệm với ta

Do ta tìm dấu đẳng thức xảy Bây giờ, quan sát biểu thức , chứa ba phân thức đồng thời thức xuất mẫu số phân thức, cách ta đánh giá khử bậc hai Với điểm rơi tìm , ta thấy , khử bậc hai ta áp dụng bất đẳng thức Cosi sau:

Tương tự cho biểu thức lại, ta suy ra:

(*)

Và chứng minh bất đẳng thức (*) tốn hồn thành Nhận thấy lấy tử cộng mẫu phân thức (*) ta đại lượng chung, thế, phân thức ta cộng thêm đó, ta có (*)

3

a b c   abc

; ;

a b c

3

2

a b c b c c a a b     

9

1 1

2

a b c b c c a a b

     

        

     

        

     

9

a b c b c a c a b b c c a a b

     

   

  

a b c 1 92

a b b c c a

 

      

  

 

a b b c c a 1

a b b c c a

 

 

            

 

         

   

3

3

1 1

a b b c c a a b b c c a a b b c c a a b b c c a

        

  

     

; ;

x y z

P mx y z k  

x y z k   P

2 2

3 9 2 4 0

2

k

P m k m k m k

     

,

m k  (*)

 

4 36 0 2 36 0 36 6

mm  m m   m   m P6

4

k    x y z P

4

x y z   y z  4

 4 4. 4 4 4

y z y z    y z   y z   y z   

3

4

2

y y

x z x z P

y z z x x y y z z x x y

 

 

         

    

 

x y z 

1

2

x y z x y z x y z y z z x x y

     

   

  

x y z 1 92

x y y z z x

 

 

      

   

(76)

Đặt ta có:

(*) ln (điều phải chứng minh) Bài tốn kết thúc

Bài tập tương tự:

1. Cho số thực dương lớn Tìm giá trị nhỏ biểu thức

2. Cho số thực dương lớn Tìm giá trị nhỏ biểu thức

Câu

1) Gọi điểm đối xứng qua trung điểm

Mà , suy tam giác cân

Mặc khác lại có trung điểm Do tứ giác hình bình hành, suy

ra

Nhận xét: Chứng minh hai đoạn thẳng ta chứng minh cho chúng đoạn thẳng thứ ba

Nhắc lại kiến thức phương pháp:

• Hai điểm đối xứng qua điểm điểm trung điểm đoạn thẳng nối hai điểm cho

x y y z z x 1

x y y z z x

 

  

          

 

 

; ;

a x y b y z c z x     

a b c 1 33abc.33 9

a b c abc

 

       

 

; ;

x y z

2 2

y

x z

P

y z z x x y

  

     

; ;

x y z

2

6 6

y

x z

P

y z z x x y

  

     

P A M

2

HP HM MB HM AH HN

     

H

NP

BH NP PNB B BN BP 

M BC AP; ACPB

(77)

là điểm đối xứng với qua nên trung điểm ta có trung điểm

• Tam giác có đường trung tuyến đường cao tam giác tam giác cân Tam giác có vừa đường trung tuyến trung điểm ) vừa đường cao ( ) nên cân

• Tam giác cân có hai cạnh bên Tam giác cân nên

• Tứ giác có hai đường chéo cắt trung điểm đường hình bình hành Tứ giác có vừa trung điểm vừa trung điểm nên hình bình hành

• Hình bình hành có cạnh đối diện

là hình bình hành nên , suy (điều phải chứng minh)

2) Do tứ giác hình bình hành, suy Mà tam giác cân

Ta có

cùng thuộc đường tròn giao điểm với , suy

Nhận xét: Chứng minh hai đoạn thẳng ta chứng minh cho chúng đoạn thẳng thứ ba

Nhắc lại kiến thức phương pháp:

• Hình bình hành có hai cạnh đối diện song song

là hình bình hành nên suy (hai góc so le trong).

• Tam giác cân có hai góc đáy

Tam giác cân nên , suy hay Có (chứng minh phần 1)

( đối xứng với qua )

• Hai tam giác có góc hai cạnh kề góc tương tứng theo trường hợp “cạnh - góc - cạnh” (c – g – c)

Xét có: + ;

+ ;

P A M M AP

PM AM= ⇔PM HM AM HM+ = +

2

PH AH HM HM AH HM AH AN HN

⇔ = + + = + = + =

H

PN

PNB

BH H PN

BH NP⊥ ∆PNB B

PNB

B BP BN=

ACPB M BC AP ACPB

ACPB AC PB= AC PB AC BN

BP BN

 =

⇒ =

 =

ACPB PAC APB

PBN

B APB ANB ANB PAC

  CAN BNQ

 

;

AC NB NQ AN 

  ; ; ;

BNQ CAN NBD NCD N B C D

     C G;

DQG DBCCAG BQG 

 

GBQ GCA GBQ∽GCA GA GQ GA GQ

AC QB NB NC

   

  

BNC BDC AGQ  NBC∽GAQ

   

GQA NCB NCB GDC GC NB NG BC

       

ACPB AC PBPAC APB

PBN

B  APB ANB= PAC ANB = CAN BNQ =

AC NB

NQ ANQ A N

CAN

∆ ∆BNQ

CA NB=

(78)

+ ;

Suy (c – g – c), suy hay

• Tứ giác có hai đỉnh liên tiếp nhìn cạnh hai góc tứ giác nội tiếp

Tứ giác có hai đỉnh liên tiếp nhìn cạnh với hai góc nên tứ giác nội tiếp hay thuộc đường tròn 3) Ta có

Mà ;

và ;

Nhận xét: Chứng minh hai đoạn thẳng ta chứng minh cho chúng đoạn thẳng thứ ba

Nhắc lại kiến thức phương pháp:

• Tứ giác nội tiếp có góc góc ngồi đỉnh đối diện tứ giác nội tiếp nên

• Hai góc nội tiếp chắn cung

(hai góc nội tiếp chắn cung đường trịn ngoại tiếp tứ giác )

• Hai tam giác có hai cặp góc đồng dạng theo trường hợp “góc - góc” (g – g); Tam giác đồng dạng có cặp cặp tương ứng tỷ lệ góc tương ứng

Xét có:

+ ;

+ ;

Suy (g – g), suy

• Hai tam giác có hai góc hai cặp cạnh kề góc tương ứng tỷ lệ đồng dạng theo trường hợp “cạnh - góc - cạnh” (c – g – c); Tam giác đồng dạng có cặp cặp tương ứng tỷ lệ góc tương ứng

Xét có:

+ ;

+ ;

Suy (c – g – c), suy ,

Câu 4.Giả sử mặt phẳng có n điểm thẳng hàng tồn đường thẳng.Theo điểm cho không nằm đường thẳng nên tồn điểm khơng nằm đường thẳng nối điểm với điểm cho ta

AN NQ= CAG BQG 

CAN BNQ

∆ = ∆  ACN NBQ=  DCN NBD=

NBCD B C; DN DCN NBD =

NBCD N B C D; ; ;

  CAG BQG

 

GBQ GCA GBQ∽GCA GA GQ GA GQ

AC QB NB NC

   

  

BNC BDC AGQ  NBC∽GAQ

   

GQA NCB NCB GDC GC NB NG BC

       

DNGB CAG BQG 

 

GBQ GCADG

BCGD

GBQ

 GCA

  CAG BQG

 

GBQ GCA

GBQ GCA

 ∽ GA GQ GA GQ ACQBNB NC

NBC

 GAQ

GA GQ NB NC

  

BNC AGQ BDC

NBC GAQ

 ∽ GQA NCB 

 

NCB GDC GC NB NG BC

     

1

nn1

(79)

đường thẳng với đường thẳng qua điểm ta đường thẳng, thay tồn đường thẳng

Nhận xét: Bài toàn tư từ việc lập đường thẳng qua điểm kết hợp với tính tốn số đường thẳng tạo thành

Nhắc lại kiến thức phương pháp:

• Với điểm thẳng hàng ln tồn đường thẳng qua điểm

• Với điểm thẳng hàng điểm nằm ngồi đường thẳng qua điểm Khi từ điểm ta kẻ đường thẳng tới điểm Trường hợp ta có đường thẳng tạo thành Đây trường hợp có đường thẳng tạo

Thay ta có đường thẳng phân biệt tạo thành

Đề s 11

Câu 1) Điều kiện: − ≤ ≤1 x

Ta có: ( )2

1+ +x 1−x = +2 1−x

Do phương trình cho tương đương với

( ) [ ]

3

2 2

1 1

2 1

0 1,1

+ + − = ⇔ + + − =

⇔ + − = ⇔ − = ⇔ =

⇔ = ∈ −

x x x x

x x x

x

Vậy phương trình cho có nghiệm x = 2) TừHPT ta có:

( 2) 2 2 ( )( )

4 xxy+ y =x +xy+2y ⇔3x −5xy+3y = ⇔0 xy 3x−2y =0

Với x – y = hay x = y, thay vào PT thứ HPT ta có 2

1

− + = ⇔ = ±

x x x x

Với 3x – 2y = hay

2 =

y x , thay vào PT thứ HPT ta có:

2 2

1

2 7

− + = ⇔ = ± ⇒ = ±

x x x x y

Vậy HPT có bốn nghiệm (x; y) là: ( ) (1;1 , 1; ,) ; , ;

7 7

   

− −   − − 

   

Câu 1) Đặt a= 1,b= 1,c=1

x y z

Từ giả thiết ta có ab + bc + ca = Do đó, đẳng thức cần chứng minh tương đương với:

nn n2015

2015

n n

1

nn1

1

nn1

n

2015

(80)

( )( )( )

( )( ) ( )( ) ( )( ) ( )( )( )

2 2

2

1 1

2

+ +

+ + =

+ + +

+ + +

+ +

⇔ + + =

+ + + + + + + + +

a b c bc ca ab

a b b c c a

a b c

a b c bc ca ab

a b a c b c b a c a c b a b b c c a

( ) ( ) ( )5

a b+ +c b c+a + c a+b bc+ ca+ ab (luôn đúng)

Vậy đẳng thức chứng minh

2) Đặt u = x + y, v = xy phương trình cho trởthành:

2

2

3

1 + + = + ⇔ =

+

v

v u u v u

v

Do x y; ∈Z+ nên u; v∈Z+suy v2 +1 ước v+ ⇒3 v2 +1 ước v2 − ⇒9 v2 +1là

ước 2

1 10

+ − ⇒ +

v v ước 10

Lần lượt xét ước nguyên 10 đểxác định v, u tìm nghiệm nguyên dương x, y tương ứng, thử lại với PT ban đầu

Phương trình có ba nghiệm ngun dương (x; y) là: (0; 3), (3; 0), (1; 1)

Câu

1) Ta có ∆ABFvà ∆ACE đồng dạng chúng cân F, E

FBA= ∠BAD= ∠DAC = ∠ECA

2) Gọi G giao điểm BE CF Ta có: GF = BF = AB = DBDG/ /BF

GC CE AC DC Mặt khác,

DA//BF suy A, D, G thẳng hàng, suy đpcm

(81)

Suy đpcm

Câu Áp dụng BĐT Cauchy ta có:

4 2

4 2

4 2

1

;

3

1

;

3

1

3

+ + ≥

+ + ≥

+ + ≥

a b abc ca a bc

b c bca ab b ca

c a cab bc c ab

Cộng theo vế ba BĐT thay ab + bc + ca = vào rút gọn ta được:

( ) 4 ( )

2

3abc a+ +b ca b +b c +c a +9

Ta có

( ) 1( )2 ( ) ( )

3 3

+ + = + + ≤ + + = ⇒ + + ≤

abc a b c ab ca bc ab ca bc ab bc ca abc a b c

Cộng theo vế(1) (2) ta có đpcm Đẳng thức xảy

3 ⇔ = = =a b c

Đề s 12

Câu

1) Dễ thấy đẳng thức sau với

, suy Do đẳng thức cho tương đương với

điều phải chứng minh

Nhận xét: Bài tốn sử dụng đẳng thức (bổđềln đúng) đểghép vào biểu thức cho để chứng minh

Ý tưởng: Trước hết, xét đẳng thức cần chứng minh ta có ta tách thành Việc tách có sở số xuất vế phải giả thiết, mối liên hệ mẫu số phân thức vếtrái Với thếngược lại giả thiết toán, tức ta cần chứng minh

(*)

a b

  

2 2

2

b b b a b a b a b

 

  

2 2

2 .

b b b a b a b a    b

2 4 8

2 2 4 4 8 8

2 2 4 4

y y y y y y y x y x y x y x y x y x y x y

   

   

 

        

          

4 4 ,

y y x y y x x y

      

5y4x

4 y

y x y

x y

   

4 y

x y

2

2 4 8

2

(82)

Đẳng thức (*) hồn tồn chứng minh phương pháp biến đổi tương đương là:

Với cách biến đổi trên, để làm xuôi ngược lại ta sử dụng bổ đề đẳng thức

đẹp

Bài toán kết thúc

2) Hệđã cho tương đương với , suy

Ta có

+ Với , thay vào phương trình đầu hệta có

+ Với , thay vào phương trình đầu hệta có

Vậy hệcó nghiệm

Nhận xét: Bài toán sử dụng phương pháp số hai phương trình, sau phương trình thu phân tích thành nhân tử ngược lại hai phương trình hệtìm nghiệm hệphương trình

Ý tưởng: Thoạt nhìn, ta nghĩ đến hướng xét delta ẩn phương trình thứ hai hệ mong muốn đenta phương Nhưng hướng thất bại, dễ thấy từ phương trình hai, ta tách Mặt khác, xét vếtrái phương trình coi phương trình đẳng cấp bậc

hai, ta có Khi hệ phương trình cho tương

đương với

Đây hệ đẹp nhân tử xuất hai phương trình, thếsuy ra:

2

2 4 8

2 0

y y y y y x y x y   xyxyxy

2

2 2 4 8 2y 2y 4y 8y 0 x y x y x y x y

     

   

4 8

4 4 8 8 8

4y 4y 8y 0 8y 8y 0 x y x y x y x y x y

        

    

2 2

2

b b b a b a b a b

 

  

      

2 12

6 12

x y x y x y xy x y

   



    



 2   6 

2

x y x y x y x y xy

x y xy   

         

    

2

2

x x y xy x y

y   

         

x

2

18 3 12

2

y y y

y   

     

2

y

2

2 12 12

4

x x x

x   

     

x y;   3; , 3; , 4; 2     

x y

      

6 x y xy x y 12 x y xy  6 12

  

2

2x 3yxy x y x2 3y

     

2 12 12

x y x y x y xy

   



   



; 12

x y

 2    6

2 (*)

x y x y x y x y xy

x y xy  

         

(83)

Với (*), dễ thấy nhân tửnhư sau: (*) Việc cịn lại thếngược lại tìm nghiệm hệphương trình Bài tốn kết thúc

Câu

1) Do số nguyên lớn nên

Mà sốchính phương ;

nên ta có , điều phải chứng minh Nhận xét Bài toán chứng minh đẳng thức từ điều kiện cho Nhắc lại kiến thức phương pháp

• Xét

+ Với ta có

+ Với ta có Suy

+ Bình phương hai sốnguyên có giá trị tuyệt đối hai số tự nhiên liên tiếp gọi hai sốchính phương liên tiếp Giữa có số phương với sốnguyên

+ Ta có có sốchính phương mà suy

(trái với điều kiện )

• Xét

Chứng minh hoàn toàn tương tựnhư trên, ta (trái với điều kiện ) Mối quan hệ số nguyên hoặc

Ta có khơng tồn nên chỉcó (điều phải chứng minh) 2) Ta có

 3 2

2

x x y

y   

      

;

x y x y; 2

4xy 7x 7y 4xy

      

2 2 2

4x y 4xy 4x y 7x 7y 4x y 4xy

        

2xy 12 4x y2 7x 7y 2xy 1 2

      

2

4x y 7x7y 2 xy 1 2xy1

 2

2

4x y 7x7y 2xy  x y

x y

x y 7 7

0

x y x y

x y xy xy xy

 

     

      

 

    

 

 

;

x y 4xy           1 8x 7x x 7x y 7x 7y 4xy 7x 7y 4xy

     

2 2 2

4x y 4xy 4x y 7x 7y 4x y 4xy

        

2xy 12 4x y2 7x 7y 2xy 12

      

n12 n12

n n

2xy12 2xy12  2xy

2xy124x y2 27x7y2xy12

 2

2 2 2

4x y 7x7y 2xy 4x y 7x7y4x y 7x 7y x y

      x y

y x

x yy x

a ba ba b

x yy xx y

3 2

xyxyxy

x y x y2 xyx2 y2 xy

(84)

+ Với

+ Với ,

suy , Dấu “=” xảy , Dấu “=” xảy

Vậy

Nhận xét: Khai thác giả thiết biểu thức cho, tìm điều kiện chặn biến để tìm giá trị nhỏ nhất, giá trị lớn biểu thức

Nhắc lại kiến thức phương pháp:

• Hằng đẳng thức

• Xét biểu thức , với , ta có:

Ý tưởng: Đi từ giả thiết toán, xuất làm ta nghĩ đến đẳng thức , giả thiết trởthành:

Ta cần xét với , mặt khác kết hợp với điều kiện biểu thức ta có điều

kiện chặn

Suy

Do

Bài tốn kết thúc

Câu

2 0 0

1

x y x y xy

x y x y

      

 

 

     

 

5

0

2

x y   P

1 ;

x y   x y

1 4

2

P    

  x1; 0y

1

2 1

P    

  x0; 1y

max 1;

P   x y m in 0;

3

P   x y

  

3 2

ab  a b aab b

   

m p f x P

n q f x  

 0 a f x b

min ; max

m p a m p a m p b m p b

P P P P

n q b n q b n q a n q a

   

     

   

3

xy

  

3 2

xyx y x xy y

  

3 2 2 2

xyxxy y  x y x xy y xxy y

 1 2 0

1

x y x y x xy y

x y    

         

1

x y  ,

x y , 1

0

x x y

y    

   

  



1 1; 2

3

2

2

x x P x x y y y y

 

       

 

     

 

       

 

 

 

1 2; 4

2

2 1

x x P x x y y y y

 

       

 

     

 

       

 

 

 

max 1;

P   x y m in 0;

3

(85)

1) Ta có suy tứ giác nội tiếp

Nhận xét Để chứng minh bốn điểm thuộc đường tròn ta đưa chứng minh tứ giác cách chỉra tứgiác có tổng hai góc đối diện

Nhắc lại kiến thức phương pháp

• Các góc nội tiếp chắn cung đường trịn

+ (hai góc nội tiếp chắn cung đường tròn ngoại tiếp tam

giác )

+ (hai góc nội tiếp chắn cung đường tròn ngoại tiếp tam giác )

• Tứ giác có tổng hai góc đối diện tứ giác nội tiếp

(hai góc kề bù) nên , suy tứ giác tứ giác nội tiếp hay bốn điểm ; ; ; thuộc đường tròn

2) Từ tứ giác nội tiếp, suy (1)

Ta lại có (2)

Từ (1) (2), suy

Nhận xét Nhớ lại trường hợp đồng dạng hai tam giác kết hợp với kiện đề ý chứng minh từý để tìm hướng chứng minh toán Đối với toán này, ta chứng minh hai tam giác đồng dạng theo trường hợp “góc - góc”

Nhắc lại kiến thức phương pháp

• Tứ giác nội tiếp có góc góc ngồitại đỉnh đối diện Tứ giác tứ giác nội tiếp (chứng minh trên) nên

• Các góc nội tiếp chắn cung đường trịn

K L

Q F

E O A

B D C

P

    180

AFC AEB ADC ADB     AEPF

AEPF 180

 

AFC ADCAC

ACD

 

AEB ADBAB

ABD

180

  180

ADC ADB    AFC AEB 180 AEPF

A E P F

AEPF AEB LFC =

  =

FCL FCB BCL =PBC BAQ DAE BAQ BAE     =  =

FCL EAB

 ∽

(86)

+ (hai góc nội tiếp chắn cung đường trịn )

+ (hai góc nội tiếp chắn cung đường trịnngoại tiếp tam giác )

• Tam giác cân có hai góc kề đáy

Tam giác có nên tam giác cân P suy

Kết hợp lại, ta

• Hai tam giác có hai góc tương ứng đồng dạng

Xét có: (chứng minh trên) (chứng minh

trên), suy (g – g)

3) Từ , suy hay (3) Chứng minh tương tự (4)

Từ (3) (4), suy hay , suy

Ta lại có

Tương tự ta có

Suy

Nhận xét Với toán chứng minh hai tổng nhau, ta dựa vào mối quan hệ góc đểđưa vếcùng lượng

Nhắc lại kiến thức phương pháp

• Tính chất hai tam giác đồng dạng

Chứng minh hoàn toàn tương tự ta có

• Định lý Ta-lét đảo

Từ chứng minh trên, ta , suy

• Đường thẳng cắt haiđường thẳng song song tạo góc đồng vị (chứng minh trên), suy

• Các góc nội tiếp chắn cung đường trịn

(hai góc nội tiếp chắn cung đường tròn ngoại tiếp tứ giác )

Kết hợp ý chứng minh ta có điều trình bày giải Hoàn toàn chứng minh tương tự ta có Từ ta có điều cần chứng minh

Câu

Từ giả thiết dễ thấy tập thuộc dãy phân biệt Vì có 31 phần tử nên số tập có phần tử Ký hiệu số tập có phần

 

BCL BAQBQ ( )O

 

EAD EBCDE

ABD

PBC

PB PC PBCPBC PCB

     

BCL FCB ABQ QAE   FCL BAE

FCL

 EAB  AEB LFCFCL BAE

FCL EAB

 ∽

FCL EAB

 ∽ FL FC=

BE AE FL EA FC EB =

=

EK FA FC EB

=

FL EA EK FA FL EK=

FA EA EF KL

       =

QLK ALK ALQ AFE ABE APE ABE PAB     

  QKL PAC

    QKL PAB QLK PAC  

 

=

FL FC FL AE FC EB

FCL EAB BE AE

FLC ABE

  



   

 



=

EK FA FC EB

FL EK=

FL AE EK FA

FA EA

  EF KL

EF KL AFE ALK

 

AFE APEAE AFPE

       =

QLK ALK ALQ AFE ABE APE ABE PAB     

 

QKL PACQKL PAB QLK PAC     

m A

A 31 30

2

  2 31

k

(87)

tử, nằm dãy cho, suy Xét tập hợp có phần tử suy số tập có phần tử tập tập có tập phần tử Mà theo giả thiết với phần tử chúng khơng thể đồng thời thuộc tập có phần tử dãy tập phần tử nói phân biệt

Suy

Vậy điều phải chứng minh Nhận xét Bài toán phần nguyên Nhắc lại kiến thức phương pháp

• Bài tốn cách chọn số từ tập hợp số: “Trong n số ta chọn n cách chọn số thứ nhất, cách chọn số thứ hai, có cách chọn hai số khác từ n số cho”

Từ giả thiết dễ thấy tập thuộc dãy phân biệt Vì có 31 phần tử nên số tập có phần tử Ký hiệu số tập có phần tử, nằm dãy cho, suy Xét tập hợp có phần tử suy số tập có phần tử tập tập có tập phần tử Mà theo giả thiết với phần tử chúng khơng thể đồng thời thuộc tập có phần tử dãy tập phần tử nói phân biệt

• Phân số có mẫu tích hai số tự nhiên liên tiếp: “Với x số tự nhiên thỏa mãn điều kiện xác định ta có ”

2 31

m a a   a k

 1

2 k

k k

a

  1

2

k k k aA

k

 1 31 30

2 k

k k a

 

 

31 30

1

k a

k k

   

2 31 31.30 1.2 2.31 30.311

a a a  

         

 

 

1 1 1

31.30

2 30 31

m            

  

900,

m

1

n  1

2

n n

m A

A 31 30

2

  2 31

k

a  k k

2 31

m a   aa k

 1

2 k

k k

a

  1

2

k k k aA

k

 1 1x x11

x x   

 1 31 30

2 k

k k a

 

 

31 30

1

k a

k k

   

2 31

1 1

31.30

1.2 2.3 30.31

a a a  

         

1 1 1

31.30 31.30 900 2 30 31 31

m                 

    

 

(88)

Đề s 13

Câu

1) Điều kiện:

Phương trình cho tương đương với

Đối chiếu với điều kiện ta được nghiệm:

Nhận xét: Bài toán sử dụng phương pháp nâng lũy thừa (bình phương) hai vếtìm nghiệm phương trình

Nhắc lại kiến thức phương pháp:

• Phương trình dạng

• Phương trình có cách giải khác sau:

Ý tưởng: Đây phương trình bản, dạng tốn vế chứa hai thức vế lại sốthì phương pháp nâng lũy thừa hai vế phương pháp tối ưu Bài toán kết thúc

Bài tập tương tự:

1. Giải phương trình Đáp số:

2. Giải phương trình Đáp số:

1 2

3 x

  

  

2x 3 3x1 2x 9

2

3x 5x x

      2 2

3

x

x x x x   

  

    



2

4 11 7

x x x

x    

    

  

7

1;

4

xx

    0     2

f xg x   m f xg xm

    2     2        

f x g x f x g x m f x g x m f x g x

       

   

       

2

1

2

2

4

m f x g x x x x x f x g x m f x g x

   

 

 

   

  

   



       

f xg x  m f x  m g x

 

     

 

     

2 2 2

m f x m f x

f x m m g x g x m g x g x m f x

 

   

 

 

 

     

 

 

 

     

      

2

1

2 2

;

4

m f x g x m f x x x x x m g x g x m f x

    

 

 

 

    



3x 1 x 1

8 x

7x 4 x 1

(89)

2) Hệphương trình tương đương với

Đặt

Hệphương trình trở thành Suy

Hệ phương trình có nghiệm

Nhận xét: Bài tốn sử dụng phương pháp đặt hai ẩn phụ, đưa hệ phương trình bậc hai giải phương pháp Sau từ nghiệm ẩn phụ suy ngược lại nghiệm hệ phương trình

Ý tưởng: Hình thức tốn phực tạp xuất phân thức, quan sát ta thấy hai phương trình hệ xuất biểu thức Ta nghĩ đến chuyện xuống phương trình hai cịn đại lượng chưa biết xử lý Có lẽ tác giá gợi mở theo đường đặt ẩn phụ, đặt ta cần biểu diễn qua hệ phương trình cho giải Ta có

1

2

1 1 2

4

x y y x

x x y y y x

                                                        1 u x y v y x         

1 2

4 u v u uv          

9 2

v u

u u u                        2

9 3 0

4 2

u u u u u

       3 0 2 3 u u v                 3 x y y x            3

1 3 3

2 2 2

1

y y y xy x x y

y xy x                

2 3 2 0 12

2 y x y y y x                

 ;  1; , 1; 2 

2

x y  

 

1

x y

1

2 x y y x            xy xy

1;

u x v y y x

    xy

xy

u v;

1 1

1 1

u x uy xy y

v y vx xy x               

 2

1

uvxy xy uv xy xy

(90)

Khi đó, hệ phươngtrình cho tương đương với

Hệ phương trình hệ phương trình bản, hồn tồn giải phương pháp

Bài toán kết thúc Bài tập tương tự:

1. Giải hệ phương trình

Đáp số:

2. Giải hệ phương trình

Đáp số:

Câu

1) Cách 1: Đẳng thức cần chứng minh tương đương với

, điều phải chứng minh

Cách 2: Đặt

Từđiều kiện suy , ta

9

1 2

4

u v

u uv    



   



 

2

2 2

1

1

x y

xy x y x

  



   



 ;    1; , 1; , 5; , 7;

4 7

x y         

   

   

   

  

2 2

1 27

1 10

x y xy x y xy

   



   



 ;  1; , 2; 2 , 2 3;

2

x y       

   

   

3

1 1

4

a b b c c a a b b c b c c a c a a b

     

        

     

     

           

         34

ac ba cb a b b c b c c a c a a b

   

     

      34   

ac a c ba b a cb c b a b b c c a

         

     

ac a c ba b a cb c b abc

      

  2    8

ac a c b a c ba abc c b abc abc

        

a c ac b ab bc 8abc

     

a c c b b a   8abc

    

1 1

a b x x

a b b a b c y y

b c b c c a z z

c a c a

 

 

    

 

   

 

 

 

    

 

   

 

 

 

    

 

   

 

 

1

8

xyz

1 1 1     

xyz xy      z x y z xy yz zx  xy

   

2xyz x y z xy yz zx

       

3

x y z xy yz zx

(91)

Nhận xét: toán sử dụng phép biến đổi tương đương ẩn phụđể chứng minh đẳng thức cho

Ý tưởng: Nhìn đẳng thức cần chứng minh ( gọi (*)) cồng kềnh, nhiên tinh ý chút, ta thấy bên vế trái (*) có tổng ba thừa số, đồng thời vế phải (*) xuất tổng hốn vị tích hai thừa số Vì chuyển vế ta nhóm nhân tử chung là:

( )

Với biểu thức ( ), hướng tối ưu có lẽlà quy đồng mẫu số biến đổi tương đương, kết hợp với giải thiết ta có:

( )

Hoặc, ta có thểđi với hướng tư ẩn phụhóa đểđơn giản tốn chút Vẫn hướng phát bên trên, ta đặt ẩn phụ thừa số Khi giả thiết ta cần chứng minh (**) Nếu dựa vào giả thiết để chứng minh (**) chưa đủ, ta cần phải khéo léo kết hợp với giải thiết tốn sau Và từ suy đẳng thức

Khai triển tích số, ta sẽđược điểu phải chứng minh Bài toán kết thúc

2) Ta có

Suy chia hết cho chia hết cho Ta có

Vậy số có chữ số lớn chia hết cho Ta có

Vậy số có chữ số nhỏ chia hết cho

Số số có chữ số thỏa mãn yêu cầu toán là: Đáp số: số

Nhận xét Bài toán chứng minh đẳng thức từ điều kiện cho Nhắc lại kiến thức phương pháp

3

1 1

4

a b b c c a a b b c b c c a c a a b

     

        

     

     

           

         34

ac ba cb a b b c b c c a c a a b

   

      i

i

a b b c c a      8abc

iac a c   ba b a   cb c b   43a b b c c a     

     

ac a c ba b a cb c b abc

      

  2    8

ac a c b a c ba abc c b abc abc

        

a c ac b ab bc 8abc

     

; ;

a b c x y z

a b b c c a

  

  

1

8

xyz

 

4

x y z   xy yz zx 

1 x b ; y c ; z a

b a b c c a

     

  

1 1 1 

xyz xyz

00 100

abcde abc de abc  de

101 1 101

abc de abc abc de

      

abcde 101abc de abc  10d e  101

99999

101 99999 990

101 101

m m

     

101 990 101

999

101 9999 99

101

n n

    

5 101 100 101

5 990 100 891.  

(92)

• Cấu tạo số

• Tính chất chia hết tích: Trong tích có thừa số chia hết cho số tích chia hết cho số

Ta có

• Tính chất chia hết tổng: Tổng hai số hạng, có số hạng chia hết cho số số hạng cịn lại chia hết cho số

Ta có , suy

• Số lớn có năm chữ số chia hết cho 101

Ta có suy số có năm chữ số lớn chia hết

cho

• Số bé có năm chữ số chia hết cho 101

Ta có suy số có năm chữ số nhỏ chia hết cho

• Số số dãy số viết theo quy luật tính theo cơng thức số cuối, số đầu, khoảng cách số liên tiếp dãy

Số số có chữ số thỏa mãn yêu cầu toán là:

Câu

1) Ta có góc nội tiếp suy

hay (2)

 

00 100 101 101

abcde abc de abc  de abc  de abc  abc de

 

101 101  abc101 101

 

 

101 101

.101 101

abc

abcde abc de abc  



  

   

  

  



 de abc 101

abc 10d e 101

   

99999

101 99999 990

101 101

m m

     

101 990 101

999

101 9999 99

101

n n

     101

100 101

1

c d s

h

 

c d h

990.101 100.101 990 101 891

101

     

A

B C

O

D M

F E

N

  (1)

BDM BCFBMA BFA

 

0

(93)

Từ , suy đồng dạng (g - g)

Nhận xét Đây toán tương đối thường gặp toán chứng minh tam giác đồng dạng ứng dụng góc nội tiếp

Nhắc lại kiến thức phương pháp

• Các góc nội tiếp chắn cung đường trịn + (hai góc nội tiếp chắn cung đường trịn )

+ (hai góc nội tiếp chắn cung đường tròn ngoại tiếp tam giác )

• Hai tam giác có hai cặp góc tương ứng đồng dạng

+ có , suy (g – g) 2) Từ phân giác suy vng góc với trung điểm

Từ 1) , ta có

Vậy ta có biến đổi sau (3) Ta lại có góc nội tiếp (4)

Từ , suy suy Vậy

Nhận xét Với toán ta đưa chứng minh EF tạo với AC góc vng Dựa vào góc biết kết nối tam giác đồng dạng

Nhắc lại kiến thức phương pháp

• Hai góc nội tiếp chắn hai cung hai cung hai dây cung cung

Từ phân giác suy suy kết hợp với (

) suy trung trực hay vng góc với trung điểm

• Các góc nội tiếp chắn cung đường trịn (hai góc nội tiếp chắn cung đường trịn )

• Các liệu suy ra, biến đổi từ điều chứng minh

• Hệ thức lượng tam giác vng “Cạnh huyền Đường cao = Tích hai cạnh góc vng”

Áp dụng hệ thức lượng vào tam giác vng , đường cao ta có kết hợp với chứng minh ta kết hợp với trên, ta suy (g – c – g)

• Góc nội tiếp chắn nửa đường trịn góc vuông

(1) (2) BDMBCF

 

BDM BCFAB ( )O

 

BMA BFAAB

ABM

   

0

180 BMA 180 BFA BMD BFC

     

BDM

 BCF BDM BCF BMD BFC BDM∽BCF

AD BACDB DCDE BC N

BC

BDM BCF

 ∽ DM BD

CFBC

2

DA DM BD CD DE

CFCFBCCNCE

 

ADE FCE

(3) (4) EAD∽EFC EFC EAD 90 EF AC

AD BACBAD DAC DB DCOB OC

R

DO DE BC DE BC

N BC

 

ADE FCEAE ( )O

2

2

DM BD DM BD DA CD

BDM BCF

CF BC CF CN CF CN

 ∽      

CDE

C CN

CD DE

CN DE CE CD

CN CE

   DA DE

CFCE

EAD EFC

(94)

Góc chắn nửa đường trịn chia đường kính nên suy

ra hay vng góc với

Câu

Với số thực dương ta có

;

Cộng bốn đẳng thức ta thu

Ta tìm cho

Chọn , ta

Ta có nghiệm dương

Với xác định ta thu

Đẳng thức xảy

Vậy giá trị nhỏ

Đề s 14

Câu

1) Cộng hai phương trình hệta thu

Ta ln có đẳng thức xảy Vậy ta suy (loại) khơng thỏa mãn phương

trình

EAD ( )O ED EAD 90

 90

EFC  EF AC

3 3 3

3 ; 3

3 3 3

d a b dab d b c dbc

      33 33 2 ; 32 2

3 3

d c a dca a b c abc

 

   

 

3 3

3 2

2 1

3

d a b c dab dbc dca abc

 

        

 

0

 3

3

2 2 4 3 6

9

3 3    

1

2 x x

   

 

3

1 6

2 x x x x

   

      

   

   

   

3

3

1 3 6 12 1 0

2 x x x x x x x x

     

  

             

     

36 35 ; 36 35 36 35 36 35

2

x  x        

 

3 3

2

4

9

d a b c

   

 

3 3

2

3

9 36

9

6 35 35

d a b c

     

 

    

 

 

3

3

1

; =

3

a b c d

  

 

P 2

3

36

6 35 35

 

    

 

 

   

  

3

3 3

2

6

2 2

x y xy x y xy x y x y xy y x

         

        

 2    

2 2 2 2

xy   xy  y   x x y  2

2 4 2 2 0

xy  xyyxx y  2

(95)

Vậy thu hệ

Suy

Nhận xét: Bài toán sử dụng phương pháp (hay cộng vế) để phương trình có mối liên hệ biến Sau thếngược lại tìm nghiệm hệphương trình

Nhắc lại kiến thức phương pháp:

• Tổng đại lượng khơng âm:

• Đẳng thức:

Ý tưởng: Cả hai phương trình hệ, xuất nhân tử ta sẽnghĩ đến chuyện từphương trình vào phương trình hai (hoặc ngược lại), ta có ( ) Đến ta mong muốn biểu diễn mối quan hệ , quan sát phương trình ( ), ta thấy đặt

thì ( ) Một biểu thức đối xứng đẹp, cách nhóm nhân tử, ta có:

(*)

Dễ thấy đó, phương trình

Cơng việc cịn lại thay suy Nhưng

loại khơng thỏa mãn phương trình hai hệ Với thay xuống phương trình hai, ta tìm nghiệm hệphương trình

Bài toán kết thúc Bài tập tương tự:

1. Giải hệphương trình

Đáp số:

2. Giải hệphương trình Đáp số: vơ nghiệm

2

7

x y y x xy y x

      

   



  1

7 2 7 12 5 9

7

x y x x x x x

x y     

        

    

a b  2 b c 2 a c2 0 a2b2c2ab bc ca 

  

3 3 3 2

abcabc  a b c ab   c ab bc ca

x yx y

3 6 8 0

xyxy  i x y,

i  2 3 8 6xy3xy 2 z 2

ix3y3z33xyz0

 3  

3 3 3 0 3 0

xyzxyz  x y zxy x y z  

x y z x 2xy y2 xz yz z2 3xy x y z  0

           

x y z x y2 z2 xy yz xz 0

        

  2  2 2

2 2

2

xyzxy yz xz    x y  y z  z x 

 

        x y zx y z

z 2

2

x y x y    

  

x y  2

2

x y 

 ;   1; , 9;

7

x y   

 

3

2

1

2

x y xy x y x y     

     



 ;   1; , 33; 33 , 33; 33

4 4

x y            

   

   

3 2 3

4

x y x xy x xy

    



  

(96)

2) Điều kiện

Phương trình tương đương với

+ Giải

+ Giải Đáp số

Nhận xét: toán sử dụng phương pháp nhóm nhân tử chung, sau nâng lũy thừa bậc hai đểtìm nghiệm phương trình

Nhắc lại kiến thức phương pháp:

• Giải phương trình

• Giải phương trình

Ý tưởng: Bài toán xuất ba thức, có điều đặc biệt thức cịn lại tích hai thức Mặt khác có sựđồng hệ số, ta sẽnhóm hai lại nên ta nhân tửchung sau: Và ta mong muốn biểu thức phân tích biểu thức có chứa Thật vậy, coi phương trình bậc hai ẩn ta thấy: Chính tốn ta giải sau:

Phần lại việc bình phương phương trình tìm nghiệm ởtrên nêu Ta nghiệm phương trình

Bài tốn kết thúc Bài tập tương tự:

1. Giải phương trình

Đáp số:

2. Giải phương trình

1 x

  

 2 2

2 x1  1xx 1 1 x x1

x 1 xx x 1 x 1 x

             x 1 1 x 2 x  1 1

2

1 2 1

x     xx   x   x

1

x   x

0

x

       

0

0

f x f x g x

g x

 

   



   

a f x  a f x b

 

 

 

   2

2 2 2

2

a f x a a f x a

a a f x b a f x b a

 

     

 

   

     

   

   

 

2

1x , 1x

 

2

1x  1 x 1x x 1

3

x  xx 1

  3

h x   x xx1

   1 2

h x   x x   x  x 

2

3 1

x  xx  x

x 1 x 2 x x 1

         

 1 1 2 1

1

x x x x

x x

  

         

   



x

2

4 2

x  xx  x

1

x

2

(97)

Đáp số:

Câu

1) Nhận xét: số nguyên thỏa mãn thật vậy,

suy

Phương trình tương đương với

suy ( )

Thay vào phương trình ta thu

Lập luận tương tựta thu ( ) Và nhận phương trình

Tương tựta có ( ) thu Từphương trình suy

Suy

Đáp số: , , ,

Nhận xét Bài toán nghiệm nguyên giải phương pháp xét số dư hay đồng dư thức Nhắc lại kiến thức phương pháp

• Một số phương chia hết cho tồn dư Do tổng hai số phương chia cho dư dư Nên số nguyên thỏa mãn

thì ,

suy

• Một số chia hết cho có dạng

Phương trình tương đương với Suy

( )

0

x ;

a b a2b23 a b; 3

   

2 0,1 mod3 ; 0,1 mod3

ab

      2 2 mod3

0 mod3 ,

0 mod3

a

a b a b b          

6x29y2  x2y228 9

 

2 0 mod3

xy   

 

2

1

0 mod3

3 ; mod3

x

x x y y y

  

   

 x y1; 1

2

1

5 9 x   8 y 28 9

2 2 1

5 x y 28

     

1 ;

xx yy x y2; 2

2 2

2

5 9 x   8 y 28 9

2 2

5 x y 28

     

2 ; 3

xx yy x y3; 3 5x23 8 y3228

2

3 288

y  

2

3

2 2

3 28 y x y x            

2 2

3 ;

x y

  

2 2

2 ;

x y

   

2 2 2 2

1 ; 9 ;

x y x y

       

3

2 ;

x  yx 2 ;3 y 33 x  2 ;3 y33 x  2 ;3 y 33

;

a b a2b23

;

a b

      2 2 mod3

0 mod3 ,

0 mod3

a

a b a b b          

x= k

6x29y2  x2y228 9

 

2 0 mod3

xy   

 

2

1

0 mod3

3 ; mod3

x

x x y y y

  

   

(98)

Thay vào phương trình ta thu

Lập luận tương tựta thu

( )

Và nhận phương trình

Tương tựta có ( ) thu

• Hai số hạng khơng âm ln nhỏhơn tổng Từphương trình suy ,

suy

Đáp số: , , ,

2) Ta có

Đặt

Ta thu

Dấu “=” xảy

Nhận xét: Bài toán sử dụng bất đẳng thức Cosi kết hợp với giả thiết tìm giá trị nhỏ thức cho

Nhắc lại kiến thức phương pháp:

• Bất đẳng thức Cosi cho hai số thực dương

• Các hệ từ bất đẳng thức Cosi

Ý tưởng: Bài tốn có đối xứng hai biến ( vai trị chúng ) điểm rơi ta khẳng định , kết hợp với giả thiết suy Mặt khác, xét với biểu thức , có xuất đại lượng căn, nghĩ đến bất đẳng thức Cosi để đánh giá đại lượng

Ta có

2

1

5 9 x   8 y 28 9

2 2 1

5 x y 28

     

1 ;

xx yy x y2; 2

2 2

2

5 9 x   8 y 28 9

2 2

5 x y 28

     

2 ; 3

xx yy x y3; 3 5x23 8 y3228

2

3 288

y  

2

3

2 2

3

28

0

5

1

y x y x

   

 

   

2 2

3 ,

x y

   x22 9 ;2 y229

2 2 2 2

1 ; 9 ;

x y x y

       

3

2 ;

x  yx 2 ;3 y 33 x  2 ;3 y33 x  2 ;3 y 33

2

2 1 2

P x y xy xy xy

   

2 1

2

x y t xy    

1 16 15 2 16 15 17 17

2

P t t t P

t t

         

min

1 17

2

x y  P

2

a b  ab

2

1 2 ;

2

a b ab

a b ab

       

 

;

x y

x y

2

x y  Pmin

P xy

1

xy f xy 

2 2

1 x y 2

P xy x y xy xy xy

(99)

Và khai thác giả thiết, ta đánh giá , với điểm rơi ta có đánh giá Và đặt ta cần tìm giá trị nhỏ biểu thức với Bằng khéo léo chọn điểm rơi, ta đánh sau:

Dấu “=” xảy

Bài toán kết thúc Bài tập tương tự:

1. Cho số thực dương thỏa mãn Tìm giá trị nhỏ biểu thức

2. Cho số thực dương thỏa mãn Tìm giá trị nhỏ biểu

thức

Câu

1) Ta có , suy tứ giác nội tiếp, nên Mặt khác từ tứ giác nội tiếp ta có

Vậy thẳng hàng

Nhận xét Từ ba điểm tạo thành hai tia có chung gốc Khi hai tia tạo với góc tù ba điểm cho thẳng hàng

Nhắc lại kiến thức phương pháp

xy x y

 2

0

2

x y x y  xy   

t xy

1

P t

t

 

4

t

1 15

2 16 15 2 16 15 2 16 17

t t t t

t t

      

min 17

P

 

1

x y 

;

x y x y 2

2

1 1 .

P x y x y

 

 

     

;

x y x y 2

 

2 2 2 2 2

4x y x y P

y x x y

  

A

B C

O H

P F

N

M

E

K A

B C

O H

P F

N M

E Q

Q

  180 

BPC BHC  BAC AEPF  BFC BEC 1800

;

AQFN AQEM

     1800

MQN MQA NAQ MEA NFA    

; ;

(100)

• Hai góc nội tiếp chắn cung đường tròn

+ (hai góc nội tiếp chắn cung đường tròn ngoại tiếp tam giác )

+ (hai góc nội tiếp chắn cung đường tròn ngoại tiếp tam

giác )

+ (hai góc nội tiếp chắn cung đường trịn ngoại tiếp tam

giác )

• Tứgiác có tổng hai góc đối diện tứ giác nội tiếp Tứ giác nội tiếp có tổng hai góc đối diện

Gọi chân đường cao từđỉnh tam giác

Tứ giác có , suy tứ giác nội tiếp,

đó mà (hai góc đối đỉnh) nên

kết hợp với ta suy tứ giác tứ giác nội tiếp

Suy

kết hợp với trên, ta có

hay ba điểm thẳng hàng

2) Ta có góc nội tiếp suy Tương tự

Từđó tứ giác hình bình hành, suy phân giác

Nếu phân giác thẳng hàng

Từ giao

Vậy , suy

Tương tự

Từ trung điểm

Nhận xét Chứng minh đường thẳng qua trung điểm đoạn thẳng khác ta chứng minh giao điểm đường thẳng với đoạn thẳng trung điểm đoạn thẳng Nhắc lại kiến thức phương pháp

• Một đường thẳng cắt hai đường thẳng khác tạo cặp góc đồng vị hai đường thẳng song song

Từ kiến thức trên, ta có mà vị trí đồng vị hai đường thẳng nên suy Hồn tồn tương tự ta có

• Tứ giác có hai cặp cạnh song song hình bình hành

 

BPC BHC BC

HBC

 

AQN AFN AN FAN

 

AQM AEM AM AME

180

180

B CABC EF

AE HF   AE H AF H       90 90 180 AE HF 

   

180 180

F AE F HE   F HE  BACF HE  BHC

 180 

BHC BAC BPC180BACBPC BAC  180 AEPF

  180 180  180  180 AFP AEP    AFP  AEP  

  180   180

BFC BEC NFA AEM

       

  180  180

NAQ AQM   NQM  N Q M; ;

   

AFQ ANQ ANM ABM   FQ BEEQ CF

EQFP QAN QFP QEP QAM    AQ .

MAN

AP MANA P Q, ,

PQ BC K KAC QAC QME NMB PCK    

AKC CKP

 ∽ KC2KP KA

2 .

KBKP KA

KB KCK

   

AFQ ANQ ANM ABM   AFQABM

FQ BE FQ BE

(101)

Tứ giác có suy hình bình hành suy phân giác Do , , thẳng hàng (hai góc nội tiếp chắn cung đường ngoại tiếp tam giác )

(hai góc nội tiếp chắn cung đường trịn ),

mà lại có góc chung hai tam giác nên suy (g – g)

hoàn toàn tương tự ta có Từ suy trung điểm

Câu

Giả sử số mà Ký hiệu

Do suy

Ta có

Dấu “=” xảy

Đề s 15

Câu

1) Điều kiện

Phương trình cho tương đương với

+ Giải

+ Giải

Vậy phương trình có hai nghiệm:

EQFP FQ BEEQ CFEQFP

   

QAN QFP QEP QAM   AQ MANA Q P  

KAC QMEQE

AME

 

QME PCKNB  O

 

KAC PCK

  PKC AKCCKP

AKC CKP

 ∽

2 .

AK KC KC AK KP

CK KP

    KB2KP KA

2

KCKBKC KBK BC

k x1x2  xk 0 xk1  x192

1 k; k k 192

Sx x x Sx x x

 

       

2013

0 2013

2

SSSSSS

         

1 192

xx   x Skx S1; 192 k x 192

1 S S S ; 192 192S

x x x

k k k k

   

       

   

192 192S S 2 1922013 20132 2 1922013 192

x x

k k k k k k

  

      

  

   

2 2

192 192

2 192

2

k k k  k     

 

 

192 2013 192 2013 1922 96

2

x x

   

1 96 2013192 ; 97 98 192 2013192

xx   x   xx   x

6

x

x 9 2012 x  6 1

 2

2012 4048135 2012

x   x  

5

x   x

5 4048135;

(102)

Nhận xét: Bài toán sử dụng phương pháp nhóm nhân tử chung nâng lũy thừa tìm nghiệm phương trình

Nhắc lại kiến thức phương pháp:

• Cách giải phương trình dạng

Ý tưởng: Bài tốn cho đơn giản, với xuất hai thức

nên khơng khó để nhóm nhân tử chung sau:

Bài toán kết thúc

Bài tập tương tự:

1. Giải phương trình

Đáp số:

2. Giải phương trình

Đáp số:

2) Cách 1: Hệ cho tương đương với

Đặt

Thu

+ Giải

+ Giải (vơ nghiệm)

Vậy hệphương trình có hai nghiệm Cách 2: Hệ tương đương với Cộng vế với vế hai phương trình ta thu

 

      

 

   

2

f x m f x m

f x m g x n

g x n g x n

   

 

     

   

 

  

9; 6;

xxxx

  

9 2012 2012

x  x   xx

 9 6

9 x x 2012 2012

x x

        

   

9 2012

x x x

       

1 x 6 x 2012

     

2

2 x 6 x  7 x 13x42

3; x  x 

2

4 12

x  x   xx

0;

xx 

 

   

2 1 5

1

x y

x y x y     

      

 

       

2

2

1

1 2

1

u x y

x y x y x y u v v x y

   

          

   

2 2 5

5

u v u v    

   

 

2 2 5 5 2 15 0

5 10

u v u u u u

u v     

            

   

1

2,

1

x y x y x y x y

      

 

 

    

 



   

1

1 10

x y x y

    

  



 1;  2; 2 2 4

4 2

x y y x y xy     

   

(103)

+ Giải

+ Giải (vô nghiệm)

Vậy hệphương trình có nghiệm

Nhận xét: Bài tốn sử dụng phương pháp ẩn phụ sau từ ẩn phụ tìm ngược lại nghiệm hệ phương trình

Ý tưởng: Sự xuất phương trình hệ, làm ta nghĩ đến đẳng thức hay nói cách khác, từ phương trình ta có: Đây phương trình có dạng tổng bình phương, dễ làm ta suy đốn đến hệ phương trình đối xứng loại I, tức đặt ẩn phụ theo định lý Vi-et ( đặt tổng tích ) sau:

Nhưng suy đoán ban đầu, ta xét phương trình hai để xuất

Thật vậy, ta có phương trình hai hệ tương đương với: Do hệ phương trình cho trở thành Thế ngược lại tìm hệ phương trình ban đầu

Hoặc, ta suy luận sau: ta kết hợp hai phương trình hệ, với xuất phương trình một, đồng thởi có tích phương trình ta liên tưởng đến đẳng thức Vì lấy phương trình hai nhân cộng với phương trình ta được:

Thế ngược lại hai phương trình hệ ban đầu để tìm nghiệm hệ ban đầu

Bài toán kết thúc Bài tập tương tự:

 2  

4 12

6

x y x y x y

x y    

        

 

2

2 1

2

2

x y

x y x y

x x x

x y xy x y

    

      

  

  

          

 

  

 

6

6 10

2

x y x y

x x x x y xy

    

    

 

 

        

 

 

x y;      1; , 2;

2 2

yy

 2 2 1 1

yy  yx2y125

1

u x y   v x y  1 ,

u v

 

2x y xy   4 x y    1 x y

2 2 5 3 2

5 10

u v u v

u v u v

 

     

 

 

      

 

2

xy xy

 2

x y

 2  

4 12

6

x y x y x y

x y    

        

(104)

1. Giải hệ phương trình

Đáp số:

2. Giải hệ phương trình

Đáp số:

Câu

1) Phương trình tương đương với

ước + Giải (vô nghiệm)

+ Giải

+ Giải

+ Giải (vô nghiệm)

Vậy

Nhận xét Bài toán nghiệm nguyên giải phương phápđưa phương trình ước số Nhắc lại kiến thức phương pháp

• Phân tích đưa vềphương trình ước số

• Phân tích số thành tích ước số

• Cho thừa số chứa biến vếnày đồng với thừa sốở vế

2 4 1

3

x y y x xy y     

   



x y;     1; , 2; 1 

2 2 4 0

3

x x y y x xy y      

   



x y;     0; , 1; 1 

x y 1xy x y  2x y  1

x y 1xy x y 2

      

1

x y

  

1

2

x y x y xy x y xy

 

      

 

 

      

 

 

1

2 1

x y x y x xy x y xy y

  

           

  

  

          

  

  

1

2 1

x y x y x xy x y xy y

  

        

  

  

        

  

  

1

2

x y x y xy x y xy

 

        

 

 

       

 

 

x y;   1;1  , 1;

x y 1xy x y   5 2x y  x y 1xy x y 2x y

        

x y 1xy x y 2x y 1

        

x y 1xy x y 2 x y 1

        

x y 1xy x y 2

      

(105)

2) Ta có

Theo bất đẳng thức Cơ si

Theo bất đẳng thức Cô si Vậy giá trị nhỏ

Nhận xét: toán sử dụng bất đẳng thức Cosi dựa điểm rơi suy đoán kết hợp với điều kiện tốn để tìm giá trị nhỏ

Nhắc lại kiến thức phương pháp:

• Bất đẳng thức Cosi cho hai số thực dương

• Mở rộng đánh giá

Ý tưởng: Đây tốn có đối xứng rõ ràng nên ta mạnh dạn dự đoán điểm rơi Thay ngược lại giả thiết tốn, ta có Với điểm rơi , ta dễ dàng đánh giá vận dụng bất đẳng thức Cosi, vậy, khai khác giả thiết, ta suy ra:

Vậy nên, ta đánh giá biểu thức theo Hiển nhiên có số điểm rơi Vậy nên ta cần tìm thỏa mãn Mà bên trên, ta tìm đó, ta cần chứng minh Đánh giá ta có cách sau:

• Biến đổi tương đương, ta có:

• Sử dụng bất đẳng thức Cosi, ta có:

• Sử dụng đánh giá mở rộng nêu, ta có:

x1 y  1 4 xyxy3

1

3

2 2

x y x y

xy x y    x y

       

2

2 2

2

2

P x y x

y x

y x y

y x y x y

x

         

   

 

P x y 1

2

x y  xy

 2

2

, ; ; ;

a b

a b a b x y x y x y

   

,

x y

x y k   k12  4 k 1

x y 

x1 y  1 4 xyxy14

1

3

2 2

3 x y x y x y

xy x y        

    

P x2 y2 f x y 

yx   

2 x y 1 f x y   f x y  

2

x y  x2 y2 x y

yx  

     

2

2 2

0 x x y y x y 0

y

x x y x y y x y x

 

         

2

2

2

2 2

x y x y x

y y x y x y y x

y x y x y x x



   

    



   

 

 2

2

2 y x y

x x y y x x y

   

(106)

Bài toán kết thúc Bài tập tương tự:

1. Cho hai số thực dương thỏa mãn Tìm giá trị nhỏ biểu thức

2. Cho số thực dương thỏa mãn Tìm giá trị nhỏ biểu

thức

Câu

1) Vì đường kính suy (1) Vì đường kính suy (2) Từ (1) (2), suy thẳng hàng

Nhận xét Chứng minh ba điểm thẳng hàng ta quy chứng minh chúng thuộc đường thẳng

Nhắc lại kiến thức phương pháp

• Góc nội tiếp chắn nửa đường trịn góc vng

+ Góc góc nội tiếp chắn nửa đường trịn chia đơi đường kính đường trịn đường kính nên hay

+ Góc góc nội tiếp chắn nửa đường trịn chia đơi đường kính đường trịn nên hay

• Từ điểm nằm ngồi đường thẳng kẻ có đường thẳng vng góc với đường thẳng

Ta có từ ta kẻ vng góc với , suy hay ba điểm ; ; thẳng hàng

;

a b ab1

 

2

2 2

a b

P a b b a

   

; ;

a b c a b c  3

2 2

1 1

a b c P

b c a

  

  

MP PN MNMD DN MN

; ;

N P D

PNM PM

PM PNM 90 PN NM

DNM DM

( )O DNM 90 DN NM

N PN DN MN PN DN

(107)

2) Tứ giác nội tiếp ( ), suy (3)

Xét , ta có (4)

Từ (3) (4) , suy phân giác góc (*) Xét , ta có

Xét đường trịn đường kính có , nên phân giác góc (**)

Từ (*) (**), suy tâm đường tròn nội tiếp tam giác

Nhận xét Chứng minh điểm tâm đường tròn nội tiếp tam giác ta chứng minh điểm giao điểm hai đường phân giác tam giác

Nhắc lại kiến thức phương pháp

• Tứ giác có tổng hai góc đối diện tứ giác nội tiếp

Tứ giác có (góc nội tiếp chắn nửa đường trịn )

(góc ngồi đỉnh đối diện góc khơng kề với tứ giác nội tiếp) suy tứ giác tứ giác nội tiếp

• Hai góc nội tiếp chắn cung đường trịn

Tứ giác tứ giác nội tiếp nên (hai góc nội tiếp chắn cung đường trịn ) hay

+ (hai góc nội tiếp chắn cung đường trịn ) + (hai góc nội tiếp chắn cung đường tròn )

+ (hai góc nội tiếp chắn cung đường trịn đường kính ) , suy phân giác phân giác

• Giao điểm hai đường phân giác hai góc tam giác tâm đường tròn nội tiếp tam giác

Tam giác có phân giác phân giác , ta có cắt nên suy tâm đường tròn nội tiếp

Câu 4.Ta có

APQD PQD MAD900

  

PAQ PDQ NDM 

( )O NDM NAM

 

PAQ NAPAP NAQ

( )O AND AMD

MP QMP QNP   ANP QNPNP

ANQ

P ANQ

180

APQDDAP 90 ( )ODQP 90

  90 90 180

DAP DQP       APQD

APQD QDP QAP  QP

( )O PAQ NDM

 

NDM NAM MN ( )O

 

AND AMD AD ( )O  

QMP QNPQPPM

 

PAQ NAPAPNAPAND QNP ND

QNA

ANQ

AP NAPND QNA

(108)

Ta chứng minh

Bất đẳng thức tương đương với

Vì (1)

Vì (2)

Vì (3)

Từ (1), (2) (3), suy điều phải chứng minh

Vậy giá trị nhỏ

Đề s 16

Câu

1) Hệ phương trình cho tương đương với

     

 1 1 1

abc bc ba b abc ca cb c a b c

abc ab ac a

Q      

  

    

        

   

1 1 1

1 1

a b c b c a c a b

Q

a b c

       

  

1 1

a b c

Qabc

  

1aa1bb1c c1 1 12     

3 0

1 1c c 1bb 1a a 1

     

     

     

     

  

              

     

4

3 0

c b a c b a

  

     

3     1 3   2    1

4 c

c b

c b b a

   

    

       

   

   

    

   

3   2  1  

2

c b a

a

     

 

 

  

3  3 4 1   2 3 1    1

12 1 1

b c a b

c b c a b c

b c b a a

            

    

    

3; 3

12 1

c b c c b c b c

       

 

    

1 ;

6 1 b c a b b c a b

b a

         

 

   

;

2

a b c a a b c a

      

Q

12 a1;b2;c3

 

 

3 3

2

6 27

xy x y

x y x y xy x y

  



      

(109)

Vậy nghiệm hệ

Nhận xét: Bài toán sử dụng phép số từ phương trình vào phương trình cịn lại sau sử dụng đẳng thức tìm nhân tử

Nhắc lại kiến thức phương pháp:

• Hằng đẳng thức bậc ba

• Phương trình dạng

Ý tưởng: Ở hai phương trình hệ, biến nằm biểu thức bậc Và đặc biệt hai phương trình chứa số Vì số vào số phương trình rõ ràng ta thu phương trình bậc ba đẳng cấp hai biến

Cụ thể sau: (*)

Tuy nhiên, phương trình (*) phương trình đẳng cấp bậc ba ta chia phương trình cho , thu phương trình bậc ba Ta sử dụng máy tính cầm tay để giải phương trình Với , ngược lại phương trình ta tìm nghiệm hệ

Bài tốn kết thúc Bài tập tương tự:

1. Giải hệ phương trình:

Đáp số:

2. Giải hệ phương trình: Đáp số:

2) Điều kiện

Phương trình cho tương đương với +Với nghiệm

+ Giải

Đặt ta thu

 

   

 

   

3 3

3

2

3 3

xy x y xy x y

x y xy x y x y x y x y

 

     

 

 

 

       

 

 

 

 

 

2

1

3

x y xy x y

x y xy x y

x y x y

 

    

 

      

  

 

 

1

x y 

 3 3 3 2 2 2 2 3 3

3

ax by a xa bx yab xyb y

       

3 ; ; ; ;

f x yg x yf x yg x y

;

x y

;

x y

    3

9xy x y3  3xy x y 26x 2y

3 3 3 27 27 9

x x y xy y x x y xy y

       

  3 3

3

x y x y x y x y x y

         

3

y

x y

1

x y 

 

2 2

2

4

y x y

x xy x y

  



    



 ;  0; , 15; 15 , 15; 15

5 5

x y        

   

   

  

2

4

2

1 32

x y

x y xy    

   



1

x y 

4 x

  

 2

x x x

x    

0

x

 

4  x 2 x 4 4;

(110)

(thỏa mãn) Vậy phương trình có hai nghiệm

Nhận xét: Sử dụng phương pháp nhân liên hợp, sau đặt ẩn phụ tìm nghiệm tốn

Nhắc lại kiến thức phương pháp:

• Hằng đẳng thức

• Giải phương trình tổng qt dạng

Ý tưởng: Khơng khó để nhận thấy, phương trình có nghiệm Đồng thời vế trái phương trình có xuất biểu thức , dễ thấy

Vì thế, phương trình cho tương đương với: Phương trình cịn lại giải cách tổng quát nêu trên, giải cách đặt ẩn phự sau: ( ) Ta có hệ phương trình

Bài tốn kết thúc

Bài tập tương tự:

1. Giải phương trình Đáp số:

2. Giải phương trình Đáp số:

Câu

1) +) (mod 100)

(mod 100) (mod 100) (mod 100) (mod 100) (mod 100) (mod 100)

(mod 100) (mod 100)

 2

2

2

2

2 8

v u

u u u u u v

  

        

   

2; 14 2 96

4

5 5 25

2 ( )

u v x x u l

  

      

   

96

0;

25

xx 

   a b

a b a b a b a b

a b

      

   

f xg xm

   

     

   

     

2

2

;

2

f x g x m f x g x

f x g x m g x m f x g x m mg x

 

    

 

  

     

  

  

 

0

x

4

x 

 2 2   

4 4

xx   x  x 

 2 2 2 2 2 0

4

x x x x x x

x x

     

          

   



4

v x u x    

  

 u v; 0

2 8 2 14 2 96

;

5 5 25

2

v u u v x x v u

  

         

   

x 1 1 1  x 1 2x

24

0;

25

xx 

x 9 1 9  x 1 3x

0

x

 2

2

41  40 1 40 80 81 

4

41 81 802160 1  61

5

41 61.41

  60.40 100 1   1

 5 21 105

41 41

(111)

+) (mod 100) (mod 100)

Suy (mod 100)

Vậy chữ số cuối 42

Nhận xét Muốn tìm hai chữ số tận số (thường gặp lũy thừa) ta xét số dư số chia cho 100

Nhắc lại kiến thức phương pháp

• Hằng đẳng thức bình phương tổng

Ta có (mod 100)

• Tính chất đồng dư thức: (mod ) (mod ) với ; ; số ngun dương

Ta có

• Tính chất đồng dư thức: với , , số ngun dương

Ta có

Hồn toàn áp dụng

các kiến thức ta có

• Tính chất đồng dư thức:

Ta có

Khi

Vậy hai chữ số cuối số 2) Tập xác định

Ta có

Cộng hai bất đẳng thức ta thu Vậy

Nhận xét: toán sử dụng việc kết hợp đánh giá điểm rơi bất đẳng thức Cosi để tìm giá trị lớn biểu thức ban đầu

Nhắc lại kiến thức phương pháp:

• Bất đẳng thức Cosi cho hai số thực dương

• Giả sử điểm rơi toán, ta sử vận dụng đánh giá quen thuộc xung

quanh điểm rơi

4

57 1 572012 574 5031

 

106 2012

41 57 41

A    A

 2 2 2

a b aab b

 2

2

41  40 1 40 80 4.100 81 81 81     

a bnacbc n a b c n

   

2

41 81 mod100 41 81 mod100

 2  

2

81  80 1 8.100 160 60 61 mod100     

mod  mod 

a bnac bcn a b c n

   

4

41 61 mod100 41 61.41 2501 mod100 

 415 21 121 1 mod100  41105 1 mod100  41106 41 mod100 

      

 503

4 2012

57 1(mod100)57  57 1(mod100)

mod 

a bn c d modna c b d   modn

 

106

41 41 mod100 5720121 mod100 

 

106 2012

41 57 41 42 mod100 

106 2012

41 57

A  42

1 5

2 x

2 2

2 4x 3x

5 4x

2

x

x      

2

2x 1 3( 1) 2x 3( ) 3x

2

x

  

   

2

3 2x 4x

y  x  

ax

m

yx1

2

a b  ab m

 2

2 2 0

(112)

Ý tưởng: Biểu thức cho chứa hai thức bận hai, đồng thời yêu cầu tìm giá trị nhỏ nên ta đánh giá qua hai bước, là: sử dụng đánh giá cosi để khử thức, khéo léo biến đổi theo điểm rơi để khử dần biến số Vậy nên việc quan trọng dự đốn điểm rơi tốn, nhiên nói ta cần sử dụng Cosi cho thức một, nên ta đánh giá cho sau:

Do đó, suy Bây giờ, ta có hướng tư sử dụng đánh giá để khử hết biến Biểu thức cuối có xuất ta nghĩ nên đánh giá hay ngược lại Và ta chọn giải pháp

Khi đó, ta có: Và

muốn khử hết biến ta quan sát hai mẫu số phải tổng hệ số Chính thế, ta được:

Và từ đó, ta có đánh giá sau:

Bài toán kết thúc

Bài tập tương tự:

1. Cho số thực dương thỏa mãn Chứng minh rằng:

2. Chứng minh với ta có bất đẳng thức sau:

Câu

  2 

2 2

2

x

x x x  

      

   

2 2 2 2

2 4

5

2

m x n x m n x n mx n x x x

mn

   

    

   4 2 5

3

2

m n x n x

P

mn

 

 

 

2 ;

x x x x2

2 2 2 2

2

x k x k kx x

k

       

2 2 2

3

2

x k k k m n x n P

mn k

    

 

2

x

 

2

2 2

;

1

2 ;

k mn n m m n k x

x m x n x

        

 

 

      

 



2 2 1

3

2

x x x Px xx      

2 2

max 3

3 4

2 2

x x x

x    P

      

; ;

x y z xy yz zx  5

2 2

3x 3yz 10

0 x

2

(113)

1) Do , nên tứ giác nội tiếp

Do (do tứ giác nội tiếp) (do ), suy tứ giác nội tiếp, suy

Nhận xét Có nhiều cách để chứng minh điểm nằm đường tròn, có cách áp dụng tính chất “Có đường trịn qua ba điểm khơng thẳng hàng”

Nhắc lại kiến thức phương pháp

• Tứ giác có hai đỉnh liên tiếpcùng nhìn cạnh đối hai góc tứ giác nội tiếp

Tứ giác có đỉnh nhìn cạnh hai góc suy tứ giác nội tiếp

• Hai góc nội tiếp chắn cung đường trịn

hay (hai góc nội tiếp chắn cung đường trịn ngoại tiếp tứ giác )

• Hai dây cung song song đường tròn tạo hai dây bị chắn hai bên (Chứng minh dựa vào tính chất hình thang cân)

Trong đường trịn có hai dây cung song song nên ta có

• Hai góc nội tiếp chắn hai cung đường trịn

Trong đường trịn có góc nội tiếp chắn cung góc nội tiếp chắn cung Kết hợp với ta suy

Áp dụng kiến thức trên, tứ giác có hai đỉnh liên tiếp nhìn cạnh hai góc ( ), suy tứ giác nội tiếp

• Có đường trịn qua ba điểm khơng thẳng hàng

A

B C

O

N M

T

Q

P R

S

  900

TPA TQA  TAPQ

  

MTC QTP QAP  TAPQBAN MAC MN BC

MTAC T O( )

TAPQ P Q AT TPA TQA900

TAPQ  

QTP QAP MTC BAN QP

TAPQ

( )O MN BCBN MC

( )O BAN BNMAC

MC BAN MAC 

MTAC T A

(114)

Theo chứng minh ta có bốn điểm ; ; ; thuộc đường tròn mà ba điểm ; ; nằm đường tròn , nên suy điểm nằm đường tròn

2) Từ tứ giác nội tiếp ta có

Từ (1)

Mà tứ giác nội tiếp, suy (2)

Từ (1) (2), suy , suy tứ giác nội tiếp, ta có điều phải chứng minh

Nhận xét Có nhiều cách để chứng minh bốn điểm nằm đường trịn, đưa chứng minh bốn điểm tạo thành tứ giác nội tiếp thông dụng

Nhắc lại kiến thức phương pháp Kiến thức nhắc lại

(hai góc nội tiếp chắn cung đường tròn ngoại tiếp tứ giác )

(hai góc nội tiếp chắn cung đường tròn )

Suy

• Một đường thẳng cắt hai đường thẳng cho tạo hai góc vị trí đồng vị hai đường thẳng cho song song

Theo chứng minh ta có mà hai góc vị trí đồng vị hai đường thẳng suy kết hợp với giải thiết ta có

• Một đường thẳng cắt hai đường thẳng song song tạo cặp góc đồng vị Theo chứng minh ta có nên hai góc vị trí đồng vị

• Tứ giác nội tiếp có tổng hai góc đối diện

Tứ giác nội tiếp đường trịn nên ta có Kết hợp với trên, ta có

• Tứ giác có tổng hai góc đối diện tứ giác nội tiếp

Tứ giác có suy tứ giác nội tiếp hay bốn điểm ; ; ; thuộc đường tròn

Câu 4.Giả sử số tập hợp theo thứ tự (đánh số lại)

Ta có , suy tập

số thực phân biệt ta ln có giá trị phân biệt tổng

Vậy

Xét tập , với

M T A C

M A C ( )O T

( )O

TAPQPQA PTA CTA ABC    PQ BC MN 

 S

Q A NMA

AMNR  ARN AMN 1800

 S 1800

QRA Q A  ARQS

 

PQA PTAPATAPQ

 

CTA ABC AC ( )O

   

PQA PTA CTA ABC  

 

PQA ABC

QS BC QS BCMN BCQS BC MN 

QS MN  ASQ AMN

180

AMNR ( )O  ARN AMN 1800

  1800

ARN ASQ 

180

ARQS  ARN ASQ 1800 ARQS A

R Q S

X

1 n xx   x

1 n n n n n

xx  x x    x xxxxx   x  x n

n   1 n 2 2n3

i j xx

 

C Xn

 

1 1; 2; ;

(115)

Vậy

Số tổng ( ) , suy Xét tập , với

Giả sử tồn

Vậy

Đề s 17

Câu

1) Hệ phương trình tương đương với

+) Nếu , suy nên từ

; Do từ (mâu thuẫn)

+) Nếu , tuơng tự suy (mâu thuẫn) +) Nếu (thỏa mãn)

Đáp số:

Nhận xét: Bài toán sử dụng kỹ thuật đánh giá theo miền nghiệm đoán trước nghiệm hệ phương trình

Nhắc lại kiến thức phương pháp:

• Cho hai biểu thức , Xét biểu thức:

Có hai trường hợp sau xảy

• Kỹ thuật nhẩm nghiệm

Ý tưởng: Bài tốn khơng phải hệ phương trình đồng bậc, đồng bậc hai ta giải cách đưa hệ số bất định Nhưng điều đáng lưu ý tốn biểu thức gắn với hai đại lượng không âm Nên

3; 4; ; 1  1

i j

x    x i jn C Xn

 min

C X n

   

 

 

i j

xx 1  i j n  1

2

n n

  n n 2 1

C X  

 

2 2; ; ;2n

X   1  i j n

2i j i j

x   x

1 : 2r 2s 2i 2j

r s i j

r s n x x x x

         

     2   |2

2 2

2 |2

r i

j i

r s r i

i r

n n

r i s j C X

  

          



 max n n 2 1

C X

  

 

 

       

2

1

2 )

x y x y y x y x

     



     



  

  

2

1 (1)

2 1 (2)

x y y y x x

    

  

    



1

x x1y2 1 0 (1) 2  y 0

  

2

y y x

     

(2)   x  x 1

xx1

1

x  y

1;

xy

 ; 

f x y g x y ;  g x y ; 0

 ;   ; 

P f x y g x y

 

0 ;

P  f x yP 0 f x y ; 0

1;

(116)

nhiều khả xảy Xét trường hợp thấy

là nghiệm hệ phương trình Hoặc ta sử dụng kỹ thuật nhẩm nghiệm sau, giả sử , ta thay thử giá trị , tất nhiên lấy giá trị nguyên đẹp Và cho ta nghiệm Với cặp nghiệm này, thực chất toán quy giải hệ phương trình

Vì ta tách hệ phương trình ban đầu, nhóm nhân tử sau:

• Hệ phương trình cho

• Đến đây, ta đánh giá miền nghiệm:

TH1 Nếu

Hệ bất phương trình vơ nghiệm

TH2 Nếu

Hệ bất phương trình vơ nghiệm

• Vậy nghiệm hệ phương trình Bài tốn kết thúc

Bài tập tương tự:

1. Giải hệ phương trình Đáp số:

2) Điều kiện

Phương trình tương đương Chia hai vế cho , ta

+ Giải

+ Giải

Đáp số

Nhận xét: Bài toán sử dụng phương pháp ẩn phụ khơng hồn tồn, sau nâng lũy thừa tìm nghiệm phương trình ban đầu

Nhắc lại kiến thức phương pháp:

x1y2 y 2x20 x y;    1; 2

x kk

kx y;    1;

3

1

x y x y y x y x

                                       2 2

1

1

2 2 1

x y y x y x y

y x y x y x x                                         2

1 1 2

1

2 2

x y y y x

y x y

                               2

1 1 2

1

2 2

x y y y x

y x y

                     1;

xy

   

2

2

3

x y x y y x y x

    



    



   x y;  1;3

x

 

2 x x x

x

   

0

x

1

2 x x

x x x  

     

 

 

 

3 2 1 0

x x

x x x x    

 

        

   

3 2 0

x x

x x x

                    

3 2 4 4 3 0

3

x x x x x

x x x                

3 3 4 0

x x x x x x x x

        

x 1x2 x 4 0 x 1

      

(117)

• Phương pháp đặt ẩn phụ khơng hồn tồn: xét phương trình bậc hai có dạng (*), ẩn phụ biểu diễn dạng Khi đó, ta có

, với bắt buộc số phương Nên ta tìm nghiệm (*)

• Cách giải phương trình

Ý tưởng: Trước hết, ta cần quy đồng mẫu số toán, ta phương trình có dạng nâng lũy thừa hai vế, ta thu phương trình bậc Và phương trình bậc khơng có nghiệm ngun khó để giải Vậy nên ta cần nghĩ đến hướng tư khác, tốn xuất thức nên ta mong muốn tạo lượng để đưa phương trình bậc hai, sau đặt để sử dụng phương pháp ẩn phụ khơng hồn tồn Tức chia hai vế phương trình, ta được:

• Ta có

(*)

• Đặt , ta có (*)

Có nên suy

• Giải ( ), ta có ( )

• Giải ( ), ta có ( )

Bài tốn kết thúc Bài tập tương tự:

1. Giải phương trình:

Đáp số:

 .2  . 0

mf x tng x t k  t t h x  

 2  

tng xkmf x

    t

 

  t  ;    t  

ng x ng x

t h x t h x

mf x mf x

     

   

          2 

;

f x g x f x g x

f x g x

             

f x g xh x

3 x xk x x              t x x  

 

2 3

2

7

2( 1) x x x x

x x

x x     

  

1 3

2 x x x x

x x x x x x x

                          0 t x x

   t2 1 t

x x             2

' 1 1

t x x x

                     

1 2 2

1

2

1 3 2

1 x i t t x x x t

t x x ii

x x x x                                   

i i 4 3 0

3

x x x x

x x              

ii ii

2

3 3 4 0 1

x x x x x x

        

x3 4x12x28x

 

4 ; 31

(118)

2. Giải phương trình:

Đáp số:

Câu

1) Giả sử tồn số nguyên thỏa mãn

Ta có với số nguyên

Mâu thuẫn với (1)

Vậy không tồn thỏa mãn đẳng thức

Nhận xét Để giải toán cần sử dụng phương pháp phản chứng: “Giả sử xảy biến đổi thấy điều mâu thuẫn với giả sử”

Nhắc lại kiến thức phương pháp

• Thêm lượng vào hai vế đẳng thức ta đẳng thức tương đương với đẳng thức ban đầu

• Lũy thừa bậc bốn số nguyên chia cho dư tức với số nguyên

• Hai vế đẳng thức có số dư chia cho số nhận số dư khác đẳng thức khơng thể tồn

Ta thấy chia dư hoặc

Mà chia dư khơng thể tồn đẳng thức

hay

Vậy không tồn ba số nguyên thỏa mãn đẳng thức

2) Phương trình tương đương với

+) Nếu (mâu thuẫn nguyên)

+) Nếu nghiệm, ta suy nghiệm, mà mâu thuẫn

+) Nếu (thỏa mãn) Vậy nghiệm

Nhận xét Để giải toán cần sử dụng phương pháp biến đổi tương đương đưa xét khoảng giá trị nghiệm

3 2 2

x  x x  x

1 5; 65

2

x  x 

, ,

x y z

4 7 5 4 8 5

xyz  xyzz  (1)

4 0,1 (mod 8)

aa

4 4

0,1,2,3 (mod 8) 5(mod 8)

x y z z

    

 

  

x y z; ; 

4 7 5 4 8 5

xyz  xyzz

 

4 0, mod8

aa

 

 

 

 

 

4

4 4

4

0, mod8

0, 1, 2, mod8 0, mod8

8 5 mod8 0, mod8

x

x y z y

z z

 

 

    

 

  

 

   

 

  

4 4

VT x yz

4

8

VPzx4y4z48z45

4 4 8 5

xyzz

x y z; ; 

4 7 5

xyz

  2  2  2 2 3

1 1

x x x x y

         

   

      

2 3

2x 4x y 8x 8x y

     

 3  3  3

3 3

1 8 2

x  xxxx  xyxy

1

x  x y;  x; y   x

0

x  y

0

x y 

(119)

Website:tailieumontoan.com

118

Nhắc lại kiến thức phương pháp

• Hằng đẳng thức

• Hằng đẳng thức

Khi phương trình cho tương đương với phương trình Giải phương trình nghiệm nguyên cách xét khoảng giá trị nghiệm

• Khơng tồn lũy thừa bậc ba hai lập phương (lũy thừa bậc ba) liên tiếp +) Nếu

(mâu thuẫn nguyên và hai lập phương liên tiếp)

+) Nếu

(mâu thuẫn nguyên và hai lập phương liên tiếp) +) Nếu (thỏa mãn)

Vậy nghiệm

Câu

1) Tứ giác nội tiếp phân giác góc , suy , nên tam giác cân , (1)

Tứ giác nội tiếp (cùng bù với góc ) (2)

Trong tam giác có vừa đường cao vừa đường phân giác nên tam giác cân

Do , suy tam giác cân , nên Từ (1), (2) (3), suy (c – g – c)

  

2

ABA B A B 

  4 4   2  2  2 2

1 1 1

VTx  x  x  x    x  x 

   

 2 2 2

A B AAB B

 2 1 2 1 2 1 2 1

VTxx xxxx xx

2x2 2 4 x 8x3 8x

   

3

8x 8x y

 3 3

1 8

x  xxxx  2x 3y32x13

y  2x  3

2x1

 3  3  3

3 3

1 8 2

x  xxxx  xyx

y  2x 2x13

0

x  y

0

x y 

OBCD CO BCDOBD OCD OCB ODB

OBD

O OB OD

OBCD ODC OBE OBC

CEF

CO

CEF

C

  

AB CF AEB AFC EAB  ABE B BE BA CD  (3)

OBE ODC

(120)

Nhận xét Có ba trường hợp hai tam giác Ở này, sử dụng trường hợp “cạnh-góc-cạnh” từ ta tìm cạnh góc

Nhắc lại kiến thức phương pháp

• Hai góc nội tiếp chắn cung đường tròn

+ (hai góc nội tiếp chắn cung đường tròn ngoại tiếp tam giác )

+ (hai góc nội tiếp chắn cung đường trịn ngoại tiếp tam giác )

Mà (vì tia phân giác ), suy

• Tam giác có hai góc tam giác cân

Tam giác có (chứng minh trên) nên cân

• Tam giác cân có hai cạnh bên Tam giác cân suy

• Tứ giác nội tiếp có góc ngồi đỉnh góc đỉnh khơng kề với Tứ giác nội tiếp đường trịn ngoại tiếp có góc ngồi đỉnh góc đỉnh khơng kề suy

• Tam giác có đường cao đường phân giác tam giác cân

Tam giác có vừa đường cao vừa đường phân giác nên tam giác cân

• Tam giác cân có hai góc kề cạnh đáy

Tam giác cân , suy hay

• Một đường thẳng cắt hai đường thẳng song song tạo hai góc vị trí đồng vị

(hai góc vị trí đồng vị ), suy , nên cân (tam giác có hai góc nhau), mà hình bình hành nên

suy

Xét có: ; ; suy (c – g

– c)

2) Từ

Mà đường cao tam giác cân , suy

Từ , tâm đường tròn ngoại tiếp tam giác

Nhận xét Đường tròn ngoại tiếp tam giác cách ba đỉnh tam giác ta chứng minh điểm cách điểm ; ; hay

Nhắc lại kiến thức phương pháp

• Hai tam giác có cặp cạnh cặp góc tương ứng

• Tam giác cân có đường cao đường trung trực cạnh tương ứng đường cao tam giác cân nên đường trung trực

• Một điểm thuộc trung trực đoạn thẳng cách hai mút đoạn thẳng

 

BCO ODBOB

BCD

 

OCD CBDDO

BCD

 

BCO OCDCO BCDOBD ODB

OBD

OBD ODB OBD O

OBD

O OB OD

BCDOBCD EBDB

CDO D B EBD CDO 

CEF

COCEF

C

CFE

C CEF CFE AEB AFC

 

AFC EABAB FCAEB EAB  EBA

B BE BAABCD

AB CDBE CD

OBE

 ODC OB ODEBD CDO  BE CD OBE ODC

OBE ODC OE OC

    

COCEF OE OF

OE OC OF  OCEF

O E C F OE OC OF 

OBE ODC OE OC

    

(121)

Điểm thuộc đường trung trực đoạn thẳng nên , suy

• Điểm cách ba đỉnh tam giác tâm đường trịn ngoại tiếp tam giác Ta có suy tâm đường tròn ngoại tiếp

3) Theo trên, ta có mà Ta có đường phân giác góc , nên

Mà trung trực , suy

Từ hai đẳng thức trên, suy

Nhận xét Chứng minh đẳng thức ta kết hợp đẳng thức cho, chứng minh để ghép vào đẳng thức cần chứng minh

Nhắc lại kiến thức phương pháp

Ta có (chứng minh trên) ( cân ) suy suy

• Đường phân giác tam giác chia cạnh dối diện thành hai đoạn thẳng có tỷ số với tỷ số hai cạnh tương ứng tam giác

Ta có phân giác góc tam giác nên , suy ta

được

• Nhắc lại kiến thức

Ta có nằm trung trực đoạn thẳng nên suy nhân vế theo vế với đẳng thức ta

Câu 4. Ta chứng minh

(đúng) Ta chứng minh (2)

Ta có

O CO FE OE OF

OE OC OF 

OE OC OF  OCEF

BE CDCE CF BC DF

CI BCD

IB CB DF IB BE ID DF ID CD  BE 

CO EF I COIE IF

IB BE EI ID DF FI

BE CDCE CF ECF C

CE BE CF CD   BC DFCB DF

CDBE

CI BCD CBD IB CB

ID CD

IB DE BE BI DE DI

IDBE  

I CO FE IE IF

BE BI DE DIBE BI IE DE DI IF

3

3 8 2

x x

xyxy (1)

 

3

3 8 2 2

2

x x x y x y

 

     

2

2 2 8

x y x x y

    4x y2 24y48xy3

2 2

x y xy

  

 

3

3 2

3

y y x y y  x y  

   

3

3

3 2

y y y x y x y

 

      

3 2

x y y yx y

      

     

2

2 2

x y y y x y

    

 2 2 2 2  3

x y x y y x y

    

 2

2 2

(122)

(đúng) Từ (1) (2), suy

Dấu “=” xảy Vậy

Nhận xét: Bài toán sử dụng phương pháp dự đốn điểm rơi, từ phát tư bất đẳng thức phụ cần thiết để tìm giá trị nhỏ toán

Nhắc lại kiến thức phương pháp:

• Bất đẳng thức Cosi cho hai số thực dương

• Hệ bất đẳng thức Cosi, là:

Ý tưởng: Đây toán chứa biểu thức đồng bậc, nên điểm rơi tốn có dạng Từ thay ngược lại biểu thức , ta có:

Các biểu thức xuất căn, nên ta mong muốn số phương trình nhau, nên dễ thấy Tức điểm rơi toán giá trị nhỏ Việc dự đốn điểm rơi cần thiết, giúp ta có nhiều lựa chọn việc đánh

hay Với điểm rơi đó, hai mẫu số nên có đánh giá , ta cần đánh giá hai hai biểu thức mẫu, ví dụ mẫusố Bây giờ, quan sát thức một:

• Với , thức mẫu số có bậc ba, tử bậc nhất, để đồng hóa bậc ta cần đánh giá thức biểu thức dạng bậc không bậc Hơn lại xuất nên ta chọn đánh giá để tối thiểu hóa ẩn , tức ta cần chứng minh:

Nhưng điều chưa hồn tồn đúng, cần phải có điều kiện , nên hướng tư chưa Tức ta lựa chọn biểu thức thay ta chọn, thế:

 

2 3 2 2 2 2 2

xyxyyxyyy x y

 2 2 2 2 1 2    3

3

2

x y x y x y y x y y x y

        (2)

1

P

x y

min

P

2

x y  xy

 2

2 2

xyx y

x kyP

 3  3

3

3 8

1

4

8 1 1

k k k

k k k k

P  

 

 

  

 3 8; 1 1

k   k

 3 8 1 1

k    k k3 8 m2

1

m k      

x yP

2

x y  xy 2 1 2

xyx yP P

2 2

2 ; ; ;

xy x y x  y xy

  3

3

3

;

8

x f x y x

x y x

x y

 

3

8y x2y

y

 2 3 3   

3

1 2 8 2 0

2

8

x x x y x y y y x x y x y

xy           y x

2 2

xy x2y

 2 22 2 3 3

3 8 2 2

x x x x y x x y xyxy    

 2

2 0

xy x y

(123)

• Với , với hướng tư tương tự, có:

Điều ln theo bất đẳng thức Cosi, ta có:

Bài toán kết thúc

Bài tập tương tự:

1. Cho hai số thực dương thỏa mãn Tìm giá trị nhỏ biểu thức

2. Cho hai số thực dương Chứng minh rằng:

Đề s 18

Câu

1) Điều kiện

Phương trình tương đương với

Nếu , đồng thời ,

suy VT VP (loại)

Thử lại ta thấy nghiệm

Nhận xét: toán kết hợp phương pháp nhân liên hợp phương pháp đáng giá để tìm nghiệm phương trình

Nhắc lại kiến thức phương pháp:

• Biểu thức liên hợp

với

• Đánh giá: với

 

  3  3

3 3

4

; y

g x y y

x y y x y y

y

 

  

      

2 3

2 2 2

3 3

2 3

2

4 y x y x y y x y x y

x y y

y         

 2

2

2 2 2

2

3 2

x y x y

x y x y y xy y    



       

 2 2 2 2  3

x y x y y x y

    

;

a b a b 2

2 2

3 3

2 3 3

a b b a P

a b b a

 

 

 

;

a b

2 3 2 2

4 2

a b b a a b

     

          

     

     

     

0 x

   

3 1 1 1 3 1 1 3

3 x x x x

x  x         

 

0  x 1  x xx 3 1 3 

1

x

  

x m x   x m  x x m  x m

x m x

x m x

   

  x0;x m 0

     

m f x  n g xh x  

2

m f x n m

  

(124)

Ta có: , suy phương trình vơ nghiệm Vậy nghiệm phương trình cho

Ý tưởng: Bài tốn xuất ba thức nằm tích, khó để định hình hướng giải, ẩn phụ phức tạp Nhưng xét hai thức ta thấy Vì ta nghĩ đến chuyện dùng đẳng thức dạng

Khi phương trình cho tương đương với: ( )

Với phương trình ( ), ta nhẩm vài giá trị nghiệm đẹp thỏa mãn yêu cầu biểu thức thức số phương ta khẳng định có nghiệm , đồng thời lại miền chặn biến ta đánh giá phương trình ( ) Tức với ta chứng minh ( ) vô nghiệm sau:

( ) vô nghiệm Vậy ta kết luận nghiệm phương trình cho Bài tốn kết thúc

Bài tập tương tự:

3. Giải phương trình

Đáp số:

4. Giải phương trình

Đáp số: 2)

+ Xét nghiệm

+ Xét hệ phương trình tương đương với

Thay (1) vào (2) ta thu

Nhận xét: toán sử dụng phép chia biến, sau kết hợp hai phương trình tìm mối liên hệ hai biến để tìm nghiệm hệ phương trình

Ý tưởng: quan sát thấy hệ phương trình có dáng dấp hệ phương trình đối xứng loại hai, tức xuất tổng tích làm ta nghĩ đến phép đặt Viet để từ giải hệ hai ẩn Nhưng làm ta thu hệ

   

   

0

2

m f x n n f x m

g x h x m

   

    

 

 x m

   2

3

x  x

  

2

a   b a b a b

 

3 1 1 1 3 1 3 3

3 x x x x

x  x          i

i

1 x

1

x

i 0 x i

3 3

0

3

x

x VT VP

x x

   



        

 i

1

x

x 2 x1 2  x 1

2

x

x 1 x2 3  x 1

3 x

0

x y 

0;

xy

2 2

1 2 1 2 (1)

1 1 4 1 2 8 (2)

x y x y

x y xy x y xy

 

 

     

 

 

 

 

     

     

               

     

     

 

 

3 1

1 8

1 1

x y x y

xy    

  

    

  

  

   

 

1

x y

  

x yxy

;

(125)

, hệ giải phương pháp thu phương trình lũy thừa bậc phức tạp, ta nghĩ đến phương án khác phương pháp “ chia để trị “ Trước hết xét phương trình hai, vế trái xuất tích đồng thời vế phải xuất tích số dạng Do ta nghĩ đến việc chia biểu thức bên vế trái cho ta sau: Phương trình chủ đạo hai biến ta chia phương trình để xuất hai biến này, Và đặt

hệ phương trình cho trở thành

Bài toán kết thúc Bài tập tương tự:

1. Giải hệ phương trình

Đáp số:

2. Giải hệ phương trình

Câu

1).Ký hiệu

Ta có

suy khơng biểu diễn dạng lập phương số nguyên dương

Nhận xét Áp dụng kiến thức phần nguyên, quan hệ phần nguyên số với số   2 2

S P P

S P P

        xy xy

xy 1 1

x y xy                   

1 1;

x y

2

1 2

xy

1 ax by            

2 2 2 2

1 2

a b a b

a b ab a b ab

                                 2 2 2 2

2 8

a b a b

a b a ab b a b                       

2 2 1 1

1 1

2

a b a b x y x y a b                         2 2

1 27

1 10

x y xy x y xy

   



   



 ;  1; , 2; 2 , 2 3;

2

x y       

        3 2 1 16 1 x y xy x y xy                                      

3 1 , 27

K n  

 

  n1 K

3

3 1 1

27 3 27

Kn    K K   n K 

   

3 1 2

3 27 27 27

K

K K n K K K

         

 3

3 3 1

3 3

K

K n K K K K K n K K

            

2 3 1

27

n K  n  n  

 

(126)

Nhắc lại kiến thức phương pháp

• Áp dụng giả thiết tốn đề tìm điều kiện

Ta có số nguyên dương nên

• Giá trị phần ngun số khơng vượt qua số

nên hay

• Một số thực có giá trị lớn giá trị phần nguyên nhỏ số nguyên liền sau số nguyên biểu diễn phần nguyên số thực

Ta có

Tiếp tục biến đổi giải tađược

• Khơng tồn lập phương hai lập phương liên tiếp

Ta có hai lập phương liên tiếp Mà nên viết dạng lập phương số nguyên dương

Vậy với số nguyên dương , biểu thức không biểu diễn dạng lập phương số nguyên dương

2).Ta có

, suy

Đẳng thức xảy Vậy

Nhận xét: Bài toán sử dụng phép từ giả thiết bất đẳng thức Cosi cho hai số thực dương đểtìm giá trị nhỏ biểu thức

Ý tưởng: Quan sát thấy, tốn có sựđối xứng hai kiến nên điểm rơi xảy Thế lại giả thiết ta tìm Giả thiết cho

đồng thời số xuất biểu thức nên ta sẽnghĩ đến chuyện giả thiết vào Khi ta có:

n n1 1

27

n  

3 1 27

n   1

27 n

 

  

 

 

 

  K1

3 1 1 1

27 27 27

n n n

   

      

   

   

      

3 1

27

K n K

     

 3

3 1

K  n KK

3

KK13 K3 n K2K13 n K

n

2

3 1

27 n  n 

 

 

 

  

   

6 x 5  y 5  z 5

        

6 x y x z y z y x z x z y

        

           

3

2 2

x y  x zx y  y zz x  z y

  

 

9 3 3 2

2

xyz x y z

   

     

3 2

3

6 6

x y z P

x y z

 

 

    

1;

x y  z

min 23

P

;

x y

x y kz  x y 1;z2 xy yz zx  5

(127)

Và điều ta cần sử dụng đánh giá đểtriệt tiêu tử số mẫu số, tức tìm số thỏa mãn:

Thì lúc Câu chuyện tìm , quan sát thấy biểu thức tích hai thừa sốdương, ta sử dụng bất đẳng thức Cosi cho hai số thực dương phải thỏa mãn điều kiện điểm rơi Với thức cuối, với điểm rơi

thì ta có

Cũng với tư đó, ta thấy:

Nên

Tức

Bài toán kết thúc Bài tập tương tự:

1. Cho thỏa mãn Chứng minh rằng:

2. Cho thỏa mãn Chứng minh rằng:

Câu

        

3

6

x y z P

x y x z y z y z z x z y

 

       

m

          

6 x y x z   y z y z   z x z y  m x3 3y2z

1

P m

m

x y    z x z y

z x z y   x y 2 2z

      

      

3

6

2

3

6

2

x y x z x y x z

y z y x y z y x

  

  

  

  

         3 

6 3

2

x y x z   y z y z   z x z y   xyz

min

2

3

P Px y 1; z2

; ;

a b ca b c  1

1

bc ca ab P

a bc b ca c ab

   

  

; ;

a b ca b c  1

3

2

bc ca ab P

a bc b ca c ab

   

(128)

1) Tứ giác nội tiếp , suy , nên tứ giác nội tiếp

Tương tự tứ giác nội tiếp

Vậy điểm thuộc đường tròn

Nhận xét Chứng minh năm điểm thuộc đường tròn ta chứng minh cho bốn điểm tạo thành tứ giác nội tiếp

Nhắc lại kiến thức phương pháp

• Hai góc nội tiếp chắn cung đường trịn

(hai góc nội tiếp chắn cung đường trịn ngoại tiếp tứ giác )

• Một đường thẳng cắt hai đường thẳng song song tạo cặp góc so le

(hai góc so le ), suy

• Tứ giác có góc ngồi đỉnh đối diện góc khơng kề với tứ giác tứ giác nội tiếp

Tứ giác có góc góc ngồi đỉnh góc góc đỉnh không kề với thỏa mãn nên suy tứ giác nội tiếp

Hoàn toàn tương tự ta chứng minh tứ giác nội tiếp Do điểm thuộc đường trịn

2) Do tứ giác nội tiếp nên ta có , suy tứ giác nội tiếp

Mà , suy tứ giác nội tiếp đường tròn Vậy thuộc đường tròn

Nhận xét Chứng minh điểm nằm đường tròn ta chứng minh điểm ba điểm nằm đường tròn tạo thành tứ giác nội tiếp

Nhắc lại kiến thức phương pháp

• Tứ giác nội tiếp có góc ngồi đỉnh góc đỉnh khơng kề với

K

Q

N M

P

I

B C

A D

BPIM AD BC MAD BPM BIM AMID

DNIA

; ; ; ;

A M I N D ( )K

 

BPM BIMBM

BPIM

 

BPM MADBP ADBIM MAD

MIDA BIMI MAD

A I BIM MAD MIDA

DNIA

; ; ; ;

A M I N D

BPIM CPIN QMI BPI CNI  MINQ

; ; ( )

M I N KMINQ ( )K

(129)

+ Tứ giác tứ giác nội tiếp có góc ngồi đỉnh góc đỉnh khơng kề với nên

+ Tứ giác tứ giác nội tiếp có góc ngồi đỉnh góc đỉnh khơng kề với nên , suy

• Áp dụng nhắc lại kiến thức

Tứ giác có góc góc ngồi đỉnh góc góc đỉnh không kề với thỏa mãn nên suy tứ giác nội tiếp

• Qua ba điểm không thẳng hàng xác định đường tròn

Bốn điểm ; ; ; thuộc đường tròn mà ba điểm ; ; thuộc đường tròn nên suy thuốc đường tròn

3) Khi thẳng hàng, kết hợp với thuộc đường trịn ta có (đối đỉnh); (do tứ giác nội tiếp)

(đối đỉnh); (do tứ giác nội tiếp)

, suy phân giác nên phân giác góc

Do

Nhận xét Chứng minh đẳng thức tỷ số đoạn thẳng ta thường sử dụng mối liên hệ đoạn thẳng thơng qua định lý Ta-lét, tính chất đường phân giác trong/ngồi tam giác, tính chất tỷ lệ thức

Nhắc lại kiến thức phương pháp

• Hai góc đối đỉnh

+ Góc góc hai góc đối đỉnh nên + Góc góc hai góc đối đỉnh nên

• Áp dụng nhắc lại kiến thức

+ (hai góc nội tiếp chắn cung đường tròn ngoại tiếp tứ giác )

+ (hai góc nội tiếp chắn cung đường tròn ngoại tiếp tứ giác ), suy

• Một tia nằm hai tia chia góc tạo hai tia thành hai góc tia tia phân giác

Ta có nằm mà suy phân giác góc

• Hai góc đối đỉnh có chung đường phân giác hay tia phân giác góc tia đối tia phân giác góc

+ phân giác góc + tia đối tia

Góc góc hai góc đối đỉnh, suy phân giác góc

• Đường phân giác tam giác chia cạnh dối diện thành hai đoạn thẳng có tỷ số với tỷ số hai cạnh tương ứng tam giác

Ta có phân giác góc tam giác nên

BPIM M I

 

QMI BPI

CPIN P N

 

BPI CNIQMI CNI 

MINQ INCN QMI

M N INC QMI  MINQ

M I N Q M I N

( )K Q ( )K

; ;

P I Q Q ( )K AIQ PIC 

 

PIC PNCNIPC

 

PNC QNDQND QID INDQ

 

AIQ QID

  IQ DIAIPBIC

PB IB ID IB ID BD PB BD

PC IC IA IC IA AC PC CA

     

AIQ PIC  AIQ PIC

PNC QNDPNC QND

 

PIC PNCPC

PINC

 

QND QIDQDQNIDAIQ QID

IQ IA ID AIQ QID IQAID IQAID

IQ IP

AID BICIP BIC

IP BIC CBI PB IB

(130)

• Định lý Ta-lét

Ta có nên , suy

• Tỷ lệ thức (điều phải chứng minh)

Câu 4. Giả sử có số, xếp chúng theo thứ tự (1)

Suy với ta có (2), với

Áp dụng kết ta thu ; ; ; ; ; , suy tập phải có phần tử

+) Giả sứ Vì

Vì (mâu thuẫn)

+) ta có tập thỏa mãn u cầu tốn Đáp số:

Nhận xét Các toán dạng chủ yếu đánh giá tư khơng có cách giải tổng quát Nhắc lại kiến thức phương pháp

• Từ đến 100 có 100 số tự nhiên Tập hợp tập hợp tập có phần tử nhỏ lớn 100 nên tập hợp không vượt 100 phần tử

Tổng quát, tập hợp có phần tử, xếp phần tử theo thứ tự

• Theo đề có với x, a, b thuộc tập hợp nên ta có

ta có với

• Áp dụng kết ta

; ; ; ; ; ,

suy tập phải có phần tử

• Giả sử theo thứ tự giả sử ta suy

• Áp dụng kiến thức trên, cách tính tốn giải phương trình bậc + Vì

+ Vì

+ Vì (mâu thuẫn)

• Vì tập hợp có phần tử mà xét trường hợp có phần tử cho kết mâu thuẫn nên tập hợp có phần tử

Với theo thứ tự giả sử ta suy từ ta tìm tập hợp thỏa mãn yêu cầu toán

Đáp số:

BC ADIB IC IB ID

ID IA ICIA

PB IB ID PCICIA

PB IB ID IB ID BD PB BD

PC IC IA IC IA AC PC AC

     

A n 1x1x2x2xn100

1,2,3, , 1

k  nxk1   x xi j xk xk2xk ,i j k

 2 x2  1 x3  2 x48 x516 x632 64

xA

8

8 100 n x

6 32 64 96 7 50

xx    xxx

5 16 32 48 6 25

xx    xxx  16 24 25 5 252

xx     xxx

n 1; 2; 3; 5; 10; 20; 25; 50; 100

9 n

A

A A 2 n 100

1 2

1xxx xn100

x a b  A x a

x b      

1,2,3, , 1

k  nxk1   x xi j xk xk2xk ,i j k

1

k i j k k k

x    x x x xx

2 1

x    x3  2 x48 x516 x632 x764

A 8

nx8100

6 32 64 96 7 50

xx    xxx  16 32 48 6 25

xx    xxx  16 24 25 5 252

xx     xxx

A

A

9

nx9100

1,2,3,5,10,20,25,50,100

(131)

Đề s 19

Câu

1) Cộng hai vế hệta (2x +3y)2 =25

Ta có hệ:

   = + = + 2 5 3 2 2 y x y x    = + − = + 2 5 3 2 2 y x y x

2) Điều kiện

2 − ≥

x

Đặt +1= ( ≥0); −2 +1= ( >0)

b b x x a a x

Ta có (1- b)(a - 3) = B =

2 ;

1 = x =

x ; a = thìx3 =4

Câu

1) Phá ngoặc

( )( ) ( )( ) ( )( ) 25 ) )( ( 25 ) ( 25 ) ( ) ( 25 1 2 2 2 = + + ⇔ = + + + ⇔ = + + + + + + ⇔ = + + + + + + y x y x xy y x xy y x xy xy y x xy y x

Vì x, y khơng âm nên (x + 1)(y + 1) = ta có (x; y) = (0; 4) ; (4; 0)

2) Xét 1( )

1 1 ) ( ) ( ) ( ) ( 2 N k k k k k k k k k k k k k k k k ∈ + + − = + + = + + + + = + + +

Thay k từ ta có

( ) n n

(132)

1) Ta có

o

o

2 2 2

AC

cot ACB AC AB.cot 30 3R

AB

AB AB

sin ACB BC 4R

BC sin 30

1 1 1

AH R

AH AB AC 12R 4R 3R

∠ = ⇔ = =

∠ = ⇒ = =

= + = + = ⇒ =

2) Ta có ∠ACB = ∠HAB (cùng phụ với ∠CAH) Mà ∠HAB = ∠HNB (cùng

2 số đo cung HB)

HBN ACB

∠ = ∠

Từ tứ giác CMNH nội tiếp Tâm đường trònnội tiếp CMNH thuộc trung trực HC cố định

Câu

Áp dụng bất đẳng thức Bu-nhi-a-cốp –xki ta có (a2;1) (1; 4) ta có

2 1 "

:" );

1 ( 17

4 1

) 4 (

) 1 (

17

2

2

4 + ≥ + ⇔ + ≥ + = ⇔ =

a Dau

a a

a a

(b2;1) (1; 4) ta có

2 1 "

:" );

1 ( 17

4 1

) 4 (

) 1 ( 17

2

2

4 + ≥ + ⇔ + ≥ + = ⇔ =

b Dau

b b

b b

Từ (1) (2) ta có (*) 17

8

2

2 + +

a b

P Mặt khác Từ GT ta có

4 5

= +

+b ab

a

(133)

2 " :" ; ) ( ) ( 4 2 2 2 2 = = ⇔ = ≥ + ⇔ = + + ≥ + + ⇔          ≥ + ≥ + ≥ + b a Dau b a ab b a b a ab b a b b a a

Thay Vào (*) ta có

2 17 17 8 2 1 = + ≥ P Vậy 2 1 2 17 )

(P = ⇔ a = b= Min

Đề s 20

Câu I

1) Điều kiện: x ≥ −

Với x = nghiệm phương trình

Với x > vếtrái lớn Phương trình vơ nghiệm Với x < 1, vếtrái nhỏhơn Phương trình vơ nghiệm Vậy nghiệm phương trình x =

2) Giải hệphương trình

( )( )

2 2

2

5x 2y 2xy 26 5x 2y 2xy 26

3x 2x 2xy xy y 11

3x 2x y x y 11

 + + =  + + =  ⇔   + + − =  + − + − =     ( ) 2

2 2

2

5x 2y 2xy 26

5x 2y 2xy 2x 3x y xy 26 2.11 48

2x 3x y xy 11

 + + =

 ⇒ + + + + − − = + =

 + − − =



2 x

9x 6x 48 8

x  =  ⇔ + − = ⇔  = − 

Với x = ta có:

2 2 y

2.2 3.2 y 2y 11 y 2y

y

 = + − − = ⇔ + − = ⇔  = −



(134)

2

2

8 8 43

2 y y 11 y y

3 3

   

− + − − + = ⇔ − + =

   

    Phương trình vơ nghiệm

Vậy nghiệm hệphương trình (2; 1); (2; -3)

Câu II

1) Giả sử n2 +391 = a2

với a nguyên dương Ta có

( )( )

( )

n a n 195

(L)

n a 391 a 196

n a n a 391

n a 391 n 195

TM

n a a 196

 + =   = −

  

− = − =

  

− + = − ⇔  ⇔ 

 + =  =

 

  

  − = −  =

 

Vậy sốnguyên dương n thỏa mãn đềbài 195 2) Ta có

( )

2 xy z x y z x y

xy z 2x 2y

1 xy xy

+ + + + +

+ + +

+ +

(x z y z)( ) x y xy z x y

1

1 xy xy

+ + + + + + +

≥ ≥ =

+ +

Dấu “=” xảy x y z = = =

Câu III

1) Chứng minh M trực tâm tam giác ABC

(135)

1 2

H P P H

∠ = ∠ = ∠ = ∠ mà ∠H2 = ∠C1 (cùng phụ với∠QHC)

1

H C

∠ = ∠ nên CM // EH ⇒ CM ⊥ ABtương tự BM ⊥ AC Vậy M trực tâm tam giác ABC

2) Chứng minh BEFC tứ giác nội tiếp

EBH HPF

∠ = ∠ (cùng bù với góc∠HPE)

HPF PFA EBH PFA

∠ = ∠ ⇒ ∠ = ∠

Vậy tứgiác BEFC nội tiếp

Câu IV

Số sốđược đánh dấu ≥

Nếu tất sốđược đánh dấu sốdương ta có đpcm

Nếu sốđánh dấu có sốâm giả sử anthì số an 1+ sốdương đánh dấu n n

a +a + > 0, sốâm có sốcó tổng dương, cặp sốnày khơng trùng Vậy tổng sốđược đánh dấu dương

Đề s 21

Câu

1)ĐKXĐ:∀xR

1 : ; 0 ) ( 1 ) 1 ( 1 1 2 2 2 2 = = ⇔ = − ⇔ = + − ⇔ = − + − ⇔ = + + − − + − ⇔ + − = + − x hoac x x x x x x x x x x x x x x x

Phương trình có nghiệm x1= 0;x2 =

2)              − = =    = =    = = ⇔           = − =    = − + − = ⇔         − = = − − = ⇔    = − + − − − = ⇔     = + − − + − − = ⇔     + = + = + − 1 0 2 1 ) )( ( 3 2 2 2 2 2 2 y x y x x y y y x y y y x y x x xy x y y x x xy x y y x xy x xy x y y y x xy y x

Vậy hệ có nghiệm (x;y)= (1; 0); (1; 1); (5;-3)

Câu

1) Ta có

( )13 13 3(mod10);6 6(mod10);2009 (2009) 2009 9(mod10)

1313 = ≡ ≡ 2009 = 2008 ≡

) 10 (mod 2009

(136)

nên 13 2009 2009

13 + + có tận

2)áp dụng BĐT Cô si cho số dương 9a 4a+5b ta có

2 13 ) (

9a a+ ba+ a+ b = a+ b

9b 4b+55 ta có

2 13 ) (

9b b+ ab+ b+ a = b+ a Nên 18 18 ) ( = + + ≥ b a b a

P nên Min(P)=

3

1khi a = b

Cách khác áp dụng bất đẳng thức Bunhicôpsky dãy

b

a; và dãy 4a+5b; 4b+5a ta có

( ) ) ( ) ( ) ( ) ( ) ( ) 9 )( ( ) ( ) ( ≥ + + + + ⇔ + ≤ + + + ⇔ + + ≤ + + + a b b b a a b a b a a b b b a a b a b a a b b b a a Min (P)=

ab b ab a a b

b a b a b a b a b a b

a+ = + ⇔ + = + ⇔ + = + ⇔ =

5 5 5 Câu

a-Tam giác BKO1dd tam giác O2KA nên

1 1 TgB BK K O AK K O A O B O b a = = = =

tgB1

BH

AH = nên

BH AH b a = b-Theo a ta có HA

a b HB=

trong ∆ vng BO2H ta có BH2+O2H2=O2B2

nên ) ( : ) ( 2 ) ( ) ( 2 2 2 2 2 2 2 2 2 2 2 2 2 b a ab BH suyra b a b a AH b a AH b a AH b AH a b AH bAH AH a b b AH bAH b AH a b b AH b AH a b + = + = ⇔ = + ⇔ = + − ⇔ = + − ⇔ = + − + ⇔ = − +

B O1

O2 K

C A

(137)

2 2 3 ) ( b a b a BH AH SABCD + = = Câu

Ta có a b ab a b a b a b 2a 3b

2 ) )( ( 14

3 2+ 2+ = + + ≤ + = +

c b c b c b c b cb c

b

2 ) )( ( 14

3 + + = + + ≤ + = +

a c a c a c a c ac a

c

2 ) )( ( 14

3 + + = + + ≤ + = +

Ta có Q a c c c b b b a a P ca a c c bc c b b ab b a a P = + + + + + ≥ + + + + + + + + = 3 14 14 14 2 2 2 2 2 2

Áp dụng BĐT Bunhiacopsky cho dãy a c c c b b b a a ; ;

2 + + + 2a+3b; 2b+3c; 2c+3a

Ta có ( )

5 ) ( ) 5

( a b c a b c Q a b c

Q + + ≥ + + ⇔ ≥ + + ( )

5 c b a Q

P≥ ≥ + +

Dấu “=”xảy khi a=b=c

Cách khác: Chứng minh Q ( )

1

c b a+ +

≥ áp dụng BĐT Cô-Si

25 25 5 25 3 2 b a b a a b a a a b a b a a − = + − ≥ + ⇔ ≥ + +

+ tương tự

25 ; 25 2 a c a c c c b c b

b ≥ −

+ − ≥ + Vậy 25 25 25

8a b b c c a a b c

Q≥ − + − + − = + +

Đề s 22

Câu

1)ĐKXĐ:x≥−1

Đặt x+35=a.; x+1=b;(a>0;b≥0)

   = = ⇔    = = ⇔ = − − ⇔ = − − + ⇔ + + + = + + + ) / ( ; 195 ) / ( ; 1 14 6 0 ) 14 )( 6 ( 0 84 6 14 35 36 84 1 6 35 14 m T x m T x b a b a ab b a x x x x

(138)

1 2 + = k k

Sk ; ta phải chứng minh với n=k+1 nghĩa

1 ) ( ) ( 2 + + + = + k k Sk 2 2 2 2 ) ( ) ( ) ( ) ( ) ( ) ( ) ( ) ( ) ( ) ( + + − + = + − + + + ⇔ + + − + + + = + + + ⇔ + + − + + = + k k k k k k k k k k k k k k S

Sk K

Ta có :

[ ] [ ] ) ( ; ) ( 24 32 16 4 ) ( ) ) ( ) ) ( ) ( ) ( ) ( ) ( 4 2 2 2 2 2 2 2 DPCM k k k k k k k k k k k k k k k k k k k k k k k k k + + + = + + + + − − − − + + + + + = + + + + + − + + = + − + + + Cách khác:

đặt a = 2n - 1( n∈N*) xét tổng quát       + + − + − =

+ 2

1 2

4 a4 a2 a a2 a

a thay n từ

1 ;2;3;4;… Ta có a 1;3;5;7;… Ta có 4 ) 1 ( 1 ) 1 ( 13 5 1 2 2

2 + = − − + + = +

+ − − + − + − = n n n n S Câu

1) Với n < khơng có số thoả mãn

Với n = thoả mãn Với n>6 n có dạng n = 7k;7k+1;7k+2;7k+3;7k+4;7k+5;7k+6 (( *)

N k∈ n = 7k n+77; n=7k+1 n+137; n=7k+2 n+57

n=7k+3 n+257; n=7k+4 n+177; n=7k+5 n+377; n=7k+6 n+17 Vậy có n=6 thoả mãn đề

2) Ta thấy a-b7;(a+c)-(b+d)=((a-b)+(c-d)) 7 mà 2014-844=1170=167x7+1

Không chia hết cho sau số hữu hạn lần thay ta nhận tập hợp cặp số M1 ={(2018,702),(844,2104),(1056,2176),(2240,912)}

Câu

1) ta có ∠DCB=∠DAB (Chắn cungBD)

(139)

Nên ∠DCB=∠QPB(1)

∠DBC=∠PAQ( bù ∠DAC)

∠PAQ =∠PBQ ( chăn cung PQ) Nên ∠DBC=∠ PBQ(2)

Từ(1) ;(2) ta có

∆BCD đ d với ∆BPQ (g.g)

2)theo a) ∠DCB=∠QPB nên tứgiác KPCB nội tiếp nên đường tròn ngoại tiếp tam giác KCP qua điểm B cốđịnh M thay đổi

Câu

Giả sử x,y,z số thực thoả mãn điều kiện

2 , ,

0≤x y z≤ x + y + z =

Tìm giá trị nhỏ lớn biểu thức :

(1 )(1 )(1 ) 12

4 4

z y x

z y x

M = + + + − − −

Giải: cách đặt 1-x = a;1 - y = b;1 – z = c a + b + c = nên a3 + b3 + c3 = 3abc

−1≤a;b;c≤1

M=(1-a)4+(1-b)4+(1-c)4+12abc=a4+b4+c4-4(a3+b3+c3-3abc)+6(a2+b2+c2)-4(a+b+c)+3

M= a4+b4+c4+6(a2+b2+c2)+3

áp dụng BĐT Bunhiacopsky cho dãy a;b;c 1;1;1ta có 3(a2+b2+c2)≥(a+b+c)2=0

Nên a2+b2+c2≥0 Dấu “=” xảy a=b=c=0

áp dụng BĐT Bunhiacopsky cho dãy a2;b2;c2và 1;1;1 ta có 3(a4+b4+c4)≥(a2+b2+c2)2=0⇔

a4+b4+c4≥0 Dấu “=” xảy a= b = c =

Vậy M≥3 suy (M)= a=b=c=0 x=y=z=1

Mặt khác a2+b2+c2=(1-x)2+(1-y)2+(1-z)2=3-2(x+y+z)+x2+y2+z2= x2+y2+z2-3

Giả sử 0≤xyz≤2; ta có x2+y2+z2-3=(x+y)2-2xy+z2 -3≤(3-z)2 +z2-3=2z2-6z+6

x2+y2+z2-3=2z2-6z+6= 2

2 3

2

≤ +     

 −z 0<z≤2 suy a2+b2+c2≤2

ta có a2;b2;c2 ∈[ ]0;1 nên (a4+b4+c4)≤( a2+b2+c2) suy M≤7(a2+b2+c2)+3≤17

Max(M)=17 (a;b;c)=(-1;0;1) hoán vị hay (x;y;z) =(0;1;2)và hoán vị Cách 2: thay x4+y4+z4=(x2+y2+z2)2-2(x2y2+y2z2+z2x2)

Mà x2+y2+z2=(x+y+z)2-2(xy+yz+zx)=9-2a (đặt xy+yz+zx=a)

x2y2+y2z2+z2x2=(xy+yz+xz)2-2xyz(x+y+z)=a2-6xyz

12(1-x)(1-y)(1-z)=12(1-x-y-z+xy+yz+zx-xyz)=12a-12xyz-24

Nên M=(9-2a)2-2(a2-6xyz)+ 12a-12xyz-24=81-36a+4a2-2a2+12xyz+12a-12xyz-24

M=2a2-24a+57=2(a-6)2-15 (*)

Ta có x2+y2+z2 ≥xy+yz+zx⇔(x+y+z)2 ≥3(xy+yz+zx) nên a≤3

Ta có (2-x)(2-y)(2-z) ≥0⇔8-4(x+y+z)+2(xy+yz+zx)-xyz≥0

⇔xy+yz+zx≥2+xyz≥2 hay a≥2 2≤a≤3 thay suy -4≤(a-6) ≤-3 Hay 16 ≥(a-6)2≥ vào (*) 3≤M≤17

Vậy Min(M)= a=3 khiđó x=y=z=1;

Ngày đăng: 24/02/2021, 12:06

Từ khóa liên quan

Tài liệu cùng người dùng

Tài liệu liên quan